geometria lineal per al batxillerat - toomates.net · la història dels vectors. 1.10 recopilatori...

162
GEOMETRIA LINEAL 2n Batxillerat Amb tots els problemes PAU 1998-2019 Gerard Romo Garrido

Upload: others

Post on 23-Sep-2019

32 views

Category:

Documents


1 download

TRANSCRIPT

Page 1: Geometria Lineal per al Batxillerat - toomates.net · La història dels vectors. 1.10 Recopilatori d’exercicis. 1.11 Taula resum dels tres productes. 2 Varietats lineals. Rectes

GEOMETRIA LINEAL

2n Batxillerat

Amb tots els problemes PAU 1998-2019

Gerard Romo Garrido

Page 2: Geometria Lineal per al Batxillerat - toomates.net · La història dels vectors. 1.10 Recopilatori d’exercicis. 1.11 Taula resum dels tres productes. 2 Varietats lineals. Rectes

Toomates Cool·lección Los documentos de Toomates Cool•lección son materiales digitales y gratuitos. Son digitales porque están pensados para ser

consultados mediante un ordenador, tablet o móvil. Son gratuitos porque se ofrecen a la comunidad educativa sin coste alguno. Los

libros de texto pueden ser digitales o en papel, gratuitos o en venta, y ninguna de estas opciones es necesariamente mejor o peor que las otras. Es más: Suele suceder que los mejores docentes son los que piden a sus alumnos la compra de un libro de texto en papel,

esto es un hecho.

Lo que no es aceptable, por inmoral y mezquino, es el modelo de las llamadas "licencias digitales" con las que las editoriales pretenden cobrar a los estudiantes, una y otra vez, por acceder a los mismos contenidos (unos contenidos que, además, son de una

bajísima calidad). Este modelo de negocio es miserable, pues impide el compartir un mismo libro, incluso entre dos hermanos, pretende convertir a los estudiantes en un mercado cautivo, exige a los estudiantes y a las escuelas costosísimas líneas de Internet,

pretende pervertir el conocimiento, que es algo social, público, convirtiéndolo en un producto de propiedad privada, accesible solo a

aquellos que se lo puedan permitir, y solo de una manera encapsulada, fragmentada, impidiendo el derecho del alumno de poseer todo el libro, de acceder a todo el libro, de moverse libremente por todo el libro.

Nadie puede pretender ser neutral ante esto: Mirar para otro lado y aceptar el modelo de licencias digitales es admitir un mundo más

injusto, es participar en la denegación del acceso al conocimiento a aquellos que no disponen de medios económicos, en un mundo en el que las modernas tecnologías actuales permiten, por primera vez en la historia de la Humanidad, poder compartir el

conocimiento sin coste alguno, con algo tan simple como es un archivo "pdf".

El conocimiento no es una mercancía.

El proyecto Toomates Cool•lección tiene como objetivo la promoción y difusión entre el profesorado y el colectivo de estudiantes

de unos materiales didácticos libres, gratuitos y de calidad, que fuerce a las editoriales a competir ofreciendo alternativas de pago atractivas aumentando la calidad de unos libros de texto que actualmente son muy mediocres, y no mediante retorcidas técnicas

comerciales.

Este documento se comparte bajo una licencia “Creative Commons”: Se permite, se promueve y se fomenta cualquier uso,

reproducción y edición de todos estos materiales siempre que sea sin ánimo de lucro y se cite su procedencia.

Todos los documentos se ofrecen en dos versiones: En formato “pdf” para una cómoda lectura y en el formato “doc” de MSWord para permitir y facilitar su edición y generar versiones parcial o totalmente modificadas. Se agradecerá cualquier observación,

comentario o colaboración a

[email protected]

Actualmente Toomates Cool·lección consta de los siguientes documentos:

GA Geometría Axiomática (Teoría)

pdf 1 2 3 ... 22 23 portada

PG Problemas de Geometría pdf 1 2 3 4 5 6 7

AG Àlgebra (Llibre de text)

pdf 1 2 3 4

PA Problemas de Álgebra (en preparación)

pdf doc

GN Geometria analítica (Llibre de text)

pdf doc

TR Trigonometria (Llibre de text)

pdf doc

PT Problemas de Trigonometria pdf doc

CO Nombres complexos (Llibre de text)

pdf doc

PC Problemas con números complejos pdf doc

DE Desigualdades (Teoría y problemas)

pdf doc

AL Àlgebra Lineal 2n Batxillerat

pdf doc

GL Geometria Lineal 2n Batxillerat

pdf doc

CI Càlcul Infinitesimal 2n Batxillerat

pdf 1 2

PL Programació Lineal 2n Batxillerat

pdf doc

PT Compendium PAU TEC 1998-2019 pdf

PS Compendium PAU CCSS 1998-2019 pdf

PM Problemas de Matemáticas pdf doc

Versión de este documento: 23/12/2019

www.toomates.net

Page 3: Geometria Lineal per al Batxillerat - toomates.net · La història dels vectors. 1.10 Recopilatori d’exercicis. 1.11 Taula resum dels tres productes. 2 Varietats lineals. Rectes

Índex

1 Punts i vectors. → 1.1 Vectors. Àlgebra vectorial.

1.2 Punts. Les dues operacions entre punts i vectors. 1.3 Mòdul d’un vector. Distància entre dos punts.

1.4 Vectors proporcionals. Punts alineats. 1.5 Producte escalar de dos vectors. Angle. 1.6 Producte vectorial de dos vectors. Àrea.

1.7 Producte mixt de tres vectors. Volum. Punts coplanaris.

1.8 Fora de programa: L’esfera. 1.9 Notes històriques. La història dels vectors.

1.10 Recopilatori d’exercicis.

1.11 Taula resum dels tres productes.

2 Varietats lineals. Rectes i plans. → 2.1 Dependència i independència lineal de vectors. 2.2 Rectes. Equacions de la recta.

2.3 Plans. Equacions del pla.

2.4 Punt d’intersecció entre recta i pla. 2.5 Recta intersecció de dos plans.

2.6 Rectes coplanàries.

2.7 Resolució de problemes mètrics amb rectes parametritzades.

3 Paral•lelisme. → 3.1 Paral·lelisme entre dues rectes. 3.2 Paral·lelisme entre dos plans.

3.3 Paral·lelisme entre recta i pla.

3.4 Pla paral·lel a dues rectes i que passa per un punt. 3.5 Pla que conté una recta i és paral·lel a una altra.

3.6 Recta paral·lela a dos plans.

4 Perpendicularitat. → 4.1 Perpendicularitat entre plans.

4.2 Perpendicularitat entre rectes. 4.3 Perpendicularitat entre recta i pla.

4.4 Projecció ortogonal d’un punt en una recta. Punt simètric respecte d’una recta. 4.5 Projecció ortogonal d’un punt en un pla. Punt simètric respecte d’un pla.

4.6 Recta perpendicular comú a dues rectes.

4.7 Problemes PAU de perpendicularitat i paral·lelisme.

5 Distància. → 5.1 Distància entre punt i recta. 5.2 Distància entre punt i pla.

5.3 Distància entre dos plans.

5.4 Distància entre recta i pla. 5.5 Distància entre dues rectes.

5.6 Problemes PAU amb distància, projecció ortogonal i punt simètric.

6 Posició relativa. → 6.1 Posició relativa entre recta i pla.

6.2 Posició relativa entre dos plans. 6.3 Posició relativa entre dues rectes.

6.4 Posició relativa de tres plans.

7 Angles. → 7.1 Angle entre dues rectes.

7.2 Angle entre dos plans. 7.3 Angle entre recta i pla.

8 Àrea i volum. → 8.1 Àrea de figures en l’espai.

8.2 Volum de figures en l’espai.

9 Recopilacions d’exercicis. →

10 Apèndix. → 10.1 Taula Paral·lelisme-Perpendicularitat-Angle.

10.2 Taula d’aplicacions de la projecció ortogonal.

10.3 Algunes demostracions. 10.4 Problemes PAU de geometria del pla.

Solucions. →

Page 4: Geometria Lineal per al Batxillerat - toomates.net · La història dels vectors. 1.10 Recopilatori d’exercicis. 1.11 Taula resum dels tres productes. 2 Varietats lineals. Rectes

Links d’interès. →

Page 5: Geometria Lineal per al Batxillerat - toomates.net · La història dels vectors. 1.10 Recopilatori d’exercicis. 1.11 Taula resum dels tres productes. 2 Varietats lineals. Rectes

1 Punts i vectors.

1.1 Àlgebra vectorial.

1.1.1 Definició. Vectors de IR3.

Un vector de IR3 és una terna ordenada de tres nombres reals: ),,( 321 vvvv

Els nombres 321 ,, vvv s’anomenen coordenades o components del vector.

Els vectors es designen, generalment, amb una lletra minúscula coronada amb una petita

fletxa al capdamunt.: ...,, twv

Per exemple: )4,1,2(v

, )1,0,3( w

.

1.1.2 Observació. Interpretació gràfica d’un vector.

Un vector en IR3 representa una direcció en l’espai, un moviment en línia recta, i per

tant es representarà per una fletxa.

1.1.3 Definició. Suma de vectors.

Dos vectors es poden sumar sumant els seus components corresponents. La

representació gràfica de la suma de vectors es basa en la llei del paral·lelogram.

Exemple: )7,2,1(v

i )1,4,5( w

, llavors )8,2,4()17,)4(2,51( wv

1.1.4 Proposició. Propietats de la suma de vectors.

Associativa: wvuwvu

Commutativa: uvvu

Existència de vector nul: vv

0 on )0,0,0(0

Existència de vector oposat: 0)(

vv , on si ),,(),,( 321321 vvvvvvvv

1.1.5 Observació. Interpretació gràfica de la suma de vectors.

Hem dit abans que un vector representa una direcció en l’espai. La suma de dos vectors

representa la direcció resultant de la combinació de les seves direccions.

Visualment la podem representar dibuixant el segon vector a continuació del primer:

1.1.6 Definició. Resta de vectors.

Per restar dos vectors wv

se suma a al primer el oposat del segon: wvwv

.

A la pràctica, restem component a component.

Exemple: )7,2,3( v

i )2,5,1(w

, llavors )5,7,4()27,52),1(3( wv

1.1.7 Multiplicació d’un vector per un escalar.

Anomenem escalar a qualsevol nombre real. Per a multiplicar un nombre per un vector

s’ha de multiplicar cada component del vector per aquest nombre. És fàcil comprovar

que aquesta operació multiplica la longitud del vector pel nombre. Si el signe del

nombre és negatiu, s’obté un vector de la mateixa longitud però en sentit contrari.

Exemple: )7,2,3( v

i 3k , llavors )21,6,9()7,2,3(3 vk

Page 6: Geometria Lineal per al Batxillerat - toomates.net · La història dels vectors. 1.10 Recopilatori d’exercicis. 1.11 Taula resum dels tres productes. 2 Varietats lineals. Rectes

1.1.8 Propietats del producte de vectors per escalars.

Associativa : wbawba

Distributiva (I): wbwawba

)(

Distributiva (II): wavawva

Producte per la unitat : ww

1

Producte per zero: 00

w

1.1.9 Exercicis.

1. Calcula vu

, on )3,4,1( u

i )2,3,5(v

.

2. Calcula vu

, on )1,0,3(u

i )3,2,4( v

.

3. Determina el vector ,u1.3 v

on u

= ( 2 , -3 , -5 )

4. Determina el vector ,u0.7 v

on u

= ( 1 , 4 , -2 )

Page 7: Geometria Lineal per al Batxillerat - toomates.net · La història dels vectors. 1.10 Recopilatori d’exercicis. 1.11 Taula resum dels tres productes. 2 Varietats lineals. Rectes

1.2 Punts. Les dues operacions entre punts i vectors.

1.2.1 Definició. Punts.

Un punt de IR3 és la representació matemàtica d’una posició en l’espai.

Analíticament ve representat per una terna ordenada ),,( 321 pppP , igual que els

vectors. Per anomenar els punts fem servir lletres majúscules: P, Q, R, ...

Per exemple: )1,2,4( P , )0,2,3(Q .

1.2.2 Observació. Representació gràfica de punts.

Els punts es representen per posicions en l’espai, marcades amb una petita “boleta”.

Atenció! Analíticament els punts i els vectors es representen de la mateixa manera: amb

matrius 1x3. Tanmateix no hem de confondre mai aquestes dues entitats matemàtiques.

No és el mateix anar en la direcció (1,3,-2) que estar a la posició (1,3,-2) . La primera

idea es representa amb un vector (1,3,-2)v

, i la segona amb un punt (1,3,-2)P .

Els punts no admeten operacions: No es poden sumar punts, ni es pot multiplicar un

punt per un escalar.

1.2.3 Definició. Primera operació entre punts i vectors: Punt+Vector=Punt.

Donat un punt P i un vector v

, definim el punt final vPQ

com el punt a on

arribem si sortim de P i anem en la direcció de v

:

),,(),,(

),,(332211

321

321vpvpvpvPQ

vvvv

pppP

Observació: La translació de punts és la única operació entre punts i vectors, i és la base

del que s’anomena “Geometria Afí”.

1.2.4 Definició. Segona operació entre punts i vectors: Punt-Punt=Vector.

Donats dos punts P i Q, existeix un únic vector v

tal que vPQ

.

Aquest vector s’anomena PQ , i es calcula restant component a component:

),,(),,(

),,(332211

321

321pqpqpqPQPQ

qqqQ

pppP

Gràficament serà la direcció que prenem quan anem des del punt inicial P al punt final

Q.

Atenció: L’ordre importa, no és igual PQ que QP , de fet són contraris l’un de l’altre:

QPPQ

El vector PQ s’anomena també “vector de desplaçament”.

Page 8: Geometria Lineal per al Batxillerat - toomates.net · La història dels vectors. 1.10 Recopilatori d’exercicis. 1.11 Taula resum dels tres productes. 2 Varietats lineals. Rectes

1.2.5 Proposició. Divisió d’un segment.

Donats dos punts en l’espai ),,( 321 pppP i ),,( 321 qqqQ , podem trobar el seu punt

mig R:

PQPR2

1

Important! En comptes de memoritzar la “fórmula del punt mig”:

2,

2,

2),,(

),,(332211

321

321 qpqpqp

qqqQ

pppP

és molt més interessant estudiar i aprendre el mètode:

PQPR2

1

perquè amb el mètode estem entenent què fem i com podem dividir el segment PQ en

qualsevol nombre de parts, o de qualsevol manera.

Exemple:

Determina el punt P que divideix el segment AB en una proporció 6/5 , on

A=(-2,0,6) i B=(10,-6,-12).

Solució: P=(8,-5,-9)

1.2.6 Definició. Simètric d’un punt respecte d’un altre.

Donats dos punts en l’espai ),,( 321 pppP i ),,( 321 qqqQ , podem trobar el punt R

que fa que Q sigui el punt mig entre P i R:

PQPRqqqQ

pppP2

),,(

),,(

321

321

Page 9: Geometria Lineal per al Batxillerat - toomates.net · La història dels vectors. 1.10 Recopilatori d’exercicis. 1.11 Taula resum dels tres productes. 2 Varietats lineals. Rectes

1.2.7 Exercicis.

1. Representa en l’espai els següents punts:

a) 1,1,4

b) 3,1,1

c) )1,1,3(

d) )3,2,2(

Page 10: Geometria Lineal per al Batxillerat - toomates.net · La història dels vectors. 1.10 Recopilatori d’exercicis. 1.11 Taula resum dels tres productes. 2 Varietats lineals. Rectes

2. Determina les coordenades dels següents punts:

a)

b)

c)

d)

1.2.8 Problemes PAU.

1. Un segment d’origen en el punt A = (–1, 4, –2) i extrem en el punt B està dividit en

cinc parts iguals mitjançant els punts de divisió A1, A2, A3 i A4 (vegeu la figura). Si

sabem que A2 = (1, 0, 2), quines són les coordenades de B?

Solució PAU CAT TEC JUNY 2005 1.4

2. a) Si A, B i M són tres punts de l'espai que compleixen la relació AMAB 2

digueu quin serà el valor de r a l'expressió MBrMA

b) Si la relació anterior entre vectors s'hagués produït al pla i les coordenades de A i B

fossin respectivament (3, –5) i (–5, 7), quines serien les coordenades del punt M?

Justifiqueu la resposta.

PAU CAT TEC SET 1997 1A.2

Page 11: Geometria Lineal per al Batxillerat - toomates.net · La història dels vectors. 1.10 Recopilatori d’exercicis. 1.11 Taula resum dels tres productes. 2 Varietats lineals. Rectes

1.3 Mòdul d’un vector. Distància entre dos punts.

1.3.1 Definició. Mòdul d’un vector.

Definim el mòdul (o “longitud”) d’un vector ),,( 321 vvvv

per

2

3

2

2

2

1 vvvv

1.3.2 Proposició. Propietats bàsiques del mòdul d’un vector.

a) 0v

b) 00 vv

c) vkvk

1.3.3 Proposició. La desigualtat triangular de Cauchy-Schwarz.

La norma de la suma de vectors és sempre menor o igual que la suma de les normes de

cadascun dels vectors:

vuvu

Demostració. Vegeu apartat 10.3.

Observació.

La desigualtat triangular de Cauchy-Schwarz garanteix que en tot triangle la longitud

d’un costat és sempre menor que la suma dels seus altres dos costats, o dit amb paraules

senzilles, que la línia recta és sempre el camí més curt entre dos punts.

1.3.4 Definició. Distància entre dos punts.

La distància entre dos punts ),,( 321 pppP i ),,( 321 qqqQ serà el mòdul del vector

que determinen:

2

33

2

22

2

11 )()()(),( pqpqpqPQQPdist

1.3.5 Definició. Longitud d’un segment.

La longitud d’un segment PQ d’extrems ),,( 321 pppP i ),,( 321 qqqQ serà la

longitud entre els seus extrems: 2

33

2

22

2

11 )()()( pqpqpqPQPQ

1.3.6 Exercicis.

1. Determina la distància entre els punts P = ( 14 , 1 , 17 ) i Q = ( 9 , -4 , 13 ).

2. Determina la distància entre els punts P = ( 4 , 2 , 7 ) i Q = ( -16 , -10 , 3 )

Page 12: Geometria Lineal per al Batxillerat - toomates.net · La història dels vectors. 1.10 Recopilatori d’exercicis. 1.11 Taula resum dels tres productes. 2 Varietats lineals. Rectes

1.3.7 Problemes.

1. Els punts A= ( 5 , -1 , 1 ) , B= ( 7 , -4 , 7 ) , C= ( 1 , -6, 10 ) i D= (-1 , -3 , 4 ) són

coplanaris. Demostra que formen un rombe.

2. a) Demostra que els punts ),2,( A , )0,,2( B i )2,0,( C són els

vèrtexs d’un triangle isòsceles.

b) Determina el valor de per al qual el triangle ABC és equilàter.

3. Determineu el punt de l’eix Y que està a una distància 10 del punt )3,2,1(

4. Determina el punt de l’eix X que és equidistant als punts )2,2,3(A i

)4,5,5(B

5. Demostra que els punts )10,7,0(P , )6,6,1( Q i )6,9,4( R formen

un triangle rectangle isòsceles.

(Nota: demostra que és un triangle rectangle mitjançant el Teorema de Pitàgores)

6. Determina el punt de l’eix Z que equidista dels punts A=(2,3,-4) i B=(6,-2,-1)

)2,0,0(P

1.3.8 Problemes d’equidistància a tres punts.

Determina les coordenades del punt del pla 0x (és a dir, és de la forma ),,0( zyP )

que equidista dels tres punts A=(2,0,3), B=(0,3,2) i C=(0,0,1)

)3,1,0(3,163

03

63214469

03)2()3()00()3()0()20(

2222

222222

Pzyzy

yz

zyzzyzzyyPCPB

yzzyzyPBPA

1. Determina les coordenades del punt del pla 0z (és a dir, és de la forma

)0,,( yxP ) que equidista dels tres punts A=(3,2,-5), B=(5,1,7) i C=(4,-3,-6).

Page 13: Geometria Lineal per al Batxillerat - toomates.net · La història dels vectors. 1.10 Recopilatori d’exercicis. 1.11 Taula resum dels tres productes. 2 Varietats lineals. Rectes

1.4 Vectors proporcionals. Punts alineats.

1.4.1 Definició. Vectors proporcionals.

Donats dos vectors ),,( 321 vvvv

i ),,( 321 wwww

, direm que són proporcionals quan

existeixi un IRk tal que wkv

, és a dir, quan podem passar d’un a l’altre

multiplicant per un escalar:

Per exemple,

)7,2,5( v

i )21,6,15( w

són proporcionals perquè vw

3 .

)3,1,2(v

i )11,4,8( w

no són proporcionals, perquè )2)(4(8 ,

1)4(4 però 3)4(11 .

Visualment dos vectors seran proporcionals quan determinin una mateixa direcció en

l’espai, independentment del seu mòdul i sentit.

1.4.2 Proposició.

Dos vectors u

i v

són proporcionals si i només si les raons de les seves components

són iguals:

3

3

2

2

1

1

321

321

),,(

),,(

v

u

v

u

v

uvku

vvvv

uuuu

Observació: Aquesta caracterització pot ser molt útil, però té l’inconvenient de què ens

podem trobar amb zeros als denominadors, cosa que fa mal als ulls.

Per exemple: Els vectors )2,0,1(u

i )4,0,2(v

són proporcionals perquè uv

2 .

Dividint components apareix un 0/0: 5.04

2

0

0

2

1

1.4.3 Definició. Punts alineats.

Direm que tres punts ),,( 321 aaaA , ),,( 321 bbbB i ),,( 321 cccC estan alineats quan

els vectors AB i AC siguin proporcionals: ABkAC .

1.4.4 Exercicis.

1. Determina si els punts )1,2,1( A , )0,3,2(B i )4,0,1(C estan o no alineats.

2. Determina si els punts )4,2,3(A , )2,1,6(B i )6,3,12(C estan o no alineats.

3. Donats els punts )1,0,2( A , )2,2,3( B i )4,6,( xC , determina el valor de x

que fa que estiguin alineats.

Page 14: Geometria Lineal per al Batxillerat - toomates.net · La història dels vectors. 1.10 Recopilatori d’exercicis. 1.11 Taula resum dels tres productes. 2 Varietats lineals. Rectes

1.4.5 Problemes.

1. Calcula els valors de a i b per als què els punts )1,1,1(A , ),2,( baB i

)0,0,1(C estiguin alineats.

2. Demostra que els punts P=(2,4,6), Q=(-2,-2,-2) i R=(6,10,14) estan alineats.

3. Determina els valors de a i c per als quals el punt (a,1,c) pertanyi a la recta

determinada pels punts (0,2,3) i (2,7,8).

Page 15: Geometria Lineal per al Batxillerat - toomates.net · La història dels vectors. 1.10 Recopilatori d’exercicis. 1.11 Taula resum dels tres productes. 2 Varietats lineals. Rectes

1.5 Producte escalar de dos vectors. Angle.

1.5.1 Definició. Producte escalar de dos vectors.

Donats dos vectors ),,( 321 vvvv

i ),,( 321 wwww

, es defineix el seu producte escalar

com:

332211 wvwvwvwv

1.5.2 Proposició. Desigualtat de Schwarz.

Sempre es compleix

vuvu

Demostració. Al final del document.

1.5.2 Definició. Angle entre dos vectors.

Sabem que vuvu i per tant 11

vu

vu

Aquesta propietat ens permet definir l’angle entre dos vectors com:

wv

wvwv

),cos(

1.5.3 Corol·lari. Vectors perpendiculars mitjançant el producte escalar.

Dos vectors ),,( 321 vvvv

i ),,( 321 wwww

són perpendiculars (o ortogonals) (i

escriurem wv

) quan el angle que determinen és de 90º, és a dir, quan formin un

angle recte.

Llavors 0)º90cos(),cos( wv

i per tant, aplicant 1.5.2 tenim

wvwvwvwvwv

wvwv

0),cos(0),cos(0

és a dir, dos vectors (no nuls) seran perpendiculars si i només si el seu producte escalar

és zero:

0 wvwv

1.5.4 Proposició.

Com que )cos(u

és la projecció escalar del vector u

sobre el vector v

, el producte

escalar es pot entendre com el producte d’aquesta projecció per la longitud de v

.

Page 16: Geometria Lineal per al Batxillerat - toomates.net · La història dels vectors. 1.10 Recopilatori d’exercicis. 1.11 Taula resum dels tres productes. 2 Varietats lineals. Rectes

1.5.5 Proposició.

vvv

Demostració. Només cal desenvolupar les definicions:

vvvvvvvvvvvv

2

3

2

2

2

1321321 ),,(),,(

1.5.6 Exercicis.

1. Determineu k per a què els vectors )2,,3( ka

i )6,9,(kb

siguin:

a) perpendiculars,

b) paral·lels.

2. Determina l’angle, en graus, entre els dos vectors )1,5,1( v

i )4,3,2( w

.

3. Determina l’angle, en graus, entre els dos vectors v

= ( 5 , -2 , 1 ) i w

= ( 4 , -4 , -1 ).

4. Donats els vectors )0,1,1(u

i )1,1,( av

, trobeu el valor d’a per al què u

i v

formen un angle de 60º.

5. Sabent que ABCD és un quadrat, )2,0,2(A , )0,1,1(B i ),,0( zyC , trobeu

raonadament les coordenades y i z de C.

1.5.7 Problemes PAU.

1. Donats els vectors )1,2,1(1 aav

, )2,,2(2 aav

i )24,2,(3 aav

de IR3:

a) Calculeu l’angle que formen 1v

i 2v

quan a= 0.

b) Trobeu el valor del paràmetre a perquè els vectors 1v

, 2v

i 3v

siguin perpendiculars

dos a dos. Solució PAU CAT TEC SET 2009 1.4

2. Considereu els punts de l’espai A= (0, –2a–1, 4a–2),B= (1, –3, 4),C= (3, –5, 3).

a) Comproveu que el triangle de vèrtexs A, B i C és rectangle en B per a qualsevol valor

de a.

b) Calculeu els valors de a que fan que aquest triangle sigui isòsceles.

Solució PAU CAT TEC SET 2003 3.4

Page 17: Geometria Lineal per al Batxillerat - toomates.net · La història dels vectors. 1.10 Recopilatori d’exercicis. 1.11 Taula resum dels tres productes. 2 Varietats lineals. Rectes

3. Els punts A(k–3, 2, 4), B(0, k+2, 2) i C(–2, 6, k+1) són tres dels vèrtexs d’un rombe

ABCD (vegeu la figura).

a) Calculeu el valor de k.

b) Demostreu que el rombe és un quadrat. Solució PAU CAT TEC JUNY 2004 1.4

4. Siguin u

i v

els dos vectors del pla :

)1,1(u

)31,31(2

1v

Calculeu l’angle que formen u

i v

.

PAU CAT TEC SET 1998 5.3

Page 18: Geometria Lineal per al Batxillerat - toomates.net · La història dels vectors. 1.10 Recopilatori d’exercicis. 1.11 Taula resum dels tres productes. 2 Varietats lineals. Rectes

1.6 Producte vectorial de dos vectors. Àrea.

1.6.1 Definició. Producte vectorial.

Donats dos vectors ),,( 321 vvvv

i ),,( 321 wwww

, es defineix el producte vectorial

dels dos vectors com el vector:

21

21

31

31

32

32

321

321 ,,ww

vv

ww

vv

ww

vv

www

vvvwv

kji

1.6.2 Proposició. Propietat fonamental del producte vectorial.

El vector vu

és perpendicular als dos vectors u

i v

, i el seu sentit és el sentit de

desplaçament d’un tirabuixó que va de u

cap a v

per l’angle més petit.

La millor manera d’obtenir un vector perpendicular a altres dos donats és calculant el

seu producte vectorial.

1.6.3 Proposició. Altres propietats del producte vectorial.

1. ),sin( vuvuvu

2. El producte vectorial de dos vectors linealment dependents és nul.

3. El mòdul del producte vectorial de 2 vectors és igual a l’àrea del paral·lelogram que

determinen si es representen amb el mateix origen. (I per tant, com que un triangle és la

meitat d’un paral·lelogram, l’àrea és la meitat).

1.6.4 Exercicis.

1. Comprova amb els vectors )1,2,2( i )1,2,1( si el producte vectorial compleix la

propietat commutativa.

2. Donats els vectors )1,2,1(u

i )3,1,2(v

, trobeu un vector perpendicular a tots

dos i de mòdul 3.

3. Siguin els vectors )3,1,1( u

, )1,2,2(v

i )5,2,3( w

. Calculeu:

a) )( wvu

b) )( wvu

c) L’angle que formen u

i v

.

4. Tenim els punts )0,0,1(A , )2,1,2(B i )2,2,1(C . Si sabem que ABCD és un

paral·lelogram, trobeu el punt D, i el seu perímetre i àrea.

Page 19: Geometria Lineal per al Batxillerat - toomates.net · La història dels vectors. 1.10 Recopilatori d’exercicis. 1.11 Taula resum dels tres productes. 2 Varietats lineals. Rectes

1.7 Producte mixt de tres vectors. Volum. Punts coplanaris.

1.7.1 Definició. Producte mixt de tres vectors.

Donats tres vectors ),,( 321 uuuu

),,( 321 vvvv

i ),,( 321 wwww

, el producte mixt

dels tres vectors és el determinant de la matriu que hi generen:

321

321

321

),,det(

www

vvv

uuu

wvu

1.7.2 Proposició. Propietats del producte mixt.

1. El valor absolut del producte mixt de tres vectors és igual al volum del paral·lepípede

que determinen.

|),,det(| wvuVolum

(observa que la fórmula porta valor absolut)

2. El volum del tetràedre desenvolupat per tres vectors és igual a un sisè del valor

absolut del producte mixt dels tres vectors (ja que el paral·lepípede correspon a 6

tetràedres).

|),,det(|6

1wvuVolum

3. Tres vectors són linealment dependents si i només si 0),,det( wvu

1.7.3 Proposició. Relació entre els tres productes:

wvuwvu

),,det(

Demostració.

),,det(

,,),,(),,(

321

321

321

21

21

3

31

31

2

32

32

1

21

21

31

31

32

32

321

321

321321

wvu

www

vvv

uuu

ww

vvu

ww

vvu

ww

vvu

ww

vv

ww

vv

ww

vvuuu

www

vvv

kji

uuuwvu

Page 20: Geometria Lineal per al Batxillerat - toomates.net · La història dels vectors. 1.10 Recopilatori d’exercicis. 1.11 Taula resum dels tres productes. 2 Varietats lineals. Rectes

1.7.4 Definició. Tres vectors coplanaris.

Direm que tres vectors ),,( 321 uuuu

),,( 321 vvvv

i ),,( 321 wwww

són coplanaris

quan el paral·lepípede que generen té volum 0, o dit amb paraules senzilles, quan

generen en l’espai una “capsa plana”.

u

v

i w

són coplanaris 0),,det( wvu

Observació: Més endavant, en 2.1.3, donarem una definició equivalent com a “tres

vectors linealment independents”:

1.7.5 Definició. Quatre punts coplanaris.

Direm que quatre punts A, B, C i D són coplanaris quan els vectors AB , AC i AD

siguin coplanaris, és a dir, quan tinguin determinant zero:

A, B, C i D són coplanaris 0),,det( ADACAB

1.7.6 Exercicis.

1. Donats els vectors u

, v

i w

, calcula wv

, wvu

i ),,det( wvu

i comprova que

es compleix la propietat ),,det( wvuwvu

.

a) )4,1,3( u

, )7,0,2( v

, )1,3,5( w

.

b) )1,0,2(u

, )3,7,0(v

, )4,1,5( w

.

c) )5,1,3( u

, )0,6,5( v

, )3,2,0( w

.

2. Calcula el volum del prisma amb arestes definides per els vectors )1,1,4( , )12,2( i

)1,1,3( .

3. Si ABCDEFGH és un paral·lepípede, i sabent que )1,1,0(A , )1,1,2(B ,

)3,1,1(C i )1,0,2(E , trobeu la resta de vèrtexs i el seu volum.

Page 21: Geometria Lineal per al Batxillerat - toomates.net · La història dels vectors. 1.10 Recopilatori d’exercicis. 1.11 Taula resum dels tres productes. 2 Varietats lineals. Rectes

4. Determina si els següents punts són o no coplanaris:

a) )1,0,1(A , )0,2,3(B , )2,1,1(C , )4,2,3(D

b) )5,1,0(A , )1,6,4(B , )1,2,2(C , )0,1,2( D

5. Determina x de forma que els següents quatre punts siguin coplanaris:

a) )1,0,0(A , )2,1,0(B , )3,1,2(C , )2,1,( xxD

b) )1,0,(xA , )2,1,0(B , )3,2,1(C , )1,2,7(D

1.7.7 Problemes.

1. Els punts )2,3,( aA , )1,,4( bB i ),1,6( aC estan alineats.

Determineu els valors d’a i b.

2. Les coordenades dels vèrtexs consecutius d’un paral·lelogram són )0,0,1(A i

)0,1,0(B . Les coordenades del centre M són )1,0,0(M . Trobeu les

coordenades dels vèrtexs C i D.

3. Determineu el volum del paral·lelepípede format pels vectors )5,2,3( u

,

)1,2,2( v

i )2,3,4( w

.

4. Determineu el volum del tetraedre de vèrtexs A=(3, 2, 1), B=(1, 2, 4), C=(4, 0, 3) i

D=(1, 1, 7).

5. Donats els punts A=(0,5,3), B=(0,6,4), C=(2,4,2) i D=(2,3,1), comproveu que els

quatre punts són coplanaris i que el polígon ABCD és un paral·lelogram.

6. Donats els punts A=(0,0,0), B=(1,0,−1), C=(0,1,−2) i D= (1,2,0), es demana

demostrar que els quatre punts no són coplanaris i calcular el volum del tetraedre que

formen.

7. Considerem els vectors )4,3,2(u

, )1,1,1( v

i )5,,1( w

.

a) Determina els valors de per als què el paral·lelepípede que formen els tres vectors

tingui volum igual a 6 unitats cúbiques.

b) Determina els valors de per als què són tres vectors linealment dependents (és a

dir, coplanaris).

8. Demostra que els quatre punts A=(1,0,-1), B=(2,1,0), C=(0,0,-1) i D=(-1,1,1) no són

coplanaris.

9. Determina els valors d’a i b per als què els punts A=(1,0,0), B=(a,b,0), C=(a,0,b) i

D=(0,a,b) siguin coplanaris.

10. Calcula el volum del tetraedre de vèrtexs A(1,1,1), B=(0, −2,2), C=(−1,0,2) i

D=(2, −1,2).

11. Donats els punts A=(1, 3, −1), B=(a, 2, 0), C=(1, 5, 4) y D=(2, 0, 2),

a) Trobeu el valor de a per al què els quatre punts siguin coplanaris.

b) Trobeu els valors de a per als quals el tetraedre de vèrtexs A,B, C i D tingui volum

igual a 7.

PAU Madrid 2012

Page 22: Geometria Lineal per al Batxillerat - toomates.net · La història dels vectors. 1.10 Recopilatori d’exercicis. 1.11 Taula resum dels tres productes. 2 Varietats lineals. Rectes

1.8 Fora de programa: L’esfera.

1.8.1 Definició. Esfera.

Donat un punt ),,( 321 cccC i un radi 0r , l’esfera de centre C i radi r serà el lloc

geomètric de tots els punts ),,( zyxP tals que la seva distància al centre C és igual a

r, és a dir,

rPCrCPdist ),(

Naturalment, l’esfera en l’espai és l’equivalent a la circumferència en el pla.

1.8.2 Proposició. Equació de l’esfera.

Donat el centre ),,( 321 cccC i el radi 0r , el punt ),,( zyxP pertany a l’esfera si i

només si

2

3

2

2

2

1

321

)()()(

),,(

),(

czcycxCP

czcycxCPCP

rPCrPCrPCrCPdist

Per tant :

22

3

2

2

2

1

2

3

2

2

2

1

)()()(

)()()(

rczcycx

rczcycxrPC

La part de l’esquerra d’aquesta equació la podem desenvolupar en forma de polinomi en

x, y i z per arribar a una equació de la forma

0222 DCzByAxzyx

Un cop més, recorda que NO has de memoritzar cap d’aquestes fórmules. Has

d’estudiar, aprendre i entendre com passar d’una a l’altra sense haver de mirar els

apunts.

Page 23: Geometria Lineal per al Batxillerat - toomates.net · La història dels vectors. 1.10 Recopilatori d’exercicis. 1.11 Taula resum dels tres productes. 2 Varietats lineals. Rectes

1.8.3 Problema resolt.

Donats dos punts de l’espai )1,0,0(A i )3,2,0(B , determineu el lloc geomètric de

tots els punts P tals que BPAP .

En primer lloc anem a fer un estudi geomètric del problema:

Si ens limitem a un pla, els punts P tals que BPAP són els punts de la

semicircumferència de diàmetre AB . Aquest resultat és la proposició 31 del Llibre III

dels Elements d’Euclides. (Vegeu www.toomates.net/biblioteca/AlgebraLineal.pdf

Apartat 9.1.4)

Com que estem en l’espai, hi ha moltes més solucions: Aquesta semicircumferència la

podem girar pel seu eix per obtenir una circumferència de diàmetre AB, formada per

punts que també satisfaran la condició de perpendicularitat:

El lloc geomètric solució del problema serà, doncs, la circumferència de diàmetre AB,

és a dir, aquella que té centre el seu punt mig M i radi 2/AB .

Page 24: Geometria Lineal per al Batxillerat - toomates.net · La història dels vectors. 1.10 Recopilatori d’exercicis. 1.11 Taula resum dels tres productes. 2 Varietats lineals. Rectes

Anem ara a resoldre aquest mateix problema amb coordenades :

)3,2,(

)1,,(

),,(

)3,2,0(

)1,0,0(

zyxBPBP

zyxAPAP

zyxP

B

A

0342

0332

0)3)(1()2(

0)3,2,()1,,(0

222

222

2

zzyyx

zzzyyx

zzyyx

zyxzyxBPAPBPAP

Comprovem que es tracta de la circumferència de diàmetre AB:

Punt mig del segment AB:

)2,1,0()1,1,0()1,0,0(2

1)1,1,0(

2

1)2,2,0( ABAMABABAB

Diàmetre de la circumferència:

228220 222 ABd

Radi de la circumferència :

22

22

2

dr

Equació de la circumferència:

3420

244120

44122

)2()1(2

)2()1(2),(2

)2()1(),(

)2,1,()2,1,0(),,(

222

222

222

222

222

222

zzyyx

zzyyx

zzyyx

zyx

zyxMPd

zyxMPPMd

zyxzyxMPMP

I efectivament arribem a la mateixa equació.

Page 25: Geometria Lineal per al Batxillerat - toomates.net · La història dels vectors. 1.10 Recopilatori d’exercicis. 1.11 Taula resum dels tres productes. 2 Varietats lineals. Rectes

1.9 Notes històriques. La història dels vectors.

La llei del paral·lelogram per a l’addició de vectors és tan intuïtiva que el seu origen és

desconegut. Podria haver aparegut en un treball ara perdut d’Aristòtil (384-322 aC), i es

troba en la Mecànica de Hieró d’Alexandria (segle I dC). Va ser, també, un dels

primers resultats del Principia Mathematica (1687) d’Isaac Newton (1642-1727). En els

Principia, Newton va tractar de manera extensa el que ara es consideren les entitats

vectorials (per exemple, velocitat, força), però mai el concepte de vector. L’estudi i l’ús

de vectors no es va sistematitzar fins als segles XIX i XX.

Els vectors van sorgir a les primeres dues dècades del segle XIX amb les

representacions geomètriques de nombres complexos. Caspar Wessel (1745-1810),

Jean Robert Argand (1768-1822) i Carl Friedrich Gauss (1777-1855) van concebre

nombres complexos com a punts en el pla de dues dimensions, és a dir, com a vectors

de dues dimensions. En 1837, William Rowan Hamilton (1805-1865) va demostrar

que els nombres complexos es podrien considerar parells de nombres (a, b). Aquesta

idea era una part de la campanya de molts matemàtics, incloent-hi el mateix Hamilton,

per a buscar una manera d’ampliar els "nombres de dues dimensions" a tres dimensions.

En 1827, August Ferdinand Möbius va publicar un llibre curt, Càlcul baricèntric, en

el qual va introduir el segment dirigit que va denotar amb les lletres de l’alfabet; ja eren

vectors, encara que no tenien aquest nom. En el seu estudi de centre de gravetat i la

geometria descriptiva, Möbius va desenvolupar el càlcul amb aquests segments dirigits;

els va sumar i va demostrar com es multiplicaven per un nombre.

William Rowan Hamilton (1805-1865)

Finalment, el mateix Hamilton va introduir en 1843 en concepte de vector, precisament

com un segment orientat de l’espai.

El desenvolupament de l’àlgebra de vectors i de l’anàlisi de vectors tal com el coneixem

avui va ser fet per primera vegada per J. Willard Gibbs (1839-1903) en les classes per

als seus estudiants en la Universitat de Yale. Gibbs va intuir que els vectors

proporcionarien una eina més eficient per al seu treball en la física. Així, doncs,

començant el 1881, Gibbs va imprimir en privat notes sobre anàlisi dels vectors per als

seus estudiants, que van ser distribuïts extensament entre els erudits dels Estats Units i

d’Europa.

Page 26: Geometria Lineal per al Batxillerat - toomates.net · La història dels vectors. 1.10 Recopilatori d’exercicis. 1.11 Taula resum dels tres productes. 2 Varietats lineals. Rectes

1.10 Recopilatori d’exercicis.

1. Donats els punts )0,1,1( A , )4,,1( bB , ),10,4( cC , )3,2,1(D i

),5,0( eE , trobeu les coordenades que falten en B, C i D sabent que A, B i C estan

alineats i A, B, D i E són coplanaris.

2. Determineu els punts que divideixen el segment AB en 5 parts iguals, suposant que

)6,3,2( A i )6,6,13(B

3. Donats els quatre punts )3,2,1(A , )3,1,( bB , )4,,2( cC i ),2,5( dD

a) Trobeu b, c i d sabent que ABCD és un paral·lelogram.

b) Trobeu el centre del paral·lelogram.

c) Trobeu els punts que divideixen el segment CD en 3 parts iguals.

4. Els punts )2,3,(aA , )1,,4( bB i ),1,6( aC estan alineats. Trobeu els valors

d’A i B.

5. Trobeu k per a què els 4 punts següents siguin coplanaris:

)1,3,1(P , )1,0,2(Q , )1,0,0(R i )2,1,(kS

6. Trobeu k per a què els 4 punts següents siguin coplanaris:

)0,1,3(P , )1,2,2( Q , )1,3,0(R i ),2,1( kS

7. Si sabem que ABCD és un quadrat, )2,0,2(A , )0,1,1(B , ),,0( zyC , trobeu

les coordenades que falten de C.

8. Donats els vectors )1,2,1(u i )3,1,2(v

, trobeu un vector perpendicular a u

i v

de mòdul 3.

9. Donats els vectors )3,1,1( a

, )3,0,2(b

i ),2,7( zc

, trobeu:

a) els mòduls de a

i b

.

b) L’angle que formen a

i b

.

c) La coordenada que falta en c

, sabent que ac

.

10. Donats els vectors )3,1,2( a

, )1,3,2(b

i )5,1,0(c

, trobeu:

a) L’àrea el paral·lelogram determinat per a

i b

.

b) El volum del paral·lepípede determinat per a

, b

i c

.

11. Trobeu l’àrea del triangle ABC , on )3,2,1(A , )4,0,2(B i )0,5,10(C .

12. a) Si )1,0,3( A , )5,4,6( B i ),3,5( zC , trobeu z si sabem que el triangle

ABC és rectangle, on º90A .

b) Trobeu la seva àrea.

13. Donats )12,4,3( u

i )6,2,5( v

, calcula:

a) vu

Page 27: Geometria Lineal per al Batxillerat - toomates.net · La història dels vectors. 1.10 Recopilatori d’exercicis. 1.11 Taula resum dels tres productes. 2 Varietats lineals. Rectes

b) u

i v

.

c) L’angle entre u

i v

.

d) Quant ha de valer y perquè )2,,7( yw

sigui perpendicular a u

?

14. Troba els vectors de 3IR que són perpendiculars a )1,0,1(v

i formen un angle de

60º amb

2

1,

2

2,

2

1v

.

15. Troba el valor de m perquè els vectors )1,5,3( u

, )1,1,2( v

i ),4,1( mw

determinin un paral·lelepípede de volum 11.

16. Donats els punts )1,1,2(A , )1,0,0(B i )0,0,(C , troba el valor de tal que

l’àrea del triangle ABC sigui 2.

17. Donats els vectors )1,,( bau

, )1,4,3(v

i ),2,1( cw

, determina els valor dels

paràmetres IRcba ,, de manera que els vectors v

i w

siguin perpendiculars i que, a

més, vwu

. Quin angle formen u

i v

en aquest cas?

18. Demostreu que els tres punts )1,1,1( , )1,0,2( i ( )1,2,0 són sobre una mateixa recta.

19. Considerem els vectors )1,1,1(u

, ),2,2( av

i )0,0,2(w

.

a) Determina els valors de a per als què els vectors u

, v

i w

son coplanaris.

b) Determina els valors de a per als què els vectors vu

i vu

son perpendiculars.

20. Siguin els punts )1,2,1(A , )1,3,2(B , )3,5,0(C i )3,4,1(D ,

a) Demostreu que els quatre punts són coplanaris.

b) Demostreu que ABCD és un rectangle.

c) Calculeu l’àrea d’aquest rectangle.

21. Calculeu un vector de mòdul 1 que sigui perpendicular als vectors )2,0,1( i )0,1,2( .

22. Donats els vectors )2,1,( aaau

, ),1,( aav

i )1,,1( aw

, trobeu els valors de a

per als què els tres vectors siguin coplanaris.

23. Se consideren els punts )0,,1( aA , )2,1,1( aB i ),1,1( aC .

a) Comproveu que no estan alineats, sigui quin sigui el valor del paràmetre a.

b) Trobeu l’àrea del triangle que determinen aquests punts.

24. Considerem el tetraedre de vèrtexs A=(1, 0, 0), B=(1, 1, 1), C=(-2, 1, 0) y D=(0, 1,

3).

a) Trobeu l’àrea del triangle ABC.

b) Trobeu el volum del tetraedre ABCD.

Page 28: Geometria Lineal per al Batxillerat - toomates.net · La història dels vectors. 1.10 Recopilatori d’exercicis. 1.11 Taula resum dels tres productes. 2 Varietats lineals. Rectes

25. Els punts )1,1,1(A , )2,2,2(B i )3,3,1(C són tres vèrtexs consecutius d’un

paral·lelogram.

a) Trobeu les coordenades del quart vèrtex D.

b) Calculeu el seu àrea.

c) Comproveu que es tracta d’un rombe i no d’un rectangle.

26. Donats els punts )2,0,2( A , )1,4,3( B , )3,4,5( C y )4,1,0(D ,

a) Calculeu l’àrea del triangle de vèrtexs A, B i C.

b) Calculeu el volum del tetraedre (piràmide) ABCD.

27. Donats els punts )1,3,1( A , )0,2,(aB , )4,5,1(C i )2,0,2(D , es demana:

a) Trobeu el valor de a per a què els quatre punts siguin coplanaris.

b) Trobeu els valors de a per a què el tetraedre (piràmide) amb vèrtexs A,B,C,D tingui

un volum igual a 7.

28. Donats els punts )0,3,1( A , )2,1,3( B , )3,2,7(C , )5,2,5( D .

a) Demostreu que els punts A, B, C i D són coplanaris.

b) Demostreu que el polígon ABCD és un paral·lelogram i calculeu la seva àrea.

29. Donats els punts ),2,( A , )0,,2( B i )2,0,( C , existeix algun valor

de per al què els punts A, B i C estiguin alineats? Comproveu que els punts sempre

formen un triangle isòsceles.

Page 29: Geometria Lineal per al Batxillerat - toomates.net · La història dels vectors. 1.10 Recopilatori d’exercicis. 1.11 Taula resum dels tres productes. 2 Varietats lineals. Rectes

1.11 Taula resum dels tres productes.

Fórmula

Interpretació

geomètrica Equació igual a zero

Pro

du

cte

esc

alar

332211 wvwvwvwv

Angle:

wv

wvwv

),cos(

Perpendicularitat:

wvwv

0

Pro

du

cte

vec

tori

al

321

321

kji

www

vvvwv

wv

és perpendicular a

v

i w

:

L’àrea del paral·lelogram

és igual a wv

Vectors proporcionals:

wkvwv

0

(no s’utilitza habitualment)

Pro

du

cte

mix

t

321

321

321

,,

www

vvv

uuu

wvu

El volum del

paral·lelepípede és igual a

wvu

,,

Vectors coplanaris:

coplanariswvuwvu

,,0,,

(és a dir, linealment dependents)

Page 30: Geometria Lineal per al Batxillerat - toomates.net · La història dels vectors. 1.10 Recopilatori d’exercicis. 1.11 Taula resum dels tres productes. 2 Varietats lineals. Rectes

2 Varietats lineals. Rectes i plans.

2.1 Combinacions lineals de vectors.

2.1.1 Combinacions lineals d’un vector. Rectes vectorials.

Direm que t

és combinació lineal del vector u

quan existeix un nombre real a tal que

uat

El conjunt de totes les combinacions lineals d’un vector 0u

forma el que s’anomena

una recta vectorial:

IRauau ,

En particular, el vector zero sempre és combinació lineal de qualsevol vector, perquè

u

0)0,0,0(0

Exercicis.

1. Donat )4,3,2( u

, calcula u

2 i u

3 .

2. Donat )3,2,4(u

, escriu )9,6,12(t

com a combinació lineal de u

.

2.1.2 Combinacions lineals de dos vectors. Plans vectorials.

Direm que t

és combinació lineal dels vectors u

i v

quan existeixen nombres reals a i

b tals que

vbuat

El conjunt de totes les combinacions lineals dels vectors u

i v

formen el que

s’anomena un pla vectorial:

IRbavbuavu ,,,

Però atenció! No sempre! Només si els vectors u

i v

no són proporcionals, és a dir,

quan vku

.

Page 31: Geometria Lineal per al Batxillerat - toomates.net · La història dels vectors. 1.10 Recopilatori d’exercicis. 1.11 Taula resum dels tres productes. 2 Varietats lineals. Rectes

Direm que els vectors u

i v

són linealment independents quan formin un pla

vectorial, és a dir, quan no siguin proporcionals. Si dos vectors són proporcionals

formen una recta vectorial.

En particular, el vector zero sempre és combinació lineal de qualsevol parella de

vectors, perquè

vu

00)0,0,0(0

Escriu el vector )13,11,12( com a combinació lineal dels vectors )3,1,2( u

i

)2,4,3( v

.

ba

ba

ba

bababa

bbbaaa

ba

2313

411

3212

)23,41,32()13,11,12(

)2,4,3()3,1,2()13,11,12(

)2,4,3()3,1,2()13,11,12(

Arribem a un sistema d’equacions sobredeterminat (3 equacions i 2 incògnites), que

hem de (intentar) resoldre:

3,2225123)114(212114

2313

411

3212

abbbbba

ba

ba

ba

Comprovem que la solució 2,3 ba satisfà les tres equacions, per tant el sistema és

compatible determinat i la resposta és:

)2,4,3(2)3,1,2(3)13,11,12(

Page 32: Geometria Lineal per al Batxillerat - toomates.net · La història dels vectors. 1.10 Recopilatori d’exercicis. 1.11 Taula resum dels tres productes. 2 Varietats lineals. Rectes

Escriu el vector )2,4,3( com a combinació lineal dels vectors )1,4,3( u

i

)1,5,2( v

.

ba

ba

ba

bababa

bbbaaa

ba

2

544

233

),54,23()2,4,3(

)1,5,2(),4,3()2,4,3(

)1,5,2()1,4,3()2,4,3(

Arribem a un sistema d’equacions sobredeterminat (3 equacions i 2 incògnites), que

hem de (intentar) resoldre:

1,3632)2(332

2

544

233

abbbbba

ba

ba

ba

Però la solució 1,3 ab no satisfà la segona equació:

419154)3(5)1(4

Per tant el sistema és incompatible, és a dir, no es pot resoldre, i per tant el vector

)2,4,3( no es pot escriure com a combinació lineal de )1,4,3( u

i

)1,5,2( v

. El problema no té solució.

2.1.3 Combinacions lineals de tres vectors. Espai vectorial.

Direm que el vector t

és combinació lineal dels vectors u

, v

i w

quan existeixen

nombres reals a , b i c tals que

wcvbuat

El conjunt de totes les combinacions lineals dels vectors u

, v

i w

formen tot l’espai

de vectors:

3,,,,, IRIRcbawcvbuawvu

Però només quan el seu determinant és diferent de zero: 0,,det wvu

i en aquest cas direm que els vectors u

, v

i w

són linealment independents, o que

formen una base de 3IR .

2.1.4 Observació. L’espai IR3 té tres dimensions.

Aquest procés no pot créixer més, s’acaba aquí: És impossible trobar quatre vectors

linealment independents en IR3. Per això diem que l’espai IR

3 té tres dimensions.

Page 33: Geometria Lineal per al Batxillerat - toomates.net · La història dels vectors. 1.10 Recopilatori d’exercicis. 1.11 Taula resum dels tres productes. 2 Varietats lineals. Rectes

Anomenem base de IR3 a qualsevol conjunt de tres vectors linealment independents, i

donada una base wvu

,, , qualsevol vector t

de IR3 sempre es pot escriure com a

combinació lineal d’aquests tres vectors, i a més a més de forma única:

wcvbuat

Donats els vectors )1,3,2( u

, )3,1,4( v

i )2,3,1(w

, demostra que formen una

base de 3IR i escriu el vector )5,10,1(t

com a combinació lineal de tots tres.

Formen una base perquè el seu determinant no és zero:

022

231

313

142

Volem trobar els nombres a, b i c de forma que

)2,3,1()3,1,4()1,3,2()5,10,1( cba

És a dir:

cba

cba

cba

cbacbacba

cccbbbaaa

235

3310

421

)23,33,42()5,10,1(

)2,3,()3,,4(),3,2()5,10,1(

Un sistema lineal 3x3 que sabem que és SCD perquè el seu determinant és diferent de

zero. El resolem per Gauss:

3

2

1

111100

25980

5231

9520

25980

5231

1142

10313

5231

5231

10313

1142

235

3310

421

a

b

c

cba

cba

cba

Per tant )2,3,1(1)3,1,4(2)1,3,2)(3()5,10,1(

Page 34: Geometria Lineal per al Batxillerat - toomates.net · La història dels vectors. 1.10 Recopilatori d’exercicis. 1.11 Taula resum dels tres productes. 2 Varietats lineals. Rectes

2.1.5 Combinació lineal de vectors.

Una combinació lineal dels vectors nvv

,,1 és qualsevol vector de la forma

nnvvv

11 per a certs escalars IRn ,,1

En particular, el vector 0

sempre és combinació lineal de qualsevol conjunt de vectors:

nvv

000 1

Direm que un conjunt de vectors nvv

,,1 són linealment dependents quan podem

trobar un conjunt d’escalars n ,,1 , no tots nuls, tal que 011

nnvv .

Equivalentment, direm que són linealment dependents quan algun d’aquests vectors es

pot escriure com a combinació lineal dels altres. En cas contrari direm que són

linealment independents.

Nombre de vectors linealment

independents Condició

Objecte geomètric que es

genera

1 vector

0v

Recta vectorial

2 vectors vku

Pla vectorial

3 vectors

0),,det( wvu

Tot l’espai de vectors

IR3

Page 35: Geometria Lineal per al Batxillerat - toomates.net · La història dels vectors. 1.10 Recopilatori d’exercicis. 1.11 Taula resum dels tres productes. 2 Varietats lineals. Rectes

2.1.8 Exercicis.

1. Donat )2,1,5( u

, determina u

2 , u

3 i u

5.0

2. Donat )5,7,2( u

, determina u

4 , u

i u

31

3. Comprova si els vectors (-2,1,0), (-1,3,1) i (3,3,0) són linealment independents.

4. Determina k per a que els vectors (-3,k,2), (4,1,0) i (-1,2,-1) siguin coplanaris.

5. Escriviu el vector )4,2,1(v

com a combinació lineal dels vectors )1,0,1(a

,

)0,1,1(b

i )1,1,0(c

.

2.1.9 Problemes PAU.

1. Sigui V= {(–1, 1, 1), (–2, –1, 0), (1, 2,a)} un conjunt de vectors de IR3.

a) Trobeu el valor o els valors de a perquè V sigui linealment dependent.

b) Quan a= 4, expresseu el vector 14) 9, (3, v

com a combinació lineal dels vectors de

V. Solució PAU CAT TEC SET 2013 1.1

2. Siguin 2) 3, (-1, 1 u

, 4) (2,-1, u2

i 2) 4a 1,-a1, (a u3

tres vectors de l’espai

vectorial IR3.

a) Trobeu el valor del paràmetre a per al qual el vector 3u

és combinació lineal dels

vectors 1u

i 2u

b) Comproveu que per a a= 0 el conjunt 321 ,, uuu

és linealment independent.

Solució PAU CAT TEC JUNY 2010 4.6

3. Considereu els vectors de IR3:

4) 3, (-1, v1

, 3)- 1,- (2, 2 v

i 3) k 1, 2k (1, 3 v

.

a) Trobeu l’únic valor de k per al qual aquests vectors no són una base de IR3.

b) Per a un valor de k diferent del que heu trobat en l’apartat a), quins són els

components del vector 321 vvvw

en la base 321 v,v,v

?

Solució PAU CAT TEC SET 2005 3.2

4. Donats els vectors 2) (1, u

i 1) (-3, v

:

a) Comproveu que u

i v

formen una base de l’espai vectorial dels vectors del pla;

b) Trobeu els components del vector = (–1, 5) en la base vu

,

Solució PAU CAT TEC SET 2004 5.4

Page 36: Geometria Lineal per al Batxillerat - toomates.net · La història dels vectors. 1.10 Recopilatori d’exercicis. 1.11 Taula resum dels tres productes. 2 Varietats lineals. Rectes

5. Donats els vectors 4) (1,-1, u

, 3) 1, (2, v

i 0) 0, (1, w

,

a) Determineu si són vectors linealment dependents o independents.

b) Calculeu la relació que hi ha d'haver entre els valors de a i b per tal que el vector (a,

1, b) sigui combinació lineal de u

i v

. Solució PAU CAT TEC SET 2000 2.3

6. Calculeu el valor de k per tal que els vectors ( 1 , 2 , -1, 1 ) , ( 1 , -1 , 1 , 1 ) i

( 2 , 5k , -3k, 2 ) siguin linealment dependents

PAU COU

7. Donats els vectors )2,1,( aaau

, ),1,( aav

i )1,,1( aw

, es demana:

a) Determineu els valors de a per als què els vectors u

, v

, w

siguin linealment

dependents.

b) Estudieu si el vector )0,3,3(c

depèn linealment dels vectors u

, v

, w

per al cas

2a . En cas afirmatiu, determineu la combinació lineal associada.

PAU MADRID TEC 2000 Opció A

Page 37: Geometria Lineal per al Batxillerat - toomates.net · La història dels vectors. 1.10 Recopilatori d’exercicis. 1.11 Taula resum dels tres productes. 2 Varietats lineals. Rectes

2.2 Rectes. Equacions de la recta.

2.2.1 Definició. Equacions de la recta.

Una recta r en l’espai queda determinada per un punt ),,( 321 pppP (“punt base”) i

un vector ),,( 321 vvvv

(“vector director”)

Conjunt de punts: IRkvkPQr ,:

Equació vectorial: ),,(),,(),,( 321321 vvvkpppzyx

Equació paramètrica:

33

22

11

vkpz

vkpy

vkpx

Equació contínua: 3

3

2

2

1

1

v

pz

v

py

v

px

Equació cartesiana:

2222

1111

dzcybxa

dzcybxa

on ),,( 111 cba i ),,( 222 cba no són proporcionals.

Més endavant veurem que l’equació cartesiana d’una recta

és, en realitat, la interpretació de la recta com a intersecció

de dos plans.

Page 38: Geometria Lineal per al Batxillerat - toomates.net · La història dels vectors. 1.10 Recopilatori d’exercicis. 1.11 Taula resum dels tres productes. 2 Varietats lineals. Rectes

2.2.2 Proposició. Recta que passa per dos punts.

Donats dos punts P i Q, existeix una única recta que passa per tots dos.

Agafem com a punt base qualsevol dels dos i com a vector director el vector que

determinen: PQv

.

2.2.3 Exercicis.

1. Determina l’equació contínua de la recta que passa per

P = ( -11 , -10 , -6 ) i Q = ( -8 , 17 , -17 )

2. Determina l’equació contínua de la recta que passa per

P =( 17 , 20 , -13 ) i Q =( -6 , -1 , -2 )

3. Determina el punt P d’intersecció entre les rectes r i s.

4

6

3

112:

zyxr

2

3

4

4

1:

z

yxs

4. Determina el punt P d’intersecció entre les rectes r i s.

2

33

3

6:

zy

xr 10

5

103:

z

yxs

Page 39: Geometria Lineal per al Batxillerat - toomates.net · La història dels vectors. 1.10 Recopilatori d’exercicis. 1.11 Taula resum dels tres productes. 2 Varietats lineals. Rectes

2.2.4 Problemes PAU.

1. Considereu els punts de l’espai )3,1,1( aP , )1,2,0( aaQ i

)66,1,2( aR

a) Trobeu el valor de a per al qual els tres punts estan alineats.

b) Quan els tres punts estan alineats, quina és l’equació de la recta que els conté?

Solució PAU CAT TEC JUNY 2009 3.6

2. Trobeu l’equació de la recta continguda en el pla 0 2-6z2yx: , que talla els

eixos OY i OZ. Solució PAU CAT TEC JUNY 2007 1.4

3. Considereu els punts de l’espai P=(–1, a–1, 3), Q=(0, a–2, 1–a) i R=(2, –1, 6 – 6a).

a) Trobeu el valor de a per al qual els tres punts estan alineats.

b) Quan els tres punts estan alineats, quina és l’equació de la recta que els conté?

Solució PAU CAT TEC JUNY 2007 1.2

4. Considereu els punts de l’espai A(1, 1, 2),B(0, 1, 1) i C(k, 1, 5).

a) Trobeu l’equació de la recta que passa per A i B.

b) Per a quins valors de k els punts A, B i C formen un triangle?

Solució PAU CAT TEC JUNY 2004 4.1

Page 40: Geometria Lineal per al Batxillerat - toomates.net · La història dels vectors. 1.10 Recopilatori d’exercicis. 1.11 Taula resum dels tres productes. 2 Varietats lineals. Rectes

2.3 Plans. Equacions del pla.

2.3.1 Definició. Equacions del pla.

Un pla en l’espai queda determinat per un punt P (“punt base”) i dos vectors

directors wv

, linealment independents (“vectors directors”).

Conjunt de punts: IRbawbvaPQ ,|:

Equació vectorial: ),,(),,(),,(),,( 321321321 wwwbvvvapppzyx

Equació paramètrica:

333

222

111

wbvapz

wbvapy

wbvapx

Equació cartesiana: DCzByAx

El vector ),,( CBAn

s’anomena vector normal del pla i

té unes propietats geomètriques molt interessants. (Vegeu

Apartat 2.3.7)

2.3.2 Proposició. Primer mètode per a obtenir l’equació cartesiana d’un pla.

Un punt ),,( zyxQ pertany al pla si i només si PQ , v

i w

son linealment dependents

i per tant, el determinant d’aquests tres

vectors ha de ser zero:

0

321

321

321

www

vvv

pzpypx

Page 41: Geometria Lineal per al Batxillerat - toomates.net · La història dels vectors. 1.10 Recopilatori d’exercicis. 1.11 Taula resum dels tres productes. 2 Varietats lineals. Rectes

2.3.3 Proposició. Pla que passa per tres punts.

Donats tres punts no alineats de l’espai

),,(),,,(),,,( 321321321 cccCbbbBaaaA

Existeix un únic pla que passa per aquests tres punts.

Agafem com a punt base el punt A, com a vectors directors ABv

, ACw

, i

apliquem 2.3.2.

Determina el pla que passa pels següents punts:

)1,5,1(,)4,1,0(,)3,4,3( CBA

414

153

343

314531

344130

343

31)4(5)3(1

34)4(1)3(0

3)4()3(

0

zyxzyxzyx

=

76231619

481236020931646020

)4(12)3()3(20)3(3)4(4)3(20

)4(3)4()3(1)1()3(54)1(3)3(41)4()4(5)3(

zyx

yxzzyx

yxzzyx

yxzzyx

Per tant, l’equació és yzxzyx 16762319076231619

Efectivament, aquesta equació es satisfà amb els punts A, B i C de l’enunciat:

)4(1676)3(23)3(19

)1(1676)4(23)0(19

)5(1676)1(23)1(19

Page 42: Geometria Lineal per al Batxillerat - toomates.net · La història dels vectors. 1.10 Recopilatori d’exercicis. 1.11 Taula resum dels tres productes. 2 Varietats lineals. Rectes

Exercicis.

1. Determina l’equació general del pla que passa pels següents tres punts:

a) P=(1, -2, 0), Q=(3, 1, 4), R=(0, -1, 2)

b) P=(1, 1, 1), Q=(1, 2, 0), R=(-1, 2, 1)

c) P=(2, 1, -1), Q=(0, -2, 0), R=(1, -1, 2)

d) P=(3, 2, 0), Q=(1, 3, -1), R=(0, -2, 3)

2. Determineu raonadament l’equació del pla que passa pels punts )0,0,0(A ,

)0,3,6( B i )1,0,3(C

3. Determina el pla que passa pel punt )7,1,2( P i té com a vectors directors

)0,1,2( v

i )2,0,1(w

.

4. Determina el pla que passa pel punt )3,0,5( P i té com a vectors directors

)4,2,1(v

i )0,1,3(w

.

Problemes PAU.

1. Donats els punts P=(1, 0, 0), Q=(0, 2, 0), R=(0, 0, 3) i S=(1, 2, 3),

a) Calculeu l’equació cartesiana (és a dir, de la forma Ax+By+Cz+D=0) del pla que

conté els punts P, Q i R.

b) Comproveu si els quatre punts són coplanaris (és a dir, si els quatre estan continguts

en un mateix pla). Solució PAU CAT TEC JUNY 2012 3.6

2. Considereu els punts de l’espai A(0, 0, 1), B(1, 1, 2) i C(0, –1, –1).

a) Trobeu l’equació del pla ABC.

b) Si D és el punt de coordenades (k, 0, 0), quant ha de valer k per tal que els

quatre punts A, B, C i D siguin coplanaris? Solució PAU CAT TEC JUNY 2004 1.2

3. Un quadrat de l'espai té tres dels seus vèrtexs consecutius situats en els punts de

coordenades enteres P = (3, –2, 4), Q = (a, –1, a + 1) i R = (2, –3, 0).

a) Tenint en compte que els vectors QP i QR han de ser perpendiculars, calculeu el

valor del nombre enter a.

b) Calculeu l'equació del pla que conté aquest quadrat.

c) Calculeu el quart vèrtex d'aquest quadrat.

d) Calculeu l'àrea d'aquest quadrat. Solució PAU CAT TEC JUNY 2000 3.6

Page 43: Geometria Lineal per al Batxillerat - toomates.net · La història dels vectors. 1.10 Recopilatori d’exercicis. 1.11 Taula resum dels tres productes. 2 Varietats lineals. Rectes

2.3.4 Proposició. Pla que conté una recta i passa per un punt.

Donada una recta vPr

: i un punt rQ , existeix un únic pla que conté la recta r i

passa per Q.

Agafem com a punt base el punt Q i com a vectors directors el vector director de la recta

v

i el vector PQw

.

Problemes PAU.

1. Determineu l'equació del pla que conté a la recta 12

1 zy

x i passa per

l'origen de coordenades. PAU CAT TEC JUNY 2003 5.2

2. Considereu la recta r de l'espai d'equacions 2

1

2

3

zy

x

Trobeu l'equació cartesiana del pla que conté r i que passa pel punt P = (1, 1, 1)

(equació cartesiana vol dir la de la forma ax+by+cz=d).

PAU CAT TEC SET 1999 2.2

Page 44: Geometria Lineal per al Batxillerat - toomates.net · La història dels vectors. 1.10 Recopilatori d’exercicis. 1.11 Taula resum dels tres productes. 2 Varietats lineals. Rectes

2.3.5 Proposició. Pla que conté dues rectes paral·leles.

Donades dues rectes paral·leles vPr

: i wQs

: , existeix un únic pla que conté a

totes dues.

Agafem com a punt base el punt P i com a vectors directors el vector director v

d’una

recta r i el vector PQw

.

Problemes PAU.

1. Donades les rectes 4

2

2

1

3

5::1

zyxr i

0112

0522::2

zyx

zyxr

a) Comproveu que són paral·leles.

b) Trobeu l’equació general (és a dir, de la forma Ax+By+Cz+D= 0) del pla que les

conté. Solució PAU CAT TEC JUNY 2010 1.4

2. Considereu les rectes 22

1:

zy

xr i

112

52:

yx

zxs

Comproveu que aquestes dues rectes són paral·leles i calculeu l'equació del pla que les

conté.

Sol: 12 zx PAU CAT TEC JUNY 1999 6.4

Page 45: Geometria Lineal per al Batxillerat - toomates.net · La història dels vectors. 1.10 Recopilatori d’exercicis. 1.11 Taula resum dels tres productes. 2 Varietats lineals. Rectes

2.3.6 Proposició. Pla que conté dues rectes que es tallen.

Donades dues rectes vPr

: i wQs

: que es tallen, existeix un únic pla que conté

a totes dues.

Agafem com a punt base qualsevol punt d’una de les rectes i com a vectors directors els

vectors directors de les rectes.

Important: El punt base del pla no necessàriament ha de ser el punt de tall de les rectes.

No l’has de calcular, una feina menys!.

Les rectes

1

02:

z

yxr i

05

2:

zy

xs són secants. Determina el pla que les

conté.

Un vector director de la recta r és )0,1,1()0,1,1(

100

011

kji

v

Un vector director de s és )1,1,0()2,2,0(

110

002

kji

w

Clarament són vectors no proporcionals )1,1,0()0,1,1( k , per tant són rectes no

paral·leles.

Com a punt base del pla podem agafar qualsevol punt de les rectes, per exemple,

prenent la recta r, i fent 0x obtenim el punt

)1,2,0(2020 Pyy

Per tant, el pla que volem té com a punt base )1,2,0( P i vectors directors )0,1,1(v

i )1,1,0(w

33

10)1(

112

010

0

zyxyzx

z

y

x

Page 46: Geometria Lineal per al Batxillerat - toomates.net · La història dels vectors. 1.10 Recopilatori d’exercicis. 1.11 Taula resum dels tres productes. 2 Varietats lineals. Rectes

Problemes PAU.

1. Siguin r i s dues rectes d’equacions

)1,1,2()4,3,4(),,(: tzyxr , 31

21:

azyxs

a) Trobeu el valor del paràmetre a perquè aquestes rectes es tallin.

b) En el cas en què es tallen, trobeu l’equació general (és a dir, de la forma

Ax+By+Cz+D=0) del pla que les conté. Solució PAU CAT TEC JUNY 2010 5.5

Page 47: Geometria Lineal per al Batxillerat - toomates.net · La història dels vectors. 1.10 Recopilatori d’exercicis. 1.11 Taula resum dels tres productes. 2 Varietats lineals. Rectes

2.3.7 Definició. Vector normal del pla.

De l’equació cartesiana DCzByAx definim el vector normal al pla com

),,( CBAn

Aquest vector és molt important geomètricament perquè és perpendicular al pla.

2.3.8 Proposició. Segon mètode per a obtenir l’equació cartesiana d’un pla.

Obtenim el vector normal del pla amb el producte vectorial dels vectors directors:

wvn

El valor D s’obté substituint el punt base P a l’equació.

Page 48: Geometria Lineal per al Batxillerat - toomates.net · La història dels vectors. 1.10 Recopilatori d’exercicis. 1.11 Taula resum dels tres productes. 2 Varietats lineals. Rectes

2.4 Punt d’intersecció entre recta i pla.

En l’apartat 6.1 veurem les diferents posicions relatives entre una recta i un pla. Aquí,

de moment, suposarem que la recta i el pla es tallen en un punt i estudiarem com

determinar aquest punt de tall.

Determina el punt de tall entre el pla 52: zyx i la recta

102

0:

zyx

zyxr .

Un mètode és resoldre el sistema 3x3

102

0

52

zyx

zyx

zyx

per Gauss.

Si no volem fer-lo, un mètode alternatiu és passar la recta a forma paramètrica:

Vector director de la recta: )3,1,2(

112

111

kji

v

Punt base de la recta (fent 0x ) )5,5,0(5,510

0

Pzy

zy

zy

Les equacions paramètriques de la recta r són, doncs:

35

5

2

z

y

x

Substituïm el punt genèric de la recta a l’equació del pla, obtenint així una equació de

primer grau en :

25)35(2)5(252: zyx

I per tant el punt de tall és )1,3,4(

235

25

22

z

y

x

Page 49: Geometria Lineal per al Batxillerat - toomates.net · La història dels vectors. 1.10 Recopilatori d’exercicis. 1.11 Taula resum dels tres productes. 2 Varietats lineals. Rectes

2.4.1 Exercicis.

1. Determina el punt d’intersecció entre el pla 032: zyx i la recta

tz

ty

tx

r 34

21

:

2.4.2 Problemes PAU.

1. Donats el pla π: 3x –2y +5z = 6 i la recta 3

2

1

1

2

1:

zyxr , busqueu el punt de

tall, si existeix. Solució PAU CAT TEC JUNY 2008 5.4

2. Donades les rectes

12

1

1

2:

zyxr i

3

5

2

7

1

1:

zyxs

i el punt P= (1, 1, –1), volem trobar l’equació de la recta que passa per P i que talla r i s.

Per aconseguir-ho:

a) Trobeu l’equació general o cartesiana (és a dir, l’equació de la forma

Ax+By+Cz+D=0) del pla π que conté la recta r i el punt P.

b) Trobeu el punt M calculant el punt d’intersecció del pla π amb la recta s.

c) Trobeu l’equació de la recta que passa pels punts P i M.

d) Comproveu que la recta trobada en l’apartat anterior és la que busquem.

PAU CAT TEC JUNY 2008 2.6 (Problema)

3. Una recta r és paral·lela a la recta 111: zyxs , talla en un punt A la recta

123

1:

z

yxt , i en un punt B la recta

32

1

2

2:

zyxl

.

a) Trobeu l’equació del pla determinat per les rectes r i t.

b) Trobeu el punt B calculant el punt d’intersecció del pla anterior amb la recta l.

c) Trobeu l’equació de la recta r.

d) Trobeu el punt A. Solució CAT TEC SET 2007 3.5

4. Considerem el cub de vèrtexs A, B, C, D, E, F, G, H que té l'aresta de longitud 4 dm.

a) Determineu l'equació del pla inclinat EHBC si prenem com a origen de coordenades

el vèrtex D i com a eixos de coordenades DA, DC i DH en aquest ordre, tenint en

compte que el sentit positiu de cada un d'ells és el que sortint de l'origen D va cap a A,

C i H, respectivament.

b) Calculeu les equacions de les diagonals CE i AG i utilitzeu-les per calcular les

coordenades del seu punt d'intersecció. Solució PAU CAT TEC SET 2002 1.6 (Problema)

Page 50: Geometria Lineal per al Batxillerat - toomates.net · La història dels vectors. 1.10 Recopilatori d’exercicis. 1.11 Taula resum dels tres productes. 2 Varietats lineals. Rectes

2.5 Recta intersecció de dos plans.

Dos plans no paral·lels es tallen en una recta.

Les equacions cartesianes dels dos plans

donen directament l’equació cartesiana

de la recta, ja que l’equació cartesiana

d’una recta no és res més que la

interpretació de la recta com a

intersecció de dos plans.

Propietat important: El vector director de la recta intersecció és perpendicular comú

als dos vectors normals dels plans, per tant:

Per determinar el vector director de la

recta intersecció podem calcular el

producte vectorial dels dos vectors

normals:

321

321

mmm

nnn

kji

mnv

Page 51: Geometria Lineal per al Batxillerat - toomates.net · La història dels vectors. 1.10 Recopilatori d’exercicis. 1.11 Taula resum dels tres productes. 2 Varietats lineals. Rectes

2.6 Rectes coplanàries.

Pensem en carreteres. Si ens mantenim “a peu de terra”, només tenim dues opcions: O

fem carreteres paral·leles, o hem de fer encreuaments. Dues rectes diferents en el pla, o

bé són paral·leles o bé es tallen en un punt.

Però ara estem en l’espai, hem passat de dos dimensions a tres, i per tant tenim una

dimensió més per on ens hi podem escapar. Ara podem fer rectes que es creuen, és a

dir, que no estan al mateix pla:

Direm que dues rectes són coplanàries quan estiguin contingudes en un mateix pla.

Direm que dues rectes es creuen en cas contrari, és a dir, quan no siguin coplanàries.

Dues rectes paral·leles sempre seran coplanàries.

Aquí, l’expressió “creuar-se” no té el sentit habitual de “creuar-se amb una

persona pel carrer” que trobem al diccionari:

encreuar - intr. pron. [LC] Dues vies, trobar-se en un punt del seu recorregut i travessar l’una l’altra. Els dos camins s’encreuen a can Gomis. - intr. pron. [LC] Un automòbil, un tren, un missatger, etc., trobar-se en un punt determinat del recorregut amb un altre que va en sentit contrari. El Talgo de París i l’exprés de Barcelona s’encreuaran a l’estació de Girona. Encreuar-se dos correus.

Page 52: Geometria Lineal per al Batxillerat - toomates.net · La història dels vectors. 1.10 Recopilatori d’exercicis. 1.11 Taula resum dels tres productes. 2 Varietats lineals. Rectes

2.6.1 Proposició. Com determinar si dues rectes són coplanàries.

Donada una recta r amb punt base P i vector director v

, i una recta s amb punt base Q i

vector director w

, les dues rectes seran coplanàries si i només si wvPQ

,, són vectors

coplanaris, és a dir, el seu producte mixt (és a dir, el determinant) és zero:

r i s coplanàries 0),,det( wvPQ

2.6.2 Exercicis.

1. Determina si les rectes r i s són o no coplanàries:

12

1:

zy

xr zyxs :

2. Determina si les rectes r i s són o no coplanàries:

022

03:

zyx

zyxr

3

1

1

1

2

1:

zyxs

2.6.3 Problemes.

1. Donades les rectes 2

112:

z

k

yxr i

2

2

1

:

z

y

x

s

Trobeu k per a què r i s siguin coplanàries.

Page 53: Geometria Lineal per al Batxillerat - toomates.net · La història dels vectors. 1.10 Recopilatori d’exercicis. 1.11 Taula resum dels tres productes. 2 Varietats lineals. Rectes

2.6.4 Problemes PAU COU.

1. Quina condició han de complir p i q per tal que les dues rectes r1 i r2 de l’espai

estiguin contingudes en un pla ?

3

2:1

zy

pyxr

qzy

zxr

2

1:2

PAU COU

2. Considereu les dues rectes de l’espai donades per les equacions següents:

43

2

zyx

yx

azyx

yx

2

3

Determineu el paràmetre a per tal que siguin coplanàries. PAU COU

3. Considereu les rectes r1 i r2 de l’espai donades per les equacions següents:

zy

xr

2

1:1 ,

2

11:2

z

b

y

a

xr

a) Calculeu a i b per tal que r1 i r2 siguin paral·leles.

b) Calculeu la relació que hi ha d’haver entre a i b per tal que r1 i r2 pertanyin al mateix

pla. PAU COU

Page 54: Geometria Lineal per al Batxillerat - toomates.net · La història dels vectors. 1.10 Recopilatori d’exercicis. 1.11 Taula resum dels tres productes. 2 Varietats lineals. Rectes

2.7 Resolució de problemes mètrics amb rectes parametritzades.

Molts dels problemes de determinació de punts sobre una recta es resolen seguint el

següent model: Prenem un punt genèric de la recta (és a dir, fem servir la seva equació

paramètrica), imposem sobre aquest punt la condició indicada a l’enunciat, i

simplifiquem fins a obtenir una equació sobre el paràmetre de la recta. Substituint les

solucions obtingudes a l’equació paramètrica obtenim els punts desitjats. Els problemes

resolts següents són exemples concrets que mostren com aplicar aquesta tècnica.

2.7.1 Determinació dels punts d’una recta a una distància fixa d’un punt.

Determineu els punts de la recta

zyx

37

1

3

2

Situats a una distància de 5 unitats del punt )1,4,3( P .

Passem la recta a forma paramètrica:

kz

ky

kxzyx

zyx

3

71

32

1

3

7

1

3

23

7

1

3

2

Un punt qualsevol de la recta serà )3,71,32( kkkQ

Per tant *))3(1())71(4())32(3(),( 222 kkkQPPQDist

168)4()31())3(1(

257049)75()714())71(4(

169)31()323())32(3(

2222

2222

2222

kkkkk

kkkkk

kkkkk

0177259

02542725954272595427259*

2

2222

kk

kkkkkk

32.0

90.0

118

293272

118

117272

592

175947272 2

k

kk

Per tant els punts són:

)1.2,3.5,7.0(

1.2)90,0(3

3.5)90,0(71

7.0)90,0(32

11

Q

z

y

x

Q

)68.2,24.1,04.1(

68.2)32,0(3

24.1)32,0(71

04.1)32,0(32

22

Q

z

y

x

Q

Page 55: Geometria Lineal per al Batxillerat - toomates.net · La història dels vectors. 1.10 Recopilatori d’exercicis. 1.11 Taula resum dels tres productes. 2 Varietats lineals. Rectes

Problemes PAU.

1. Considereu la recta 11

1

2

4:

z

yxr .

a) Trobeu els dos punts, A i B, de la recta r que estan situats a una distància d=√6 del

punt P=(–1, 1, 2).

b) Trobeu l’àrea del triangle de vèrtexs A, B i P. Solució PAU CAT TEC SET 2010 2.6

2. Considereu la recta r d’equació

tz

ty

tx

3

25

23

i el punt M (2, 3, 7).

a) Trobeu, en funció de t, la distància de M a un punt qualsevol de la recta r.

b) Trobeu les coordenades dels punts A i B de r situats a distància 23 del punt M.

c) El triangle ΔAMB, és rectangle en M?

d) Els punts A i B formen part d’un paral·lelogram de vèrtexs ABCD que té el

centre de simetria en el punt M. Calculeu les coordenades de C i D.

Solució PAU CAT TEC JUNY 2004 4.6 (Problema)

3. Considereu la recta r d'equacions: 4

7

3

51

zyx . Calculeu els punts d'aquesta

recta situats a una distància 3 del punt A = (1, 0, 1). Solució PAU CAT TEC JUNY 2002 3.4

4. Considereu la recta de 3IR d’equacions

34

322

zyx

zyx

Trobeu tots els punts de la recta donada, tals que la seva distància a l’origen de

coordenades és 14 . PAU COU

Page 56: Geometria Lineal per al Batxillerat - toomates.net · La història dels vectors. 1.10 Recopilatori d’exercicis. 1.11 Taula resum dels tres productes. 2 Varietats lineals. Rectes

2.7.2 Projecció ortogonal d’un punt en una recta. Distància entre punt i recta.

Volem determinar el punt P de la recta r que és més proper a un punt A donat. Aquest

punt s’anomena projecció ortogonal de A en la recta i verifica una propietat molt

interessant: El punt de la recta amb distància mínima determina un angle recte:

0 vPA

on v

és el vector director de r

Amb aquesta condició podem determinar P, i amb aquest punt, la distància entre A i P:

),(),( PAdistrAdist

Nota: A l’apartat 4.4 resoldrem aquest mateix problema amb tècniques diferents.

Determineu la distància entre el punt )1,2,1( A i la recta definida per les equacions

72

2

zyx

yx

La recta tindrà com a vector director

)1,1,1(1,1,112

11,

12

01,

11

01

112

011

kji

v

Com a punt base fem 0z i per tant obtenim el sistema

(5,-3,0)basePunt 3,572

2

yx

yx

yx

Passem la recta a forma paramètrica:

z

y

x

r 3

5

: Per tant, un punt genèric de la recta serà ),3,5( P

)1,1,4()1,23,15( APAP

Imposem la condició de perpendicularitat:

Page 57: Geometria Lineal per al Batxillerat - toomates.net · La història dels vectors. 1.10 Recopilatori d’exercicis. 1.11 Taula resum dels tres productes. 2 Varietats lineals. Rectes

2036

011)4)(1(0)1,1,1()1,1,4(

vAP

La projecció ortogonal de A en la recta és el punt )2,1,3()2,23,25( P

i la distància del punt A a la recta r serà la distància entre A i P:

611212,21,13),(),( 222 APPAdistrAdis

Problemes.

1. Determineu el punt de la recta

142

12

zyx

zyx més proper al punt )1,3,2( A , i la

distància entre el punt i la recta.

Page 58: Geometria Lineal per al Batxillerat - toomates.net · La història dels vectors. 1.10 Recopilatori d’exercicis. 1.11 Taula resum dels tres productes. 2 Varietats lineals. Rectes

2.7.3 Punt simètric respecte d’una recta.

Donat un punt A i una recta r, volem determinar el punt A’, simètric de A respecte de r.

La projecció ortogonal C de A en r és el punt mig del segment AA’, i per tant

ACAA 2'

Determineu el punt simètric de )7,1,3( A respecte de la recta

2

1

2

31:

zyxr

Passem la recta a equació paramètrica:

Vector director: )2,2,1(v

Punt base: )1,3,1( P

)62,22,2()21,23,1(

21

23

1

:

ACC

z

y

x

r

Imposem la condició de perpendicularitat:

)5,1,3(201890)62(2)22(22

0)2,2,1()62,22,2(

C

vACrAC

Finalment:

)3,3,3()4,4,0(2)7,1,3(2' ACAA

Page 59: Geometria Lineal per al Batxillerat - toomates.net · La història dels vectors. 1.10 Recopilatori d’exercicis. 1.11 Taula resum dels tres productes. 2 Varietats lineals. Rectes

2.7.4 Determinació del punt d’una recta que equidista respecte de dos punts.

Donats els punts )1,1,2(A i )1,0,1( B , i la recta r d’equació 2

25:

zyxr ,

trobeu el punt C de r que equidista de A i B. PAU VALÈNCIA TEC JUNY 2008

En primer lloc passem la recta a forma paramètrica:

22

0

5

)2,1,1(:directorVector

(5,0,-2)P :basePunt

2

25:

z

y

x

v

zyxr

Per tant, un punt genèric de la recta serà )22,,5( P

Imposem sobre aquest punt la condició de l’enunciat:

BPAPPBdistPAdist ),(),(

191662313

23,1,3

2222

AP

APAP

17126214

21,,4

2222

AP

BPBP

1,2

1,

2

9

2

1

171219161712619166

1712619166

22

22

C

BPAP

Problema PAU.

1. Donats els punts P= (1, 0, –1) i Q= (–1, 2, 3), trobeu un punt R de la recta

1

3

3

4

2

3:

zyxr que compleixi que el triangle de vèrtexs P, Q i R és isòsceles,

en què PR i QR són els costats iguals del triangle.

Solució PAU CAT TEC JUNY 2013 4.5

Page 60: Geometria Lineal per al Batxillerat - toomates.net · La història dels vectors. 1.10 Recopilatori d’exercicis. 1.11 Taula resum dels tres productes. 2 Varietats lineals. Rectes

2.7.5 Perpendicular comú a dues rectes.

Donades dues rectes r i s, volem determinar els únics punts P de r i Q de s tals que la

recta PQ és perpendicular a r i a s al mateix temps.

Prenem un punt genèric R de la recta r i prenem un punt genèric S de s i imposem la

condició de la doble perpendicularitat:

wRS

vRS

Amb aquestes dues condicions obtenim els punts P i Q. La recta que busquem serà la

recta que passa per aquests dos punts.

Nota: A l’apartat 4.6.1 resoldrem aquest mateix problema amb tècniques diferents.

Page 61: Geometria Lineal per al Batxillerat - toomates.net · La història dels vectors. 1.10 Recopilatori d’exercicis. 1.11 Taula resum dels tres productes. 2 Varietats lineals. Rectes

Determineu l’equació de la perpendicular comú a les rectes:

3

2

1

1

2

4:

zyxr i

2

8

2

21:

zyxs

Les rectes venen donades per:

)3,1,2(

)2,1,4(:

u

Ar i

)2,2,1(

)8,2,1(:

v

Bs

Agafem un punt genèric R de la recta r i un punt genèric S de la recta s. Hem de fer

servir la equació paramètrica de cada recta, i hem de fer servir dues lletres diferents per

als paràmetres:

32,1,24

32

1

24

:

R

z

y

x

r

28,22,1

28

22

1

:

S

z

y

x

s

Per tant: )2310,23,23( RS

Ara imposem les condicions de perpendicularitat sobre aquest vector:

0)2310(3)23(1)23(20 uRSuRS

0)2310(2)23(2)23(10 vRSvRS

Simplifiquem el sistema de dues equacions amb dues incògnites anterior:

23910

271014

091023

0101427

0462042623

0693023246

Que té com a solució: 22

1

Substituïm per obtenir els punts P i Q:

2

1,

2

1,5P 4,2,1Q

Un vector director de la recta perpendicular serà:

3,1,49,3,122

9,

2

3,6

PQ

i per tant la seva equació contínua serà: 3

4

1

2

4

1:

zyxt

Page 62: Geometria Lineal per al Batxillerat - toomates.net · La història dels vectors. 1.10 Recopilatori d’exercicis. 1.11 Taula resum dels tres productes. 2 Varietats lineals. Rectes

Determineu la perpendicular comú a les rectes

2

1

1

:

z

y

x

r i

1

1:

z

y

x

s

PAU ANDALUCÍA JUNY 2009 4A

Un punt genèric de r és )2,1,1( R i un punt genèric de s és )1,1,( S .

Per tant :

)1,2,1( RS

Imposem les condicions de perpendicularitat :

010 uRS

0210 vRS

Resolem el sistema que hem obtingut:

1 , 2

3

Per tant la recta demanada serà la recta que passa pels punts )1,1,1( P i

1,

2

1,

2

3Q .

El seu vector director serà )0,1,1(0,2

1,

2

1

PQ

I per tant tindrà equació:

1,1

1

1

1

z

yx

Page 63: Geometria Lineal per al Batxillerat - toomates.net · La història dels vectors. 1.10 Recopilatori d’exercicis. 1.11 Taula resum dels tres productes. 2 Varietats lineals. Rectes

2.7.6 Determinació dels punts d’una recta equidistants a dos plans.

1. Sigui r la recta que passa pels punts A = (0, 1, 1) i B = (1, 1, –1).

a) Trobeu l’equació paramètrica de la recta r.

b) Calculeu tots els punts de la recta r que estan a la mateixa distància dels plans π1: x +

y = –2 i π2: x – z = 1.

Nota: Podeu calcular la distància d’un punt de coordenades (x0, y0, z0) al pla d’equació

Ax + By + Cz + D = 0 amb l’expressió 222

000

CBA

DCzAyAx

Solució PAU CAT TEC JUNY 2018 1.2

2. Considereu a IR3 la recta que té per equació r: (x, y, z) = ( –4+2λ , –2 , 1–λ ) i els

plans π1 i π2 d’equacions π1: x + 2y + 2z = –1 i π2: x – 2y + 2z = –3, respectivament.

a) Determineu la posició relativa de π1 i π2.

b) Comproveu que tots els punts de la recta r estan situats a la mateixa distància dels

plans π1 i π2.

Nota: Podeu calcular la distància d’un punt de coordenades (x0, y0, z0) al pla d’equació

Ax + By + Cz + D = 0 amb l’expressió 222

000

CBA

DCzByAx

Solució PAU CAT TEC JUNY 2015 2.4

3. A l’espai tridimensional considereu la recta r: (x, y, z) = (3+2α , –α , 3–α ) i els plans

π1: x + y + z = –1 i π2: (x, y, z) = (2 + λ, 1 – λ + μ, μ).

a) Calculeu l’equació cartesiana (és a dir, que té la forma Ax + By + Cz = D) del pla π2.

b) Trobeu els dos punts de la recta r que equidisten dels plans π1 i π2.

Nota: Podeu calcular la distància d’un punt de coordenades (x0, y0, z0) al pla d’equació

Ax + By + Cz + D = 0 amb l’expressió 222

000

CBA

DCzByAx

Solució PAU CAT TEC SET 2015 5.2

4. Siguin r i s les rectes de IR3

d’equacions 4

1

3

2:

zy

xr i

)34,3,21(),,(: zyxs , amb IR .

a) Comproveu que els punts mitjans dels segments que tenen un extrem situat sobre la

recta r i l’altre extrem situat sobre la recta s formen un pla.

b) Trobeu l’equació general (és a dir, que té la forma Ax+By+Cz=D) del pla de l’apartat

anterior.

Solució PAU CAT TEC JUNY 2014 3.5

5. Trobeu els punts de la recta r: x–1 = y+2 = z que equidisten dels plans

01-3z-4x:1 i 0.1-4y3x:2

Solució PAU CAT TEC JUNY 2007 2.4

Page 64: Geometria Lineal per al Batxillerat - toomates.net · La història dels vectors. 1.10 Recopilatori d’exercicis. 1.11 Taula resum dels tres productes. 2 Varietats lineals. Rectes

6. Determineu els extrems d’un segment AB sabent que el punt A pertany al pla

02 zyx , el punt B pertany a la recta 32

2

2

1 zyx

i el punt mitjà del segment

és (0,0,0).

Solució PAU CAT TEC SET 2006 4.4

7. Trobeu els punts situats sobre la recta d’equacions paramètriques següents:

1

12

z

y

x

i que disten 3

6 del pla 52 zyx .

8. Siguin la recta r: (x, y, z) = (5 + k, k, –2 – 2k) i els punts P = (1, 0, –1) i Q = (2, 1, 1).

a) Calculeu l’equació paramètrica de la recta que passa pel punt Q i és perpendicular al

pla determinat per la recta r i el punt P.

b) Calculeu el punt de la recta r que equidista dels punts P i Q.

Solució PAU CAT TEC SET 2016 1.1

Page 65: Geometria Lineal per al Batxillerat - toomates.net · La història dels vectors. 1.10 Recopilatori d’exercicis. 1.11 Taula resum dels tres productes. 2 Varietats lineals. Rectes

3 Paral·lelisme.

3.1 Paral·lelisme entre dues rectes.

3.1.1 Definicions. Concepte de paral·lelisme en l’espai.

En el pla, diem que dues rectes són paral·leles quan no tenen cap punt en comú.

Es pot demostrar que, en el pla, dues rectes són paral·leles quan són diferents i els seus

vectors directors són proporcionals.

En l’espai, direm que dues rectes són paral·leles quan els seus vectors directors siguin

proporcionals.

En l’espai, dues rectes paral·leles o bé no tenen cap punt de tall o bé són la mateixa

recta, i sempre seran dues rectes coplanàries.

Per tant, en l’espai ens podem trobar amb tres situacions possibles:

1. Dues rectes coincidents:

2. Dues rectes que es tallen en un únic

punt:

3. Dues rectes paral·leles i no diferents

(i que, per tant, no tenen cap punt en

comú)

4. Dues rectes que es creuen, és a dir,

que no siguin coplanàries:

Page 66: Geometria Lineal per al Batxillerat - toomates.net · La història dels vectors. 1.10 Recopilatori d’exercicis. 1.11 Taula resum dels tres productes. 2 Varietats lineals. Rectes

3.1.2 Definició. Rectes paral·leles.

Dues rectes són paral·leles, i escriurem sr // , si els seus vectors directors són

proporcionals, és a dir, determinen la mateixa direcció.

wkvwvsr

wwwbqqqzyxs

vvvapppzyxr

////

),,(),,(),,(:

),,(),,(),,(:

321321

321321

3.1.3 Proposició.

Si les rectes venen donades per les equacions cartesianes o implícites:

'''':

DzCyBxA

DCzByAxr

''''''''''''

'''''''':

DzCyBxA

DzCyBxAs

0

'''''''''

'''0

''''''

'''2

'''''''''

''''''

'''//

CBA

CBA

CBA

i

CBA

CBA

CBA

CBA

CBA

CBA

CBA

Rangsr

3.1.4 Proposició. Recta paral·lela a una altra i que passa per un punt.

Donada una recta vkQr

: i un punt rP , existeix una única recta s paral·lela a r i

que passa per P.

Agafem com a punt base el punt P i

com a vector director el vector director

de la recta r.

Page 67: Geometria Lineal per al Batxillerat - toomates.net · La història dels vectors. 1.10 Recopilatori d’exercicis. 1.11 Taula resum dels tres productes. 2 Varietats lineals. Rectes

3.1.5 Exercicis.

1. Determina l’equació contínua de la recta paral·lela a la recta 4

5

3

2

2

3 zyx

que passa pel punt )3,5,1( P .

2. Donats els punts )4,2,3( P , )2,1,5( Q y )3,4,1(R , determina la recta

paral·lela a la recta PQ que passa pel punt R.

3. Donats els punts )7,3,2(P , )3,1,4(Q y )4,5,3( R ,

a) Determina la recta paral·lela a la recta PQ que passa pel punt R

b) Determina punt d’intersecció entre aquesta recta i el pla 0z .

4. Donats els punts )2,1,3( P , )1,3,2(Q y )3,5,1( R ,

a) Determina la recta paral·lela a la recta PQ que passa pel punt R.

b) Determina punt d’intersecció entre aquesta recta i el pla 0z .

3.1.6 Problemes PAU.

1. Donada la recta

01

232:

zx

zyxr

a) Trobeu-ne un vector director.

b) Calculeu l’equació contínua de la recta que és paral·lela a r i que passa pel punt

)1,0,1( P

Solució PAU CAT TEC JUNY 2011 1.1

2. Calculeu l’equació de la recta paral·lela a la recta

12

0:

zyx

zyxr

que passa pel punt (0,1,0).

Solució PAU CAT TEC SET 2006 4.3

3. Considereu la recta de 3IR d’equacions

53

13

zyx

zyx

Digueu si el punt )2,2,6( es troba o no sobre la recta paral·lela a l’anterior que passa per

l’origen. Raoneu la resposta.

PAU COU

4. Donades les rectes zy

k

xr

1: i

1

2:

zx

yxks , determineu el valor de k per tal

que les rectes siguin paral·leles.

Solució: -1k PAU COU

Page 68: Geometria Lineal per al Batxillerat - toomates.net · La història dels vectors. 1.10 Recopilatori d’exercicis. 1.11 Taula resum dels tres productes. 2 Varietats lineals. Rectes

5. Considereu les dues rectes r i s de 3IR donades per les equacions següents:

0

0:

zya

yxr i

azy

yaxs

1:

Digueu si hi ha algun valor d’a per al qual r i s es tallin, i si hi ha algun valor d’a per al

qual siguin paral·leles. Justifiqueu la resposta.

Solució: Paral·leles si a=1 PAU COU

6. Considereu les dues rectes r i s de 3IR donades per les equacions següents:

222

1:

zyax

zayxr i

2

5323:

zx

zyxbs

Determineu els valors d’a i b sabent que r i s són paral·leles.

PAU COU

Page 69: Geometria Lineal per al Batxillerat - toomates.net · La història dels vectors. 1.10 Recopilatori d’exercicis. 1.11 Taula resum dels tres productes. 2 Varietats lineals. Rectes

3.2 Paral·lelisme entre plans.

3.2.1 Definició.

Direm que dos plans 1 i 2 són paral·lels quan són el mateix pla ( 21 ) o no tenen

cap punt en comú:

21

Dos plans DCzByAx :1 i '''':2 DzCyBxA seran paral·lels quan els

seus vectors normals siguin proporcionals:

'''

)',','(),,(

)',','(//),,(

// 21

C

C

B

B

A

A

CBAkCBA

CBACBA

3.2.2 Proposició.

Si els plans els tenim en forma vectorial:

),,(),,(),,(),,(: 3213213211 vvvbuuuapppzyx

),,(),,(),,(),,(: 3213213212 tttdwwwcqqqzyx

2//

3333

2222

1111

21

twvu

twvu

twvu

Rang

3.2.3 Proposició. Pla paral·lel a un altre i que passa per un punt.

Donat un pla DCzByAx i un punt P que no pertany al pla, existeix un únic pla

paral·lel a aquest i que passa per P.

Agafarem com a vector normal el vector

normal del pla i deduirem el valor D

amb el punt P.

Page 70: Geometria Lineal per al Batxillerat - toomates.net · La història dels vectors. 1.10 Recopilatori d’exercicis. 1.11 Taula resum dels tres productes. 2 Varietats lineals. Rectes

3.2.4 Exercicis.

1. Determina el pla que passa per )2,3,4(Q i és paral·lel al pla zyx 426:

2. Determina el pla que passa per )3,2,1( Q i és paral·lel al pla

5452: zyx

3.2.5 Problemes PAU.

1. Un segment d’extrems A= (5, 3, 1) i B= (4, 2, –1) es divideix en tres parts iguals

mitjançant dos plans perpendiculars a aquest segment. Calculeu les equacions dels dos

plans i la distància entre ells. Solució PAU CAT TEC SET 2003 3.6

2. Quines condicions han de complir els coeficients de les equacions cartesianes de dos

plans de l’espai per tal que siguin paral·lels? Justifiqueu la resposta. Com a exemple,

determineu els valors de a que fan que els dos plans d’equacions

5)2( zayaxa i 3 zyxa

siguin paral·lels. Per a cada un d’aquests valors de a calculeu la distància entre els dos

plans paral·lels.

PAU COU

Page 71: Geometria Lineal per al Batxillerat - toomates.net · La història dels vectors. 1.10 Recopilatori d’exercicis. 1.11 Taula resum dels tres productes. 2 Varietats lineals. Rectes

3.3 Paral·lelisme entre recta i pla.

Direm que la recta r és paral·lela al pla quan no tinguin cap punt en comú:

r

Un pla serà paral·lel a una recta quan el

vector director de la recta i el vector

normal del pla siguin perpendiculars.

0),,(),,(

),,(//

:

),,(),,(:

321

321321

CBAvvv

CBAvr

DCzByAx

vvvapppvkPr

Observació: A més a més, hem de

comprovar que el punt P no pertanyi al

pla.

Problemes PAU.

1. Determineu per a quins valors del paràmetre a el pla π : ax +2y +z = a és

paral·lel a la recta r :

1

1

azax

zayx

Solució PAU CAT TEC SET 2001 4.1

2. Digueu si existeix cap valor de per al qual la recta següent :

354

1232

zyx

zyx és

paral·lela al pla 51312 zyx . Expliqueu bé el perquè de la vostra resposta.

Solució: 1 , 4

PAU COU

Page 72: Geometria Lineal per al Batxillerat - toomates.net · La història dels vectors. 1.10 Recopilatori d’exercicis. 1.11 Taula resum dels tres productes. 2 Varietats lineals. Rectes

3.4 Pla paral·lel a dues rectes i que passa per un punt.

Donades dues rectes vPr

: i wQs

: , que es creuen, i un punt R, existeix un únic

pla que passa per R i és paral·lel a les dues rectes.

El vector normal del pla serà

perpendicular als vectors directors de

les rectes, per tant, prendrem el seu

producte vectorial:

321

321

www

vvv

kji

wvn

I com a punt base del pla prendrem el

punt R.

Problemes PAU.

1. Donades les rectes

0122

04:1

zyx

zyxr i z

yxr

3:2

Calculeu l'equació del pla paral·lel a les dues rectes que passa per l'origen.

PAU CAT TEC SET 1999 5.2

Page 73: Geometria Lineal per al Batxillerat - toomates.net · La història dels vectors. 1.10 Recopilatori d’exercicis. 1.11 Taula resum dels tres productes. 2 Varietats lineals. Rectes

3.5 Pla que conté una recta i és paral·lel a una altra.

Donades dues rectes vPr

: i wQs

: , que es creuen, existeix un únic pla que

conté a r i és paral·lel a s.

El vector normal del pla serà

perpendicular als vectors directors de

les rectes, per tant, prendrem el seu

producte vectorial:

321

321

www

vvv

kji

wvn

I com a punt base del pla prendrem el

punt base de la recta.

Problemes PAU.

1. Trobeu l’equació general (és a dir, de la forma 0 DCzByAx ) del pla que

conté la recta zyx

r

22

1:1 i és paral·lel a la recta

02

0:2

zyx

zyxr

Solució PAU CAT TEC JUNY 2010 1.1

2. Determineu el valor de a per al qual el pla de 3IR d’equació 1 zyx és

paral·lel a la recta

32

523

zyx

zayx. Escriviu després l’equació del pla que conté

aquesta recta i és paral·lel al pla .

Solució: 0a . El pla és 2 zyx

PAU COU

3. Trobeu l’equació cartesiana del pla de l’espai que conté la recta d’equacions

)1(23

2

z

yx i és paral·lel a la recta

02

632

zyx

zyx

Solució: 554 zyx

PAU COU

Page 74: Geometria Lineal per al Batxillerat - toomates.net · La història dels vectors. 1.10 Recopilatori d’exercicis. 1.11 Taula resum dels tres productes. 2 Varietats lineals. Rectes

3.6 Recta paral·lela a dos plans.

Direm que la recta r és paral·lela a dos plans 1 i 2 quan no tingui cap punt en comú

amb ells:

1r , 2r

Donat un punt P que no pertany als plans 1 i 2 , existeix una única recta r paral·lela a

tots dos plans i que passa per P.

El vector director v

de la recta és

perpendicular als dos vectors normals

dels plans, per tant podem agafar com a

vector director de la recta el producte

vectorial dels vectors normals dels

plans:

321

21

mmm

nnn

kji

mnv e

Exercici.

1. Trobeu les equacions de la recta que és paral·lela als plans 0 yx i 0 zx i

passa pel punt )0,0,2( .

Page 75: Geometria Lineal per al Batxillerat - toomates.net · La història dels vectors. 1.10 Recopilatori d’exercicis. 1.11 Taula resum dels tres productes. 2 Varietats lineals. Rectes

4 Perpendicularitat.

4.1 Perpendicularitat entre plans.

4.1.1 Definició. Perpendicularitat entre dos plans.

Dos plans són perpendiculars si els vectors normals a cada pla són perpendiculars entre

ells.

0)',','(),,(

)',','(),,(

'''':

:

21

2

1

CBACBA

CBACBA

DzCyBxA

DCzByAx

4.1.2 Problemes.

1. Responeu a les qüestions següents:

a) Calculeu l’equació cartesiana (és a dir, que té la forma Ax + By + Cz = D) del pla que

passa pel punt de coordenades (0, 0, 1) i és perpendicular als plans 3x + y – z = 1 i x + y

+ 2z = 5.

b) Suposeu que un pla π1 és perpendicular a un segon pla π2 i que el pla π2 és a la

vegada perpendicular a un tercer pla π3. Expliqueu raonadament si necessàriament els

plans π1 i π3 han de ser perpendiculars entre ells.

Solució PAU CAT TEC SET 2016 1.6

2. Siguin π1el pla 2x +3y –z=4 i π2 el pla x– 2y– 4z= 10.

a) Comproveu que els plans π1 i π2 són perpendiculars.

b) Trobeu l’equació contínua de la recta paral·lela als plans π1 i π2 i que passa pel punt

P= (–1, 3, 2).

Solució PAU CAT TEC JUNY 2013 5.1

Page 76: Geometria Lineal per al Batxillerat - toomates.net · La història dels vectors. 1.10 Recopilatori d’exercicis. 1.11 Taula resum dels tres productes. 2 Varietats lineals. Rectes

4.2 Perpendicularitat entre dues rectes.

4.2.1 Definició. Perpendicularitat entre dues rectes.

Dues rectes són perpendiculars si els seus vectors directors són perpendiculars.

0

),,(),,(),,(:

),,(),,(),,(:

321321

321321

wvwvsr

wwwbqqqzyxs

vvvapppzyxr

4.2.2 Problemes PAU.

1. Siguin 2

1

2

32::1

zyxr

i

2

11

2

3:2

zy

xr .

a) Comproveu que r1 i r2 són perpendiculars.

b) Comproveu que es tallen mitjançant la determinació del punt de tall.

Solució PAU CAT TEC JUNY 2011 1.5

2. Considereu en l’espai IR3 les rectes r i s, les equacions respectives de les quals són:

)1,1,()0,1,4(),,(: mzyxr ,

1

02:

zyx

zmyxs

en què m és un paràmetre real. Estudieu si hi ha cap valor d’aquest paràmetre per al qual

les rectes siguin perpendiculars i es tallin.

Solució PAU CAT TEC SET 2009 1.2

Page 77: Geometria Lineal per al Batxillerat - toomates.net · La història dels vectors. 1.10 Recopilatori d’exercicis. 1.11 Taula resum dels tres productes. 2 Varietats lineals. Rectes

4.3 Perpendicularitat entre recta i pla.

4.3.1 Definició. Perpendicularitat entre recta i pla.

Una recta és perpendicular a un pla quan el seu vector director és proporcional al vector

normal al pla.

),,(),,(),,//(

:

),,(),,(:

321

321321

CBAkvvvCBAvr

DCzByAx

vvvapppvkPr

4.3.2 Proposició. Recta que passa per un punt i és perpendicular a un pla.

Donat un punt ),,( 321 pppP i un pla DCzByAx : , volem una recta r que

passi pel punt i sigui perpendicular al pla.

Agafarem com a vector director de la recta el vector normal del pla i com a punt base de

la recta el punt P.

4.3.3 Proposició. Pla que passa per un punt i és perpendicular a una recta.

Donat un punt P i una recta r, volem un pla que passi pel punt i sigui perpendicular a la

recta.

Agafarem com a vector normal del pla el vector director de la recta, i com a punt base

del pla el punt P.

4.3.4 Problemes.

1. Sigui r la recta de l’espai que té per equació zyx

r

1

3

2

1: i sigui P el punt de

coordenades (6, 0, –1).

a) Trobeu l’equació cartesiana (és a dir, que té la forma Ax + By + Cz = D) del pla que

passa pel punt P i talla perpendicularment la recta r.

b) Trobeu l’equació paramètrica del pla que passa pel punt P i conté la recta r.

Solució PAU CAT TEC JUNY 2015 2.2

2. Sigui π: 3x –2y +z = 10.

a) Trobeu l’equació contínua de la recta r perpendicular a π que passa pel punt

P= (–1, 3, 2).

b) Trobeu també l’equació cartesiana (és a dir, de la forma Ax+By+Cz+D= 0) del pla π1

paral·lel a π que passa pel mateix punt P. Solució PAU CAT TEC JUNY 2013 3.1

3. Considereu els punts A= (–1, 2, 4) i B= (3, 0, –2).

a) Trobeu l’equació del pla format per tots els punts que equidisten de A i B.

b) Donat un punt C = (x,y,z), dividim el segment AC en tres parts iguals i obtenim els

punts A, A1, B i C. Trobeu el punt C. Solució PAU CAT TEC JUNY 2013 5.5

Page 78: Geometria Lineal per al Batxillerat - toomates.net · La història dels vectors. 1.10 Recopilatori d’exercicis. 1.11 Taula resum dels tres productes. 2 Varietats lineals. Rectes

4. Considereu les rectes de l’espai següents:

1

11

2

1:

zy

xr

2

2

1

1

3

4:

zyxs

a) Comproveu que són secants.

b) Calculeu l’equació contínua de la recta que les talla i que és perpendicular a totes

dues. Solució PAU CAT TEC SET 2012 4.5

5. Donada la recta

01

232

zx

zyx, calculeu l’equació general (és a dir, de la forma

Ax+By+Cz+D=0) del pla perpendicular a la recta que passa pel punt P=(1, 0, –1).

Solució PAU CAT TEC SET 2011 2.2

6. Donats el pla π: x +2y +3z –4 = 0 i els punts P= (3, 1, –2) i Q= (0, 1, 2):

a) Calculeu l’equació contínua de la recta perpendicular al pla π que passa pel punt P.

b) Calculeu l’equació general (és a dir, de la forma Ax+By+Cz+D= 0) del pla

perpendicular a π que passa pels punts P i Q.

Solució PAU CAT TEC JUNY 2010 4.1

7. Donats el punt P= (1, 2, 3) i la recta 1

5

3

2

2

1:

zyxr :

a) Trobeu l’equació cartesiana (és a dir, de la forma Ax+By+Cz+D=0) del pla π que

passa per P i és perpendicular a la recta r.

b) Trobeu el punt de tall entre la recta r i el pla π. Solució PAU CAT TEC JUNY 2009 4.1

8. Trobeu l’equació de la recta perpendicular al pla π: 2x –y +z +3 = 0, que passa pel

punt (–1, 3,a) del pla. Solució PAU CAT TEC JUNY 2008 2.4

9. Trobeu l’equació del pla perpendicular a la recta

32

1:

yx

zyxr que passa per

l’origen de coordenades. Solució PAU CAT TEC JUNY 2007 2.1

10. Donats els punts de l'espai A = (2, 0, 0), B = (0, 1, 0) i C = (0, 0, 3).

a) Determineu l'equació del pla π que els conté.

b) Calculeu l'equació de la recta r perpendicular al pla π i que passa per l'origen.

Solució PAU CAT TEC JUNY 2001 2.3

Page 79: Geometria Lineal per al Batxillerat - toomates.net · La història dels vectors. 1.10 Recopilatori d’exercicis. 1.11 Taula resum dels tres productes. 2 Varietats lineals. Rectes

4.4 Projecció ortogonal d’un punt en una recta. Punt simètric respecte

d’una recta.

4.4.1 Definició. Projecció ortogonal d’un punt en una recta.

La projecció ortogonal d’un punt P

sobre una recta r és un altre punt Q que

pertany a la recta, i de tal manera que el

vector PQ és perpendicular al vector

director de la recta.

4.4.2 Proposició. Mètode de determinació de la projecció ortogonal d’un punt en

una recta.

Primer pas: Trobem l’equació del pla

perpendicular a la recta r que passa pel

punt P.

Segon pas: Calculem el punt Q tall de la

recta amb el pla, i aquest punt serà la

projecció ortogonal.

Nota: A l’apartat 2.7.2 vam resoldre aquest mateix problema mitjançant rectes

parametritzades.

Exercicis.

1. Determina la projecció del punt )5,3,3( P sobre la recta

53

2

3

2:

z

yxr .

2. Determina la projecció del punt )3,2,0( P sobre la recta 25

5

4

4:

z

yxr .

3. Determina la projecció del punt )3,2,5( P sobre la recta

23

2

3

4:

zyxr

Page 80: Geometria Lineal per al Batxillerat - toomates.net · La història dels vectors. 1.10 Recopilatori d’exercicis. 1.11 Taula resum dels tres productes. 2 Varietats lineals. Rectes

4.4.3 Proposició. Punt simètric respecte d’una recta.

Primer pas: Determinem la projecció

ortogonal Q de P en r.

Segon pas: Fem PQPS 2

Exercicis.

1. Determina el punt simètric del punt )2,5,0( P respecte de la recta

4

3

2

43:

zyxr .

2. Determina el punt simètric del punt )2,1,2(P respecte de la recta

2

25

3

2:

zy

xr .

3. Considera el punt )0,2,3(P i la recta

012

03:

zx

zyxr . Determina les

coordenades del punt Q simètric de P respecte de la recta r.

4.4.4 Problemes PAU.

1. Donats el punt P= (1, 0, –2) i la recta 3

3

2

3

2

5:

zyxr :

a) Trobeu l’equació contínua de la recta que passa pel punt P i talla perpendicularment

la recta r.

b) Calculeu la distància del punt P a la recta r.

Solució PAU CAT TEC JUNY 2010 5.2

2. Considereu el punt P = (5, –2, 9) i la recta 63

1

2

1:

zyxr

a) Calculeu l’equació de la recta s que talla perpendicularment r i que passa per P.

b) Calculeu el punt de tall T entre les rectes r i s. Solució PAU CAT TEC JUNY 2003 2.3

3. Calculeu el peu de la recta perpendicular a la recta (x, y, z) = (1, –1, 1) + λ(0, 1, 1)

traçada des del punt (1, 0, –1).

Solució CAT TEC SET 2000 6.3

Page 81: Geometria Lineal per al Batxillerat - toomates.net · La història dels vectors. 1.10 Recopilatori d’exercicis. 1.11 Taula resum dels tres productes. 2 Varietats lineals. Rectes

4. Considereu la recta r de 3IR d’equacions 6

33

3

33

3

33 xyx

. Calculeu les

coordenades del peu de la perpendicular traçada des de l’origen de coordenades a la

recta r i calculeu després la distància de l’origen a aquesta recta. PAU COU

5. Preneu tres eixos perpendiculars de coordenades a l’espai. Trobeu el peu de la

perpendicular traçada des de l’origen fins a la recta que passa pel punt (-1,7,1) i que té

(1,-1,2) com a vector director.

PAU COU

6. Considereu la recta r de 3IR d’equacions:

22

322

yx

zyx

Trobeu el peu de la perpendicular traçada des del punt P=(5,5,3) a la recta r, i calculeu

la distància entre P i la recta r.

Page 82: Geometria Lineal per al Batxillerat - toomates.net · La història dels vectors. 1.10 Recopilatori d’exercicis. 1.11 Taula resum dels tres productes. 2 Varietats lineals. Rectes

4.5 Projecció ortogonal d’un punt en un pla. Punt simètric respecte

d’un pla.

4.5.1 Definició. Projecció ortogonal d’un punt en un pla.

La projecció ortogonal d’un punt P

sobre un pla és un punt Q que

pertany al pla, i tal que la recta PQ és

perpendicular al pla.

4.5.2 Proposició. Mètode de determinació de la projecció ortogonal d’un punt en

un pla.

Primer pas: Trobem l’equació de la

recta r perpendicular al pla que passa

per P

Segon pas: Calculem el punt de tall del

pla amb la recta r, i aquest punt serà la

projecció ortogonal.

Exercicis.

1. Determina la projecció ortogonal del punt )3,4,0( P en el pla zyx 245: .

2. Determina la projecció ortogonal del punt )3,2,4( P en el pla

632: zyx .

3. Determina la projecció ortogonal del punt )2,3,3( P en el pla

zyx 2535: .

4. Determina la projecció ortogonal del punt )3,5,4( P en el pla 7: yzx .

Page 83: Geometria Lineal per al Batxillerat - toomates.net · La història dels vectors. 1.10 Recopilatori d’exercicis. 1.11 Taula resum dels tres productes. 2 Varietats lineals. Rectes

4.5.3 Proposició. Punt simètric respecte d’un pla.

Primer pas: Trobem la projecció

ortogonal Q del punt P

Segon pas: Fem PQPR 2

Exercicis.

1. Determina el punt simètric del punt )0,1,4( P respecte del pla

5255: zyx

2. Determina el punt simètric del punt )2,4,2(P respecte del pla 1134: zyx .

3. Determina el punt simètric del punt )4,0,2( P respecte del pla

1425: zyx .

Problemes PAU.

1. Un dron es troba en el punt P = (2, –3, 1) i volem dirigir-lo en línia recta fins al punt

més proper del pla d’equació π : 3x + 4z + 15 = 0.

a) Calculeu l’equació de la recta, en forma paramètrica, que ha de seguir el dron.

Quina distància ha de recórrer fins a arribar al pla?

b) Trobeu les coordenades del punt del pla on arribarà el dron.

Nota: Podeu calcular la distància que hi ha d’un punt de coordenades (x0, y0, z0) al pla

d’equació Ax + By + Cz + D = 0 amb l’expressió

222

000

CBA

DCzByAx

PAU CAT TEC JUNY 2019 1.3 (Solució: "Compendium Tec", Pàg. 532)

2. A IR3, siguin la recta

42

2

zy

zx i el punt P = (0, 1, –1).

a) Calculeu l’equació general (és a dir, la que té la forma Ax + By + Cz = D) del pla π

perpendicular a la recta r i que passa pel punt P.

b) Calculeu el punt simètric del punt P respecte del pla x + y + z = –3.

Solució PAU CAT TEC SET 2017 2.5

Page 84: Geometria Lineal per al Batxillerat - toomates.net · La història dels vectors. 1.10 Recopilatori d’exercicis. 1.11 Taula resum dels tres productes. 2 Varietats lineals. Rectes

3. Considereu el punt A=(1, 2, 3).

a) Calculeu el punt simètric del punt A respecte de la recta d’equació

).-3,1,3(z)y,(x,:r

b) Calculeu el punt simètric del punt A respecte del pla que té per equació

3: zyx .

Solució PAU CAT TEC JUNY 2014 3.2

4. Donats el pla π: 2x –y +3z –8 = 0 i el punt P=(6, –3, 7),

a) Trobeu l’equació contínua de la recta que passa per P i és perpendicular a π.

b) Trobeu el punt del pla π que està més proper al punt P. Solució PAU CAT TEC SET 2013 1.5

5. Donats el pla π: x +2y –z = 0 i el punt P= (3, 2, 1):

a) Calculeu l’equació contínua de la recta r que passa per P i és perpendicular a π.

b) Calculeu el punt simètric del punt P respecte del pla π.

Solució PAU CAT TEC JUNY 2009 3.4

Page 85: Geometria Lineal per al Batxillerat - toomates.net · La història dels vectors. 1.10 Recopilatori d’exercicis. 1.11 Taula resum dels tres productes. 2 Varietats lineals. Rectes

4.6 Recta perpendicular comú a dues rectes.

4.6.1 Proposició. Determinació de la perpendicular comú amb intersecció de plans.

Donades la recta r amb vector director

v

, i la recta s amb vector director w

,

volem obtenir la única recta de l’espai

que talla r i s i és perpendicular a totes

dues.

Pas 1: Busquem un vector que sigui

perpendicular a les dues rectes, per tant,

calculem el producte vectorial dels dos

vectors directors:

wvu

Pas 2: Determinem el pla 1 que conté

a r i la direcció u

.

Pas 3: El punt Q serà el punt de tall

entre aquest pla 1 i la recta s.

Pas 4: Determinem el pla 2 que conté

a s i la direcció u

.

Pas 5: El punt P serà el punt de tall

entre aquest pla 2 i la recta r.

La perpendicular comú serà la

intersecció dels dos plans anteriors:

21

O també és la recta que passa per P i Q

Nota: A l’apartat 2.7.5 vam resoldre aquest mateix problema mitjançant rectes

parametritzades.

Page 86: Geometria Lineal per al Batxillerat - toomates.net · La història dels vectors. 1.10 Recopilatori d’exercicis. 1.11 Taula resum dels tres productes. 2 Varietats lineals. Rectes

Considereu la recta r que passa pel punt (1,2,0) i que té (1,1,1) com a vector director, i

la recta s que passa per (0,0,1) i que té (1,2,1) com a vector director. Escriviu les

equacions paramètriques de la recta que talla les dues anteriors i que és perpendicular a

cadascuna.

Pas 1. Vector perpendicular als dos vectors directors. )1,0,1(

121

111

kji

wvu

Pas 2. Pla que conté la recta r i la direcció u

.

Punt base: )0,2,1(A

Vectors directors: )1,2,1(

101

111)1,0,1(

)1,1,1(1

kji

uvnu

v

3230)2(2)1(121 zyxDDDzyx

Pas 3. El punt Q és el punt de tall entre el pla anterior i la recta s.

321

4220

2210

21323)1()2(21

1

20

10

:

z

y

x

kkkkk

kz

ky

kx

s

)3,4,2(Q

Pas 4. Pla que conté la recta s i la direcció u

.

Punt base: )1,0,0(B

Vectors directors: )1,1,1()2,2,2(

101

121)1,0,1(

)1,2,1(2

kji

uwnu

w

11100111 zyxDDDzyx

Pas 5. El punt P és el punt de tall entre el pla anterior i la recta r.

2210

422

321

212111)2(1

10

12

11

:

z

y

x

kkkkk

kz

ky

kx

r

)2,4,3(P

Pas 6. La perpendicular comú és la recta que passa per P i Q

Punt base: )2,4,3(P

Vector director: )1,0,1( PQPQ

kz

y

kx

2

4

3

Page 87: Geometria Lineal per al Batxillerat - toomates.net · La història dels vectors. 1.10 Recopilatori d’exercicis. 1.11 Taula resum dels tres productes. 2 Varietats lineals. Rectes

Determineu la perpendicular comú de les rectes

12

1:

zx

zyxr i

2

1

1

2:

zyxs

Passem les dues rectes a forma vectorial:

Recta r :

)1,1,2()1,1,2(

201

111

kji

v

Punt base )0,2,1(A (fent z=0)

Recta s:

)2,1,1( w

Punt base: )1,2,0( B

1. Vector perpendicular a v

i w

: )1,3,1()1,3,1(

211

112

kji

wvu

2. Pla 1 que conté la recta r i la direcció u

:

Punt base del pla: )0,2,1(A

Vectors directors del pla:

)7,1,4()7,1,4(

131

112)1,3,1(

)1,1,2(1

kji

uvnu

v

El pla té per equació Dzyx 74 , i substituint amb A obtenim la D:

274:2072)1(4 1 zyxDD

3. El punt Q serà la intersecció entre 1 i la recta s:

11

3,

11

15,

11

7

11

721,

11

72,

11

7

11

7

2147242)21(7)2(4

21

2

0

:

Q

z

y

x

s

4. Pla 2 que conté la recta s i la direcció u

:

Punt base del pla: )1,2,0( B

Vectors directors del pla:

)4,1,7(

131

211)1,3,1(

)2,1,1(2

kji

uwnu

w

El pla té per equació 247 Dzyx , i substituint amb B obtenim la 2D :

247:214)2(07 222 zyxDD

Page 88: Geometria Lineal per al Batxillerat - toomates.net · La història dels vectors. 1.10 Recopilatori d’exercicis. 1.11 Taula resum dels tres productes. 2 Varietats lineals. Rectes

5. El punt P serà la intersecció entre 2 i la recta r:

11

7,

11

15,

11

3

11

7,

11

72,

11

721

11

7

24214724)2()21(7

0

12

21

:

P

z

y

x

s

Més endavant, a l’apartat 5.5.2, veurem que els punts P i Q que acabem de trobar són

molt útils per calcular la distància entre les dues rectes, perquè la distància entre les

dues rectes serà la distància entre P i Q:

11

10,

11

30,

11

10

11

7

11

3,

11

15

11

15,

11

3

11

7PQPQ

02.311

1110

11

10

11

100

11

10

11

30

11

10),(),(

222

PQQPdistsrdist

Determineu l’equació de la perpendicular comú a les rectes:

3

2

1

1

2

4:

zyxr i

2

8

2

21:

zyxs

Les rectes venen donades per:

)3,1,2(

)2,1,4(:

v

Ar i

)2,2,1(

)8,2,1(:

w

Bs

3,1,421

12,

21

32,

22

31

221

312

kji

wvu

Pla :1

019930382186

042182460)2(2)1(18)4(6

0)2(14

12)1(

34

32)4(

31

31

0

314

312

214

)3,1,4(

)3,1,2(

)2,1,4(

:1

zyxzyx

zyxzyx

zyx

zyx

u

v

A

Page 89: Geometria Lineal per al Batxillerat - toomates.net · La història dels vectors. 1.10 Recopilatori d’exercicis. 1.11 Taula resum dels tres productes. 2 Varietats lineals. Rectes

Pla :2

0427118

05672211880)8(7)2(11)1(8

0)8(14

21)2(

34

21)1(

31

22

0

314

221

821

)3,1,4(

)2,2,1(

)8,2,1(

:2

zyx

zyxzyx

zyx

zyx

u

w

B

Per tant, la perpendicular comú serà la intersecció dels dos plans anteriors, i per tant la

seva equació cartesiana serà:

0427118

01993:

zyx

zyxt

Font: I.E.S. “Miguel de Cervantes” (Granada)

4.6.2 Exercicis.

1. Recta perpendicular comú a dues rectes que es creuen. A cada apartat, determineu la

perpendicular comú de les dues rectes:

a) zyxr 1: , zyxs : .

b) 2

8

2

9

3

3

zyx ,

2

1

1

2

2

3

zyx

c)

02

02:

z

yxr ,

064

0:

zy

yxs

d)

1

1:

zy

zxr ,

3

1

:

z

ty

tx

s e)

2

1

1

z

y

x

,

1

1

z

y

x

4.6.3 Problemes COU.

1. Doneu les equacions paramètriques de la recta que talla perpendicularment les rectes

1

3

1

:

z

y

x

r 24

4

2

4:

z

yxs

i doneu també els punts d’intersecció d’aquesta recta amb r i s. PAU COU

Page 90: Geometria Lineal per al Batxillerat - toomates.net · La història dels vectors. 1.10 Recopilatori d’exercicis. 1.11 Taula resum dels tres productes. 2 Varietats lineals. Rectes

2. Considereu la recta r de 3IR que passa pel punt )1,1,1( i té )0,1,2( com a vector

director, i la recta r’ que passa per )2,2,2( i té )1,1,0( com a vector director. Calculeu el

punt P de r i el punt P’ de r’ que compleixen la condició que la recta 'PP és

perpendicular a r i a r’. PAU COU

3. Considereu la recta que passa pel punt (1,2,0) i que té (1,1,1) com a vector director, i

la recta que passa per (0,0,1) i que té (1,2,1) com a vector director. Escriviu les

equacions paramètriques de la recta que talla les dues anteriors i que és perpendicular a

cadascuna. PAU COU

4. Considereu la recta r de 3IR d’equacions paramètriques

6

3

3

z

y

x

i la recta r’

d’equacions 2

1

2

11

zyx .

Doneu les coordenades del punt A de r i del punt B de r’ que compleixen la condició

que la recta AB és perpendicular a r i a r’. PAU COU

Page 91: Geometria Lineal per al Batxillerat - toomates.net · La història dels vectors. 1.10 Recopilatori d’exercicis. 1.11 Taula resum dels tres productes. 2 Varietats lineals. Rectes

4.7 Problemes PAU de perpendicularitat i paral·lelisme.

1. Siguin les rectes 51

21:1

z

yxr i r2: (x, y, z) = (2 – 3λ, –1 + λ, 2).

a) Trobeu l’equació cartesiana (és a dir, que té la forma Ax + By + Cz = D) del pla que

conté la recta r1 i és paral·lel a la recta r2.

b) Digueu quina condició s’ha de complir perquè existeixi un pla que contingui la recta

r1 i sigui perpendicular a la recta r2. Amb les rectes r1 i r2 de l’enunciat, comproveu si

existeix un pla que contingui la recta r1 i sigui perpendicular a la recta r2.

Solució PAU CAT TEC JUNY 2018 5.5

2. Considereu els punts P = (3, –2, 1), Q = (5, 0, 3), R = (1, 2, 3) i la recta

0532

01:

zy

yxr

a) Determineu l’equació general (és a dir, que té la forma Ax + By + Cz = D) del pla

que passa per P i Q i és paral·lel a la recta r.

b) Donats el pla x + 2y + m · z = 7 i el pla que passa per P, Q i R, trobeu m perquè

siguin paral·lels i no coincidents.

Solució PAU CAT TEC JUNY 2018 1.4

3. Donats el pla 052: zyx i la recta

102

0:

zyx

zyxr

a) Calculeu el punt d’intersecció entre el pla i la recta.

b) Trobeu l’equació contínua de la recta s continguda en el pla π, que és perpendicular a

la recta r i talla la recta r.

Solució PAU CAT TEC JUNY 2012 3.3

4. Considereu les rectes r i s amb les equacions següents:

022

03:

zx

yxr

032

03

1

:

zx

ys

a) Calculeu, de cada una de les rectes, un punt i un vector director.

b) Determineu si existeix cada un dels objectes següents i en cas afirmatiu calculeu la

seva equació:

i) El pla paral·lel a la recta s que conté la recta r.

ii) El pla perpendicular a la recta s que conté la recta r.

iii) La recta perpendicular a les rectes r i s que passa per (0, 0, 0).

Solució PAU CAT TEC JUNY 2002 2.5 (Problema)

Page 92: Geometria Lineal per al Batxillerat - toomates.net · La història dels vectors. 1.10 Recopilatori d’exercicis. 1.11 Taula resum dels tres productes. 2 Varietats lineals. Rectes

5. Donats el pla d'equació x + 2y + 3z – 1 = 0, la recta r d'equacions

4

32

z y

z x i el punt P = (2, 1, 1), calculeu:

a) Unes equacions de la recta que passa per P i és perpendicular a .

b) L'equació del pla que passa per P i és perpendicular a la recta r.

c) Unes equacions de la recta que passa per P i talla perpendicularment r.

d) Unes equacions de la recta que passa per P, és paral·lela al pla i tal que el seu

vector director és perpendicular al de r.

Solució PAU CAT TEC JUNY 2000 1.6 (Problema)

6. Donat el tetràedre de vèrtexs A = (0, 0, 0), B = (1, 1, 1), C = (3, 0, 0) i D = (0, 3, 0)

a) Calculeu l'equació del pla que conté la cara BCD i la del pla que conté la cara ACD.

b) Calculeu les equacions de dues de les altures del tetràedre, la que passa pel vèrtex A i

la que passa pel vèrtex B, respectivament. (Nota: altura d'un tetràedre és la recta que

passa per un vèrtex i és perpendicular al pla que determina la cara oposada.)

c) Comproveu que les dues altures anteriors es tallen en un punt P.

d) Comproveu si la recta que uneix qualsevol vèrtex del tetràedre amb P és

perpendicular a la cara oposada (i és, per tant, una altura del tetràedre).

Solució PAU CAT TEC JUNY 1999 1.6 (Problema)

Page 93: Geometria Lineal per al Batxillerat - toomates.net · La història dels vectors. 1.10 Recopilatori d’exercicis. 1.11 Taula resum dels tres productes. 2 Varietats lineals. Rectes

5 Distància.

5.1 Distància entre punt i recta.

5.1.1 Definició. Distància entre punt i recta.

La distància d’un punt P a una recta r serà la mínima distància entre el punt P i

qualsevol punt de la recta.

5.1.1 Proposició. Distància entre punt i recta amb el mètode geomètric

Primer Pas: Trobarem la projecció

ortogonal Q de P en r (Vegeu 4.4.1)

Segon Pas: La distància entre el punt i

la recta serà la distància entre P i Q.

5.1.3 Proposició. Fórmula de la distància entre punt i recta.

Distància d’un punt ),,( 321 pppP a la recta ),,(),,(),,(: 321321 vvvkqqqzyxr

v

vPQrPd

),(

Page 94: Geometria Lineal per al Batxillerat - toomates.net · La història dels vectors. 1.10 Recopilatori d’exercicis. 1.11 Taula resum dels tres productes. 2 Varietats lineals. Rectes

Calcula la distància de )1,0,1( P a la recta ),1,2(: r

Solució:

- Pla que passa per P i és perpendicular a r:

El seu vector normal és el vector director de la recta: )1,1,2( n

.

La seva equació és 0320)1(1)1(2 zyxzyx

Intersecció Q de i r:

3

2,

3

1,

3

4

3

204603)1()2(2 Q

- La distància demanada és la distància entre Q i P:

58.03

3

3

21

3

10

3

41),(),(

222

QPdistrPdist

5.1.4 Problemes.

1. Trobeu la distància de l’origen de coordenades a la recta següent:

2

1

zx

yx

Solució: 2 PAU COU

Page 95: Geometria Lineal per al Batxillerat - toomates.net · La història dels vectors. 1.10 Recopilatori d’exercicis. 1.11 Taula resum dels tres productes. 2 Varietats lineals. Rectes

5.2 Distància entre punt i pla.

5.2.1 Definició. Distància entre un punt i un pla.

La distància entre un punt i un pla és la mínima distància que hi ha entre el punt i

qualsevol punt del pla.

5.2.2 Proposició. Mètode geomètric per trobar la distància entre un punt i un pla.

La distancia d’un punt P a un pla és la distancia entre el punt P i la seva projecció

sobre el pla . Per tant:

a) Trobem la projecció ortogonal Q de P

en (Vegeu 4.5)

b) Calculem la distància entre P i Q.

5.2.3 Proposició. Fórmula de la distància d’un punt a un pla.

Distància d’un punt ),,( 321 pppP a un pla 0: dcxbyax

222

321),(

cba

dpcpbpaPd

Atenció!: Observa que l’equació del pla està igualada a zero, és a dir, el valor d està a l’esquerra de la igualtat.

5.2.4 Exercicis.

1. Determina la distància entre el punt P= ( -5 , -1 , 2 ) i el pla 115: zyx

2. Determina la distància entre el punt P= ( -2 , -1 , -3 ) i el pla zyx 53174:

5.2.5 Problemes PAU.

1. Considereu el pla que té com a vectors directors u = (–1, 3, 2) i v = (2, 1, 0) i que

passa pel punt A = (1, 0, 3).

a) Calculeu l’equació de la recta que és perpendicular al pla i passa pel punt A.

b) Calculeu la distància del punt P = (1, 5, 0) al pla.

Nota: Podeu calcular la distància d’un punt de coordenades (x0, y0, z0) al pla d’equació

Ax + By + Cz + D = 0 amb l’expressió 222

000

CBA

DCzAyAx

Solució PAU CAT TEC SET 2018 3.3

Page 96: Geometria Lineal per al Batxillerat - toomates.net · La història dels vectors. 1.10 Recopilatori d’exercicis. 1.11 Taula resum dels tres productes. 2 Varietats lineals. Rectes

2. Siguin el pla d’equació π: x + y – z = 0 i el punt P = (2, 3, 2).

a) Calculeu el punt simètric del punt P respecte del pla π.

b) Calculeu l’equació cartesiana (és a dir, que té la forma Ax + By + Cz = D) dels dos

plans paral·lels a π que estan a una distància 3 del punt P.

Nota: Podeu calcular la distància d’un punt de coordenades (x0, y0, z0) al pla d’equació

Ax + By + Cz + D = 0 amb l’expressió 222

000

CBA

DCzAyAx

.

Solució PAU CAT TEC JUNY 2018 5.2

3. Donat el pla 52: zyx :

a) Calculeu l’equació del pla paral·lel al pla π que passa pel punt P=(1, 0,−1).

b) Determineu també la distància entre el punt P i el pla π.

Solució PAU CAT TEC JUNY 2011 4.2

4. Determineu l’equació del pla perpendicular a la recta

02

01:

zx

yxr que passa pel

punt (1,1,2). Quina distància hi ha d’aquest pla a l’origen de coordenades?

Solució PAU CAT TEC JUNY 2006 3.3

5. Considereu la recta r de l'espai que passa pel punt P = (1, 1, 3) i té per vector director

)1,,1( aav

. Sigui π el pla que té per equació 2x +y –z = 1.

a) Determineu per a cada valor del paràmetre a la posició relativa de la recta r respecte

al pla π (paral·lela, continguda o amb un punt d'intersecció).

b) Hi ha alguna de les rectes r que sigui perpendicular al pla π ?

c) Calculeu la distància que hi ha entre el punt P i el pla π.

Solució PAU CAT TEC SET 2000 6.5 (Problema)

6. a) Sigui P un punt de l’espai, i , un pla. Definiu el concepte de distància del punt P

al pla .

b) Sigui P el punt de coordenades (1,1,0), i , el pla d’equació x+y+z=5. Trobeu la

distància de P a .

Solució PAU CAT TEC JUNY 1998 6.4

Page 97: Geometria Lineal per al Batxillerat - toomates.net · La història dels vectors. 1.10 Recopilatori d’exercicis. 1.11 Taula resum dels tres productes. 2 Varietats lineals. Rectes

5.3 Distància entre dos plans.

5.3.1 Definició. Distància entre dos plans paral·lels.

És la mínima distància possible entre qualsevol punt del primer pla i qualsevol punt del

segon pla.

Si els plans són paral·lels: És la

distància entre un punt qualsevol del

primer pla al segon pla

Si els dos plans no són paral·lels la

distància serà zero.

Calcula la distància entre els següents dos plans:

03:1 zy 0562:2 zy

Els plans són paral·lels perquè els vectors normals són proporcionals. Per tant, la

distància entre tots dos serà la distància entre un punt del primer pla al segon pla.

Prenem )1,3,0( P com punt de 1 .

79.040

5

364

5)1(66),(),( 221

Pdistdist

Page 98: Geometria Lineal per al Batxillerat - toomates.net · La història dels vectors. 1.10 Recopilatori d’exercicis. 1.11 Taula resum dels tres productes. 2 Varietats lineals. Rectes

5.3.3 Determinació dels plans paral·lels a un pla donat a una distància fixa.

Determineu l’equació dels plans paral·lels al pla 012423: zyx que

equidisten 5 unitats de .

Els plans paral·lels a tindran per equació 0423:' Dzyx

Obtenim un punt qualsevol del pla fent 0 yx i obtenim

)3,0,0(301240203: Qzz

2951229512

295122951212295

29

125

29

12

423

12

423

340203)',()',(5

222222

DD

DDD

D

DDDQdistdist

Per tant els dos plans buscats són:

029512423:' zyx i 029512423:' zyx

Exercicis.

1. Trobeu les equacions dels plans paral·lels a 622: zyx situats a 10 unitats de

distància d’aquest.

5.3.4 Problemes PAU.

1. Trobeu les equacions dels plans paral·lels a π: 2x –y +2z = 3 situats a 6 unitats de

distància d’aquest. Solució PAU CAT TEC SET 2007 3.1

2. Trobeu les equacions d’un pla paral·lel al pla d’equació 0822 zyx i que

dista d’aquest sis unitats. N’hi ha més d’un, de pla, que compleixi aquestes condicions?

Solució PAU CAT TEC JUNY 1998 3.2

Page 99: Geometria Lineal per al Batxillerat - toomates.net · La història dels vectors. 1.10 Recopilatori d’exercicis. 1.11 Taula resum dels tres productes. 2 Varietats lineals. Rectes

5.4 Distància entre recta i pla.

5.4.1 Definició. Distància entre una recta i un pla.

És la mínima distància entre qualsevol punt de la recta i qualsevol punt del pla.

Si la recta no és paral·lela al pla, la distància serà zero.

Si la recta és paral·lela al pla, la

distància serà la distància entre

qualsevol punt de la recta al pla, i

aplicarem 5.2.

A IR3, siguin la recta r que té per equació (x, y, z) = (1 + λ, λ, 1 – λ) i el pla π d’equació

-2=z+y-2x .

a) Determineu la posició relativa de la recta r i el pla π.

b) Calculeu la distància entre la recta r i el pla π.

Nota: Podeu calcular la distància d’un punt de coordenades (x0, y0, z0) al pla d’equació

Ax + By + Cz + D = 0 amb l’expressió 222

000

CBA

DCzByAx

Solució PAU CAT TEC JUNY 2016 3.2

Punt base de la recta: )1,0,1(P , vector director de la recta: )1,1,1( v

.

Vector normal del pla: )1,1,2( n

.

a) Posició relativa de la recta i el pla:

nvnv

01121)1()1(121

El punt base de la recta no pertany al pla: -21+0-12

Per tant la recta i el pla són paral·lels.

b) Distància entre la recta i el pla amb el mètode geomètric.

Primer pas: Recta s perpendicular al pla i que passa pel punt base de la recta: Prenem

com a vector director d’aquesta recta el vector normal del pla. L’escrivim en forma

paramètrica perquè serà la que necessitarem després per trobar el seu punt de tall amb el

pla:

kz

ky

kx

s

11

10

21

:

Segon pas: Punt Q d’intersecció entre el pla i la recta perpendicular.

Page 100: Geometria Lineal per al Batxillerat - toomates.net · La història dels vectors. 1.10 Recopilatori d’exercicis. 1.11 Taula resum dels tres productes. 2 Varietats lineals. Rectes

6

5

236-2k1k4k2-2=1k)(1+1k)-(0-2k)2(1

k

k

6

1,

6

5,

3

2

6/1)6/5(11

6/5)6/5(10

3/2)6/5(21

: Q

z

y

x

s

Tercer pas: La distància entre la recta i el pla serà la distància entre P i Q:

04.26

5

6

25

6

5

6

5

3

5

6

5,

6

5,

3

51

6

1,

6

5,1

3

2)1,0,1(

6

1,

6

5,

3

2

),(),(

222

PQ

PQPQ

PQQPdistrdist

b) Distància entre la recta i el pla amb la fórmula:

Abans de tot hem d’igualar l’equació del pla a zero: 0=2z+y-2x-2=z+y-2x

6

5

1)1(2

2110)1(12),(

222

Pdist

5.4.2 Problemes PAU.

1. Siguin r i s les rectes de IR3 que tenen les equacions següents:

2

355:

zyxr i

1

1

3

2

2

3:

zyxs

a) Estudieu el paral·lelisme i la perpendicularitat entre les rectes r i s.

b) Trobeu l’equació general (és a dir, que té la forma Ax+By+Cz=D) del pla π que

conté la recta r i és paral·lel a la recta s. Calculeu la distància entre la recta s i el pla π

obtingut. Solució PAU CAT TEC SET 2014 5.1

2. Considereu la recta azyx

r

1

2

3

1: i el pla π:2x +y −5z =5.

a) Estudieu la posició relativa de la recta r i el pla π en funció del paràmetre a.

b) Quan a=3, calculeu la distància de la recta r al pla π.

Solució PAU CAT TEC SET 2011 2.5

Page 101: Geometria Lineal per al Batxillerat - toomates.net · La història dels vectors. 1.10 Recopilatori d’exercicis. 1.11 Taula resum dels tres productes. 2 Varietats lineals. Rectes

3. Trobeu la distància entre la recta 3

2

3

1

4

3:

zyxr i el pla π: 3x + 4y + 7 = 0.

Solució PAU CAT TEC JUNY 2005 1.3

4. Trobeu la distància entre la recta 3

2

3

1

2

3:

zyxr i el pla π : 2x – 3y + 3z + 5

= 0. Solució PAU CAT TEC SET 2005 3.3

5. Comproveu que la recta que passa pels punts A=(4,0,0) i B=(0,2,2) és paral·lela al pla

d'equació x –3y +5z =2, i calculeu la distància entre la recta i el pla.

Solució PAU CAT TEC SET 2002 1.3

Page 102: Geometria Lineal per al Batxillerat - toomates.net · La història dels vectors. 1.10 Recopilatori d’exercicis. 1.11 Taula resum dels tres productes. 2 Varietats lineals. Rectes

5.5 Distància entre dues rectes.

5.5.1 Definició. Distància entre dues rectes.

La distància entre dues rectes és la mínima distància entre qualsevol punt de la primera

recta i qualsevol punt de la segona. Si les rectes es tallen, la distància serà zero.

5.5.2 Proposició. Mètode 1: Mitjançant la perpendicular comú.

Trobem la recta perpendicular comú a r

i s i els punts de tall P i Q respectius

(Vegeu apartat 4.6).

La distància entre r i s serà la distància

entre P i Q:

),(),( QPdistsrdist

5.5.3 Proposició. Mètode 2: Amb el producte mixt.

Si r és una recta que passa per ),,( 321 pppP i té per vector director ),,( 321 uuuu

i s és una recta que passa per ),,( 321 qqqQ i té per vector director ),,( 321 vvvv

Si u

i v

són proporcionals, és a dir les rectes r i s són paral·leles, la distància entre r i s

serà la distància entre qualsevol punt de la r a s.

Si u

i v

no són proporcionals, la distància entre les dues rectes és la distància entre els

respectius punts de contacte amb la recta perpendicular comú a totes dues, o si fem

servir el producte vectorial, amb la fórmula:

vu

vuPQsrd

),,det(),(

Per tant, dues rectes r i s no paral·leles es tallen en un punt si i només si

0),,det( vuPQ

Page 103: Geometria Lineal per al Batxillerat - toomates.net · La història dels vectors. 1.10 Recopilatori d’exercicis. 1.11 Taula resum dels tres productes. 2 Varietats lineals. Rectes

Trobeu la distància entre les rectes

3

2

1

1

2

4:

zyxr i

2

8

2

21:

zyxs

Les rectes venen donades per:

)3,1,2(

)2,1,4(:

u

Ar i

)2,2,1(

)8,2,1(:

v

Bs

)10,3,3( AB

391812104096

221

312

1033

),,det(

vuAB

3,1,421

12,

21

32,

22

31

221

312

kji

vu

263)1(4 222 vu

2

263

26

39

26

39),,det(),(

vu

vuABsrd

5.5.4 Problemes PAU.

1. Considereu les rectes

21

1

2

2:

zyxr i

tz

ty

tx

5

41

31

a) Estudieu la seva posició relativa.

b) Trobeu l’equació del pla que conté s i és paral·lel a r.

c) Calculeu la distància entre r i s. Solució PAU CAT TEC SET 2004 5.6 (Problema)

2. Considereu les dues rectes r1 i r2 de l’espai donades per les equacions següents:

1,23

2:1

yz

xr )1,1,1()0,0,1(),,(:2 tzyxr .

Calculeu la distància entre elles. PAU COU

Page 104: Geometria Lineal per al Batxillerat - toomates.net · La història dels vectors. 1.10 Recopilatori d’exercicis. 1.11 Taula resum dels tres productes. 2 Varietats lineals. Rectes

5.6 Problemes PAU amb distància, projecció ortogonal i punt simètric.

1. Siguin P= (3 – 2a, b, –4), Q= (a–1, 2 +b, 0) i R= (3, –2, –2) tres punts de l’espai IR3.

a) Calculeu el valor dels paràmetres a i b per als quals aquests tres punts estiguin

alineats.

b) Trobeu l’equació contínua de la recta que els conté quan estan alineats.

c) Quan b=0, trobeu els valors del paràmetre a perquè la distància entre els punts P i Q

sigui la mateixa que la distància entre els punts P i R.

d) Si b= 0, calculeu el valor del paràmetre a perquè els punts P, Q i R determinin un

triangle equilàter. Solució PAU CAT TEC JUNY 2009 3.6 (Problema)

2. Donats el punt P= (7, 5, 1), el pla π: x –2y –3z =10 i la recta

526

7223:

zyx

zyxr

a) Trobeu la distància del punt P al pla π.

b) Trobeu la distància del punt P a la recta r.

c) Trobeu la distància de la recta r al pla π. Solució PAU CAT TEC SET 2008 4.4

3. Trobeu les coordenades dels punts situats sobre la recta d’equació

(x,y,z) = (–1,1,1) +t·(1,2,1) que estan a distància 1 del pla 2x +2y +z = 5. Solució PAU CAT TEC JUNY 2006 1.1

4. Donats els punts A = (1, 0, 0) i B (0, 0, 1):

a) Trobeu un punt C sobre la recta d’equació paramètrica

1

1

1

z

y

x

que faci que el

triangle ABC sigui rectangle en C.

b) Trobeu l’àrea del triangle ABC. Solució PAU CAT TEC SET 2005 3.4

5. Sigui π el pla d'equació x –y +2z = 3 i P el punt (1, 1, 0).

a) Calculeu la distància d de P a π .

b) Determineu l'equació de l'altre pla π' paral·lel a π que també dista d del punt P.

c) Determineu l'equació de la recta r perpendicular a π que passa per P.

d) Calculeu la intersecció de la recta r amb el pla π. Solució PAU CAT TEC SET 2001 4.6 (Problema)

7. Considereu a l'espai la recta r d'equacions 1

1

3

3

2

2

zyx i la recta s

d’equacions 1

4

3

1

2

4

zyx

a) Determineu el punt de tall de la recta r amb el pla z = 0.

b) Comproveu que les rectes r i s són paral·leles i calculeu la distància entre elles.

c) Quina és l'equació del pla que conté les dues rectes?

d) Calculeu la distància del pla anterior a l'origen de coordenades. Solució PAU CAT TEC JUNY 2001 5.5

Page 105: Geometria Lineal per al Batxillerat - toomates.net · La història dels vectors. 1.10 Recopilatori d’exercicis. 1.11 Taula resum dels tres productes. 2 Varietats lineals. Rectes

6 Posició relativa.

6.1 Posició relativa entre recta i pla.

6.1.1 Posició relativa entre recta i pla mitjançant l’estudi del sistema.

Sigui

2222

1111:

dzcybxa

dzcybxar una recta i 3333: dzcybxa un pla a l’espai.

Considerem el sistema

3333

2222

1111

dzcybxa

dzcybxa

dzcybxa

333

222

111

cba

cba

cba

M

3333

2222

1111

'

dcba

dcba

dcba

M

1. SCD: Rang M = 3. Es tallen en un punt (que és la solució del sistema)

*#00

**#0

***#

Sistema Compatible

Determinat

SCD

2. SI: Rang M = 2 i Rang M’ = 3. La recta és paral·lela i externa al pla.

#000

**#0

***# Sistema Incompatible

SI

3. SCI: Rang M = 2 i Rang M’ = 2 La recta està continguda al pla.

0000

**#0

***# Sistema Compatible

Indeterminat

SCI

Page 106: Geometria Lineal per al Batxillerat - toomates.net · La història dels vectors. 1.10 Recopilatori d’exercicis. 1.11 Taula resum dels tres productes. 2 Varietats lineals. Rectes

6.1.2 Posició relativa entre recta i pla mitjançant equacions vectorials.

Pla: DCzByAx : : vector normal ),,( CBAn

Recta r: Punt base ),,( 321 pppP i vector director ),,( 321 uuuv

vn

?

SI

NO: Recta i pla es tallen en un punt

El punt P pertany a ?

SI

NO: Recta paral·lela externa al pla.

Recta continguda al pla

Page 107: Geometria Lineal per al Batxillerat - toomates.net · La història dels vectors. 1.10 Recopilatori d’exercicis. 1.11 Taula resum dels tres productes. 2 Varietats lineals. Rectes

6.1.3 Exercicis.

1. Determineu la posició relativa del pla 323: zyx i la recta

323

1:

z

yxr

2. Determineu la posició relativa entre la recta i el pla per als diferents valors del

paràmetre k:

22

12:

zyx

zyxr 42: zyxk

3. Determineu la posició relativa entre la recta i el pla per als diferents valors del

paràmetre t:

1

1:

zytx

zyxtr tztyx :

6.1.4 Problemes PAU.

1. Considereu el pla π: x + y + z = 1 i la recta r que passa pels punts P = (0, 0, 6) i Q =

(1, 2, 3).

a) Estudieu la posició relativa de la recta r i el pla π.

b) Calculeu la distància entre la recta r i el pla π.

Nota: Podeu calcular la distància d’un punt de coordenades (x0, y0, z0) al pla d’equació

Ax + By + Cz + D = 0 amb l’expressió 222

000

CBA

DCzByAx

.

Solució PAU CAT TEC SET 2017 2.1

2. Donats el pla π: x +2y –z = 3 i la recta 42

1:

mzy

xr

,

a) Comproveu que el vector característic (o normal) de π i el vector director de r són

perpendiculars.

b) Estudieu la posició relativa de π i r en funció del paràmetre m.

Solució PAU CAT TEC SET 2013 1.3

3. Donats el pla π: 5x +y +3z = 4 i la recta

32

2:

zy

yxar , estudieu-ne la posició

relativa en funció del paràmetre a.

Solució PAU CAT TEC SET 2010 2.2

Page 108: Geometria Lineal per al Batxillerat - toomates.net · La història dels vectors. 1.10 Recopilatori d’exercicis. 1.11 Taula resum dels tres productes. 2 Varietats lineals. Rectes

4. Sigui el pla de l’espai que passa pel punt (0,0,3) i que conté els vectors

)5,2,1( u

i )3,1,2( v

. Sigui r la recta d’equacions:

02

04

zyx

zyx

a) Escriviu l’equació cartesiana del pla (equació de la forma ax+by+cz=d).

b) Estudieu la posició relativa de r respecte a (heu de dir si r és paral·lela a , si està

continguda en o bé si talla ). PAU CAT TEC JUNY 1998 6.3

5. Estudieu, segons els diferents valors que pot tenir el paràmetre m, les posicions

relatives del pla i de la recta r que es donen a continuació:

123: zymx

12

13:

zmyx

yxr

PAU CAT TEC JUNY 1997 3A.4

Page 109: Geometria Lineal per al Batxillerat - toomates.net · La història dels vectors. 1.10 Recopilatori d’exercicis. 1.11 Taula resum dels tres productes. 2 Varietats lineals. Rectes

6.2 Posició relativa entre dos plans.

6.2.1 Posició relativa entre dos plans mitjançant l’estudi de sistema.

Un pla en l’espai queda determinat algèbricament amb una equació lineal de tres

variables:

dczbyax :

Per tant, l’estudi de la posició relativa de dos plans equival a discutir el sistema format

per les seves equacions:

22222

11111

:

:

dzcybxa

dzcybxa

o en forma matricial :

2222

1111

dcba

dcba

1. SCI : Rang M = Rang Ma = 2. Els dos plans es tallen en una recta.

**#0

***#

2. SI : Rang M = 1, Rang Ma = 2. Els plans són paral·lels i no coincidents.

#000

***#

3. SI : Rang M = Rang Ma = 1. Els plans són coincidents.

0000

***#

Page 110: Geometria Lineal per al Batxillerat - toomates.net · La història dels vectors. 1.10 Recopilatori d’exercicis. 1.11 Taula resum dels tres productes. 2 Varietats lineals. Rectes

6.2.2 Posició relativa entre dos plans mitjançant equacions vectorials.

Pla 11111 : DzCyBxA : vector normal ),,( 111 CBAn

Pla 22222 : DzCyBxA : vector normal ),,( 222 CBAm

mkn

?

SI

NO: Els dos plans es tallen en una recta

),,,(),,,( 11111111 dcbakdcba ?

(és a dir, mirem si són equacions

equivalents)

SI

NO: Dos plans paral·lels i no

coincidents

Els dos plans són coincidents

Page 111: Geometria Lineal per al Batxillerat - toomates.net · La història dels vectors. 1.10 Recopilatori d’exercicis. 1.11 Taula resum dels tres productes. 2 Varietats lineals. Rectes

6.2.3 Exercicis.

1. Estudia la posició relativa dels dos plans. En el cas que es tallin en una recta,

determina la seva equació vectorial.

a)

1153

45

zyx

zyx b)

1)5(3

45

xyz

zyx c)

yxz

zyx

45

045

2. Determineu m i n de forma què els plans 0946:1 zymx i

039:2 nznyx siguin paral·lels.

6.2.4 Problemes.

1. Considereu els plans d'equacions:

π1: x + 2y – z = 3 i π2 : ax + (a – 2)y + 2z = 4.

a) Hi ha algun valor del paràmetre a per al qual la intersecció dels plans π1 i π2 no és una

recta?

b) Calculeu un vector director de la recta que s'obté quan es fa la intersecció de π1 i π2

per al valor del paràmetre a=0.

Solució PAU CAT TEC JUNY 2002 3.3

2. Discutiu la posició relativa dels plans

0:1 azyax i 1)/1()3(:2 zyaxa

segons els valors de a ( 0a ).

PAU COU

Page 112: Geometria Lineal per al Batxillerat - toomates.net · La història dels vectors. 1.10 Recopilatori d’exercicis. 1.11 Taula resum dels tres productes. 2 Varietats lineals. Rectes

6.3 Posició relativa entre dues rectes.

6.3.1 Posició relativa entre dues rectes mitjançant l’estudi del sistema.

2222

1111:

dzcybxa

dzcybxar

4444

3333:

dzcybxa

dzcybxas

Considerem el sistema:

4444

3333

2222

1111

dzcybxa

dzcybxa

dzcybxa

dzcybxa

444

333

222

111

cba

cba

cba

cba

M

4444

3333

2222

1111

'

dcba

dcba

dcba

dcba

M

1. SCD. Rang M = 3, Rang M’ = 3. Les dues rectes es tallen en un únic punt.

2. SCI. Rang M = 2, Rang M’ = 2. Les dues rectes són coincidents.

3. SI. Rang M = 2, Rang M’ = 3. Les dues rectes són paral·leles i no coincidents.

4. SI. Rang M = 3, Rang M’ = 4. Les dues rectes es creuen.

Page 113: Geometria Lineal per al Batxillerat - toomates.net · La història dels vectors. 1.10 Recopilatori d’exercicis. 1.11 Taula resum dels tres productes. 2 Varietats lineals. Rectes

6.3.2 Posició relativa entre dues rectes mitjançant equacions vectorials.

Recta r: Punt base ),,( 321 pppP i vector director ),,( 321 uuuu

Recta s: Punt base ),,( 321 qqqQ i vector director ),,( 321 vvvv

0),,det( PQvu

?

SI

NO: Les dues rectes es creuen.

vku

?

SI

NO: Les dues rectes es tallen en un únic

punt.

El punt P pertany

a la recta r ?

SI

NO: Les dues rectes són paral·leles i no

coincidents.

Les dues rectes són coincidents.

Page 114: Geometria Lineal per al Batxillerat - toomates.net · La història dels vectors. 1.10 Recopilatori d’exercicis. 1.11 Taula resum dels tres productes. 2 Varietats lineals. Rectes

6.3.3 Exercicis.

1. Determineu la posició relativa de les rectes

2

92:

y

zxr i

022

1:

zyx

yxs

2. Determineu la posició relativa de les rectes

32

12:

yx

zyxr i

025

523:

zyx

yxs

3. Determineu la posició relativa de les rectes

3

3

2

1:

zy

xr i

43:

zyxs

4. Estudieu la posició relativa de les rectes, en funció del paràmetre t:

1

1:

zytx

zyxtr i

tzyx

ztyxs

1:

Page 115: Geometria Lineal per al Batxillerat - toomates.net · La història dels vectors. 1.10 Recopilatori d’exercicis. 1.11 Taula resum dels tres productes. 2 Varietats lineals. Rectes

Posició relativa de dues rectes que depenen d’un paràmetre. Exemple resolt.

Estudieu la posició relativa de les rectes

0

0:

zya

yxr

azy

ayxs

1:

en funció del paràmetre a PAU COU

Vector director de r: ),1,1(

10

011 a

a

kji

v

Punt base de r: (Prenent 0y ) )0,0,0( P

Vector director de s: )1,1,(

110

01 aa

kji

w

Punt base de s: (Prenent 0y ) ),0,1( aQ

),0,1( aPQPQ

10

1

011

11

a

aa

a

Si 1a Les dues rectes es creuen.

Si

)1,1,1(

)1,1,1(1

w

va

Els vectors directors són proporcionals, per tant són

paral·leles. I com que )0,0,0(P no pertany a la recta s:

100

100:

as

Les dues rectes són paral·leles i no coincidents.

Page 116: Geometria Lineal per al Batxillerat - toomates.net · La història dels vectors. 1.10 Recopilatori d’exercicis. 1.11 Taula resum dels tres productes. 2 Varietats lineals. Rectes

6.3.4 Problemes PAU.

1. Siguin r i s dues rectes de l’espai les equacions respectives de les quals, que depenen

d’un paràmetre real b, són les següents:

152

13:

zyx

zyxbr

1

1

11:

z

b

byxs

a) Trobeu el punt de tall de la recta r amb el pla d’equació x= 0 i el punt de tall de la

recta s amb aquest mateix pla.

b) Calculeu un vector director per a cada una de les dues rectes.

c) Estudieu la posició relativa de les dues rectes en funció del paràmetre b.

Solució PAU CAT TEC JUNY 2009 4.6 (Problema)

2. Les rectes 4

1

12:1

zyaxr i

1

4

21

2:2

zbyxr són coplanàries (és a dir,

estan incloses en un mateix pla).

a) Expliqueu, raonadament, quina és la posició relativa d’aquestes rectes.

b) Trobeu la relació que hi ha entre els paràmetres a i b.

c) Trobeu els valors de a i b si el pla que les conté passa pel punt P= (2, 4, 6).

Solució PAU CAT TEC SET 2008 4.6 (Problema)

3. Estudieu la posició relativa de les dues rectes r i s de l'espai donades per les

equacions següents:

1

92:

y

zxr

532:

xzy

yxs

PAU CAT TEC JUNY 1997 2B.3

Page 117: Geometria Lineal per al Batxillerat - toomates.net · La història dels vectors. 1.10 Recopilatori d’exercicis. 1.11 Taula resum dels tres productes. 2 Varietats lineals. Rectes

6.3.5 Problemes PAU COU.

1. Considereu la recta r de 3IR donada per

1

2

yz

ayx i la recta r’ que passa pels punts

)1,0,1( i )4,1,3( . Calculeu el valor d’a sabent que r i r’ es tallen. Calculeu també el punt

d’intersecció de r i r’. PAU COU

2. Considereu la recta de 3IR donada per 2

11

zyx i la recta donada per

422

4

zy

x. Digueu si aquestes dues rectes es tallen en un punt, són paral·leles

o bé es creuen (dues rectes de l’espai es creuen quan no es tallen ni són paral·leles).

PAU COU

3. Considereu, a 3IR , les dues rectes següents :

4

12

zx

zyx

0

0

yx

zax

on a és un nombre real. Digueu si existeix algun valor de a per al qual les rectes es

tallin.

PAU COU

4. Donades les rectes

zy

k

xr

1:1

1

2:2

zx

ykxr

determineu el valor de k per tal que les rectes siguin paral·leles. PAU COU

5. Donades les rectes d’equacions

332

42

yx

azay

33

233

yax

aazy

busqueu els valors de a pels quals les dues rectes es tallen.

PAU COU

Page 118: Geometria Lineal per al Batxillerat - toomates.net · La història dels vectors. 1.10 Recopilatori d’exercicis. 1.11 Taula resum dels tres productes. 2 Varietats lineals. Rectes

6.4 Posició relativa de tres plans.

6.4.1 Posició relativa de tres plans mitjançant l’estudi del sistema.

L’estudi de la posició relativa de tres plans equival a discutir el sistema format per les

seves equacions:

33333

22222

11111

:

:

:

dzcybxa

dzcybxa

dzcybxa

o en forma matricial :

3333

2222

1111

dcba

dcba

dcba

1. SCD: Rang M = Rang Ma = 3. Els tres plans es tallen en aquest punt formant un

tríedre.

*#00

**#0

***#

2. SCI: Dos casos:

2.1 SCI i Rang M = Rang Ma = 2 Els tres plans es tallen en una recta.

0000

**#0

***# (Una única fila de zeros)

2.2 SCI i Rang M = Rang Ma = 1. Tots tres plans són coincidents.

0000

0000

***# (Dues files de zeros)

Page 119: Geometria Lineal per al Batxillerat - toomates.net · La història dels vectors. 1.10 Recopilatori d’exercicis. 1.11 Taula resum dels tres productes. 2 Varietats lineals. Rectes

3. SI: Els tres plans no tenen cap punt en comú. Hem d’estudiant el paral·lelisme entre

els plans:

3.1 Els tres plans són paral·lels:

3.1.1 Dues equacions són proporcionals : Dos plans coincidents i paral·lels (però no

coincidents) al tercer.

3.1.2 No hi ha equacions proporcionals : Tres plans paral·lels i no coincidents.

3.2 Una parella de plans paral·lels :

3.3 Cap parella de plans paral·lels: Els plans es tallen dos a dos amb rectes paral·leles.

Tenim el que col·loquialment s’anomena « tenda de campanya »

Page 120: Geometria Lineal per al Batxillerat - toomates.net · La història dels vectors. 1.10 Recopilatori d’exercicis. 1.11 Taula resum dels tres productes. 2 Varietats lineals. Rectes

6.4.2 Exercicis resolts.

Determina la posició relativa dels tres plans

1:1 zyx

05:2 zyx

13:3 zyx

El sistema d’equacions dels punts d’intersecció és:

13

05

1

zyx

zyx

zyx

, en forma matricial:

1113

0115

1111

'A

016

113

115

111

AA

Per tant, el sistema és compatible determinat, i.e. els tres plans es tallen en un únic punt.

Determina la posició relativa dels tres plans

32:1 zyx

12:2 zyx

233:3 zy

El sistema d’equacions dels punts d’intersecció és:

233

12

32

zy

zyx

zyx

, en forma matricial:

2330

1211

3112

'A

0

330

211

112

AA

Agafant les 2 primeres files i columnes, obtenim un determinant no nul:

31211

12

, per tant, Rang(A) = 2

Page 121: Geometria Lineal per al Batxillerat - toomates.net · La història dels vectors. 1.10 Recopilatori d’exercicis. 1.11 Taula resum dels tres productes. 2 Varietats lineals. Rectes

Rang(A’) = 3, només cal substituir la primera columna per la quarta :

09

332

211

113

Per tant, el sistema és incompatible. Els tres plans no tenen punts en comú.

1 i 2 es tallen en una recta, ja que el sistema associat

12

32

zyx

zyxés compatible:

La matriu associada

1211

3112té rang 3, només cal agafar les dues primeres

columnes : 31211

12

De la mateixa manera comprovem que també 1 i 3 es tallen en una recta, i 2 i 3 es

tallen en una recta, per tant, estem davant d’un cas de tres plans formant un prisma

triangular.

6.4.3 Exercicis.

1. Estudieu la posició relativa dels plans següents, en funció del paràmetre t:

1:1 ztytx

tztyxt :2

1:3 zytxt

6.4.4 Problemes PAU.

1. Considereu la matriu

134

11

101

a

a

a

A , en què a és un paràmetre real.

a) Trobeu els valors del paràmetre a per als quals la matriu és invertible.

b) Discutiu la posició relativa dels plans π1: x + (a – 1)z = 0, π2: x + ay + z = 1 i

π3: 4x + 3ay + z = 3 en funció dels valors del paràmetre a.

PAU CAT TEC SET 2019 5.2 (Solució: "Compendium Tec", Pàg. 543)

2. Siguin els plans de IR3 π1: –y +z = 2, π2: –2x + y + z = 1 i π3: 2x – 2z = –1.

a) Calculeu la posició relativa dels tres plans.

b) Comproveu que el pla π3 és paral·lel a la recta definida per la intersecció dels plans

π1 i π2.

Solució PAU CAT TEC SET 2015 5.4

Page 122: Geometria Lineal per al Batxillerat - toomates.net · La història dels vectors. 1.10 Recopilatori d’exercicis. 1.11 Taula resum dels tres productes. 2 Varietats lineals. Rectes

3. Digueu per a quin valor del paràmetre m els plans

1:1 mzyx , mzyx :2 i 32:3 zmy

tenen com a intersecció una recta. Solució PAU CAT TEC JUNY 2012 3.1

4. A l’espai es consideren els tres plans d’equacions:

1z2yx:1 , 1pzypx:2 i 12zypx:3 , on p és un paràmetre real.

a) Esbrineu per a quins valors de p els tres plans es tallen en un únic punt. Trobeu

aquest punt quan p=1.

b) Hi ha algun valor de p que faci que la intersecció comuna sigui una recta? Si és així,

escriviu l’equació vectorial d’aquesta recta.

c) Trobeu quina és la posició relativa dels tres plans quan p=1/2

Solució PAU CAT TEC JUNY 2007 2.6 (Problema)

5. Discutiu el sistema següent en funció del paràmetre p.

1236

1022

52

zypx

zypx

zyx

Doneu la interpretació geomètrica del sistema en cada cas i resoleu-lo quan sigui

compatible. Solució PAU CAT TEC JUNY 2007 1.6

6. Donada la matriu següent dependent d’un paràmetre m:

mm

mmA

22

22

211

a) Estudieu-ne el rang segons els valors de m.

b) Digueu quina és la posició relativa dels plans π1:x +y +2z = 2, π2:2x +my +2mz

=2+m i π3: mx +2y +(2+m)z= 0, segons els valors de m.

Solució PAU CAT TEC SET 2007 3.2

7. Donat el sistema

1)22()22(

12

2

mzmxm

zyx

zy

on m és un paràmetre:

a) discutiu el sistema segons els valors de m;

b) resoleu els casos compatibles;

c) en cada un dels casos de la discussió de l’apartat a), feu una interpretació

geomètrica del sistema. Solució PAU CAT TEC JUNY 2004 3.5

Page 123: Geometria Lineal per al Batxillerat - toomates.net · La història dels vectors. 1.10 Recopilatori d’exercicis. 1.11 Taula resum dels tres productes. 2 Varietats lineals. Rectes

8. Considerem els punts de l’espai A(1, 1, 0), B(0, 1, 2) i C(–1, 2, 1). Ens diuen que

aquests tres punts formen part del conjunt de solucions d’un sistema de tres equacions

lineals amb tres incògnites. Es demana:

a) aquests punts, estan alineats?

b) podem saber el rang de la matriu del sistema d’equacions?

Solució PAU CAT TEC JUNY 2004 3.4

9. Se sap que el sistema d'equacions

5102

182

2

zyx

zyx

azyx

té més d'una solució.

Calculeu a i digueu quina és la interpretació geomètrica que té el conjunt de

totes les solucions d'aquest sistema. Solució PAU CAT TEC JUNY 2000 3.3

10. Discutiu, segons els valors del paràmetre a, el sistema següent:

2

1

4

azyx

zayx

zyax

Doneu en cada cas la interpretació geomètrica.

PAU COU

11. Estudieu en funció del paràmetre a el sistema següent:

13554

5

92

zayx

azay

yax

Doneu en cada cas la interpretació geomètrica.

PAU COU

Page 124: Geometria Lineal per al Batxillerat - toomates.net · La història dels vectors. 1.10 Recopilatori d’exercicis. 1.11 Taula resum dels tres productes. 2 Varietats lineals. Rectes

7 Angles.

7.1 Angle entre dues rectes.

7.1.1 Definició. Angle entre dues rectes.

És l’angle que formen els seus vectors directors, amb valor absolut:

vu

vu

cos

7.1.2 Exercicis.

1. Determina l’angle entre les dues rectes (en graus).

2

361:

zyxr

2

11

3

4:

zy

xs

2. Determina l’angle entre les dues rectes (en graus).

2

82

2

7:

zy

xr

z

yxs

3

5

9

3:

7.1.3 Determinació d’una recta coneixent l’angle. Problemes resolts.

Considereu la recta r de R3 d’equacions

1y

zx i la recta r’ d’equacions

a

zy

x 11

2

1

on a és una constant. Aquestes dues rectes es tallen en el punt (1,1,1) ja que aquest punt

compleix les equacions de les dues rectes. Determineu els valors de a que fan que r i r’

es tallin en aquest punt formant un angle de 30 graus.

PAU CAT COU

Passem les equacions de r a forma paramètrica per a extreure un vector director:

0

1

0

1

0

1

01

10

01

1z

z

y

x

zz

zy

zx

y

zx

Un vector director de r serà doncs )1,0,1(v

Un vector director de r’ es pot determinar agafant els denominadors de la seva equació

general: ),1,2( aw

Page 125: Geometria Lineal per al Batxillerat - toomates.net · La història dels vectors. 1.10 Recopilatori d’exercicis. 1.11 Taula resum dels tres productes. 2 Varietats lineals. Rectes

Així doncs, hem de resoldre l’equació wv

wv

)30cos(

512

2101

211021

2

3)30cos(

2222

222

aaw

v

aawv

2256

2252352

2

2

3)30cos(

2

2

2

aa

aaa

a

wv

wv

Elevant al quadrat tots dos costats

1

77801538820

882153)44(253

)2(253)2(456

)2(42256

222

2222

2222

2222

aaaaaa

aaaaaa

aaaa

aaa

Efectivament, per a a=7 )7,1,2(w

i

)30cos(2

3

32

33

32

3

332

9

322

9

542

9

3

wv

wv

i per a a=1 , )1,1,2(w

i

)30cos(2

3

32

33

32

3

32

3

wv

wv

Page 126: Geometria Lineal per al Batxillerat - toomates.net · La història dels vectors. 1.10 Recopilatori d’exercicis. 1.11 Taula resum dels tres productes. 2 Varietats lineals. Rectes

Considereu la recta r de l’espai donada per les equacions

0

0)1(

zx

zyxa

on a és un paràmetre, i la recta r’ que té com a vector director 1,2,1 i que passa per

l’origen formant un angle de 45 graus. Determina els possibles valors del paràmetre a.

PAU CAT COU

Passem les equacions de r a forma paramètrica per a extreure un vector director:

yazyzazzyzza

yzxazyxa

zxzx

))(1(

)1(0)1(

0

0

0

0

1

1

1

za

z

y

x

z

azy

zx

Per tant un vector director de la recta és )1,,1( a o equivalentment )1,,1( av

.

Un vector director de la recta r’ ja ens ve donat directament: 1,2,1 w

Així doncs, hem de resoldre l’equació wv

wv

)45cos(

24121)1()2(1

2)1(1

22121)1)(1(211

2

1)45cos(

222

2222

w

aav

aaawv

2222222

22

2

1)45cos( 2

2

aa

a

a

wv

wv

Elevem al quadrat tots dos costats de l’equació:

028

00280848284

8284844242284

2222422222

22

2222

22

222

aaaaa

aaaaaa

aaaa

Efectivament, per a a=0, )1,0,1( v

i )45cos(2

1

22

2

22

2

wv

wv

Page 127: Geometria Lineal per al Batxillerat - toomates.net · La història dels vectors. 1.10 Recopilatori d’exercicis. 1.11 Taula resum dels tres productes. 2 Varietats lineals. Rectes

7.1.4 Problemes PAU COU.

1. En un sistema de coordenades rectangulars tenim la família de rectes r:

3

2

2

13

zy

a

x i la recta

1

1:

zyx

zyxs

a) Determineu les dues rectes de la família r que fan un angle de 60º amb la recta s.

b) Determineu l’angle d’aquestes dues rectes de la família r.

PAU COU

2. Considereu la recta r donada per

02

0224

yx

zyxi la recta r’ de vector director

)1,,1( a que passa per l’origen. Determineu per a quins valors de a, r i r’ es tallen a

l’origen formant un angle de 45 graus. PAU COU

3. Considereu a l’espai 3 les dues rectes següents: )1,,1()2,2,2(),,( azyx i

02

0

zx

yx, on a és un nombre real. Comproveu que aquestes dues rectes es tallen per

a qualsevol valor de a i determineu a perquè formin un angle de 60 graus. PAU COU

4. En un sistema de coordenades rectangulars tenim la recta 3

2

2

13:

zy

a

xr i la

recta

1

1:

zyx

zyxs . Determineu els dos valors d’a per als què les dues rectes fan un

angle de 60º.

PAU COU

Page 128: Geometria Lineal per al Batxillerat - toomates.net · La història dels vectors. 1.10 Recopilatori d’exercicis. 1.11 Taula resum dels tres productes. 2 Varietats lineals. Rectes

7.2 Angle entre dos plans.

7.2.1 Definició. Angle entre dos plans.

L’angle que formen els plans DCzByAx :1 i '''':2 DzCyBxA és

l’angle que formen els seus vectors ortogonals )',','(),,( 21 CBAniCBAn

21

21cos

nn

nn

7.2.2 Exercicis.

1. Determina l’angle entre els dos plans (en graus)

187118:1 zyx 8625:2 zyx

2. Determina l’angle entre els dos plans (en graus)

6203:1 zyx 414132:2 zyx

7.2.3 Problemes PAU.

1. Considereu els plans π1: 5x – y –7z = 1 i π2: 2x + 3y + z = 5.

a) Determineu l’equació general (és a dir, la que té la forma Ax + By + Cz = D) del pla

que passa per l’origen de coordenades i és perpendicular als plans π1 i π2.

b) Calculeu l’angle que formen els plans π1 i π2.

Solució PAU CAT TEC JUNY 2017 1.2

Page 129: Geometria Lineal per al Batxillerat - toomates.net · La història dels vectors. 1.10 Recopilatori d’exercicis. 1.11 Taula resum dels tres productes. 2 Varietats lineals. Rectes

7.3 Angle entre recta i pla.

7.3.1 Definició. Angle entre recta i pla.

L’angle que formen una recta i un pla és el complementari de l’angle que formen un

vector normal al pla i un vector director de la recta:

un

un

cos

º90

7.3.2 Problemes PAU.

1. Calculeu l’equació general (és a dir, de la forma Ax+By+Cz+D=0) dels plans que

contenen la recta

1

2:

z

yr i que formen un angle de 45° amb el pla z=0.

Solució PAU CAT TEC JUNY 2011 4.5

2. Considereu la recta

023

0352:

zyx

zyxr i el pla π: 2x –y +az +2 = 0 on a és un

paràmetre.

a) Trobeu un vector director de la recta i un vector perpendicular al pla.

b) Quin ha de ser el valor de a per tal que la recta i el pla siguin paral·lels?

c) Esbrineu si existeixen valors de a per als quals la recta i el pla siguin perpendiculars.

En cas afirmatiu, calculeu-los.

d) Esbrineu si existeixen valors de a per als quals la recta i el pla formin un angle de

30º. En cas afirmatiu, calculeu-los. Solució PAU CAT TEC JUNY 2006 3.6

3. Una recta r passa pel punt A=(3,0,2) i té la direcció del vector (–1,1,4).

a) Trobeu quin angle forma r amb el pla horitzontal.

b) Comproveu que no passa pel punt B=(1,3,10).

c) Trobeu l’equació de la recta que passa per A i B.

Solució PAU CAT TEC JUNY 2006 1.5 (Problema)

4. Calculeu l’angle que forma el pla 12 zyx amb la recta determinada per les

equacions

2

1

z

ty

tx

Solució PAU CAT TEC JUNY 2002 2.4

Page 130: Geometria Lineal per al Batxillerat - toomates.net · La història dels vectors. 1.10 Recopilatori d’exercicis. 1.11 Taula resum dels tres productes. 2 Varietats lineals. Rectes

5. Considereu la recta

023

0352

zyx

zyx

i el pla 022 zayx , on a és un paràmetre.

a) Per a quin valor de a la recta i el pla són paral·lels? Quina serà llavors la distància

entre el punt P=(1,0,-1) de la recta i el pla?

b) Existeix algun valor de a per al qual la recta i el pla siguin perpendiculars?

c) Determineu el valor de a perquè la recta i el pla formin un angle de 30º.

Solució PAU CAT TEC SET 2000 2.6

6. Considereu la recta de 3 donada per

1

2

zx

zyx i el pla d’equació 3 yx .

Digueu quin angle formen aquesta recta amb aquest pla.

Solució: 30º PAU COU

7. Trobeu els valors de a que fan que el vector perpendicular al pla 0z i la recta de

l’espai d’equacions

5223

52

zayx

zayx formin un angle de 30 graus.

Solució: 3,3 PAU COU

Page 131: Geometria Lineal per al Batxillerat - toomates.net · La història dels vectors. 1.10 Recopilatori d’exercicis. 1.11 Taula resum dels tres productes. 2 Varietats lineals. Rectes

8 Àrea i volum.

8.1 Àrees de figures en l’espai.

8.1.1 Exercicis.

1. Determina l’àrea del paral·lelogram definit pels vectors

v

= ( -2 , -1 , 2 ) i w

= ( -1 , 0 , 4 )

2. Determina l’àrea del paral·lelogram definit pels vectors

v

= ( 3 , -4 , 1 ) i w

= ( 5 , 0 , 2 )

8.1.2 Problemes PAU.

1. Siguin P, Q i R els punts d’intersecció del pla d’equació x + 4y + 2z = 4 amb els tres

eixos de coordenades OX, OY i OZ, respectivament.

a) Calculeu els punts P, Q i R, i el perímetre del triangle de vèrtexs P, Q i R.

b) Calculeu l’àrea del triangle de vèrtexs P, Q i R.

Nota: Per a calcular l’àrea del triangle definit pels vectors v i w podeu fer servir

l’expressió wvS 2

1 , en què wv és el producte vectorial dels vectors v i w.

PAU CAT TEC SET 2019 5.5 (Solució: "Compendium Tec", Pàg. 548)

2. Un triangle d’àrea 3/2 té dos dels vèrtexs als punts P=(0, 0, 0) i Q=(2, 0, 1). El tercer

vèrtex, R, és un punt de la recta

1

0:

y

zyxr

i té la primera coordenada no nul·la. Calculeu les coordenades del vèrtex R.

Solució PAU CAT TEC JUNY 2013 3.4

3. Donats els punts de l'espai A=(2, 1, 0), B=(0, 2, 0), C=(–3, 0, 0) i D=(0, –1, 0)

a) Són coplanaris? Formen un paral·lelogram?

b) Calculeu l'àrea del polígon ABCD.

c) Calculeu el punt simètric del punt E= (1, 1, 2) respecte del pla que determinen A, B i

C.

d) Calculeu la distància entre la recta que passa per E i A i la recta que passa per B i C.

PAU CAT TEC SET 1999 2.6 (Problema)

Page 132: Geometria Lineal per al Batxillerat - toomates.net · La història dels vectors. 1.10 Recopilatori d’exercicis. 1.11 Taula resum dels tres productes. 2 Varietats lineals. Rectes

8.2 Volum de figures en l’espai.

8.2.1 Exercicis.

1. Determina el volum del paral·lelepípede generat pels vectors .w i v ,u

u

= ( 1 , -6 , -2 ), v

= ( 3 , -4 , -3 ), w

= ( -5 , -2 , 2 )

2. Determina el volum del paral·lelepípede generat pels vectors .w i v ,u

u

= ( 3 , 1 , -1 ), v

= ( -1 , -2 , 3 ) , w

= ( 3 , -5 , -3 )

8.2.2 Problemes PAU.

1. Considereu els punts de l’espai tridimensional A = (1, 1, 0), B = (3, 5, 0) i C = (1, 0,

0) i la recta 2

1:z

yxr .

a) Trobeu el punt d’intersecció de la recta r amb el pla que passa pels punts A, B i C.

b) Trobeu els punts P de la recta r per als quals el tetraedre de vèrtexs P, A, B i C té un

volum de 2u3.

Nota: El volum d’un tetraedre de vèrtexs P, Q, R i S es pot calcular amb l’expressió

),,det(6

1PSPRPQ

Solució PAU CAT TEC SET 2018 3.5

2. Donats els vectors u=(2, –1, 0), v=(–1, 3, 4) i w=(0, 3a–1, 4a),

a) Calculeu els valors del paràmetre a perquè els vectors u, v i w siguin linealment

dependents.

b) Calculeu els valors del paràmetre a perquè un tetràedre d’arestes u, v i w tingui un

volum de 2/3 d’unitats cúbiques.

Solució PAU CAT TEC SET 2014 5.5

3. Donats els punts P= (1, –1, 2), Q= (2, 0, 1) i R= (3, 2, –1),

a) Trobeu l’equació cartesiana (és a dir, de la forma Ax+By+Cz+D= 0) del pla que

determinen.

b) Trobeu un punt S pertanyent a la recta 3

5

1

1

2

5:

zyxr , de manera que el

tetràedre de vèrtexs P, Q, R I S tingui un volum igual a 1/2.

Solució PAU CAT TEC JUNY 2013 5.3

Page 133: Geometria Lineal per al Batxillerat - toomates.net · La història dels vectors. 1.10 Recopilatori d’exercicis. 1.11 Taula resum dels tres productes. 2 Varietats lineals. Rectes

4. Una piràmide de base quadrada té el vèrtex en el pla d’equació z = 3. Tres dels

vèrtexs de la base són els punts del pla OXY: A = (1, 0, 0), B = (1, 1, 0) i C = (0, 1, 0).

a) Feu un gràfic dels elements del problema. Quines són les coordenades del quart

vèrtex de la base, D?

b) Quin és el volum de la piràmide?

3

alturabaseàreaVolum

c) Si el vèrtex de la piràmide és el punt V = (a, b, 3), quina és l’equació de la recta que

conté l’altura sobre la base?

Solució PAU CAT TEC JUNY 2005 4.6 (Problema)

5. Tenim quatre punts a l’espai: A(0, 0, 0); B(0, 0, 2); C(0, 2, 0) i D(2, 0, 0). Es demana:

a) representeu gràficament els quatre punts;

b) calculeu el volum del tetràedre (piràmide de base triangular) ABCD;

c) trobeu l’equació del pla que passa per B, C i D;

d) calculeu la distància de l’origen al pla de l’apartat anterior.

Solució PAU CAT TEC JUNY 2004 3.6

6. Donat el pla p d'equació x+ 4y+z= 8 i sent A, B i C els punts d'intersecció d'aquest

pla amb els eixos de coordenades OX, OY i OZ, respectivament:

a) Determineu les coordenades dels punts A, B i C.

b) Determineu les equacions de la recta perpendicular al pla p que passa per l'origen de

coordenades.

c) Calculeu el volum del tetràedre determinat per OABC, on O és l'origen de

coordenades.

d) Calculeu la distància de l'origen de coordenades al pla p. Determineu l'àrea del

triangle ABC (podeu utilitzar el volum calculat en l'apartat anterior)

PAU CAT TEC SET 1998 5.6

Page 134: Geometria Lineal per al Batxillerat - toomates.net · La història dels vectors. 1.10 Recopilatori d’exercicis. 1.11 Taula resum dels tres productes. 2 Varietats lineals. Rectes

9 Recopilacions d’exercicis.

9.1 Exercicis de repàs per al primer parcial.

1. Sigui la recta

0

1:

zyx

zyxr i el pla 0 zmx , on m és un paràmetre real.

Trobeu:

a) Un vector director de la recta r.

b) El valor de m per al qual la recta i el pla són paral·lels. PAU VALENCIA TEC SET 2012

2. Donades les rectes

42

4

3

4:

z

yxr i

32:

zyxs

es demana calcular raonadament:

a) Les coordenades del punt P d’intersecció entre les rectes r i s.

b) L’angle que formen les dues rectes.

c) L’equació implícita 0 DCzByAx del pla que conté les dues rectes.

PAU VALENCIA TEC SET 2010

3. Siguin A,B i C els punts d’intersecció del pla d’equació 0424 zyx amb els

tres eixos OX, OY i OZ, respectivament. Es demana calcular raonadament:

a) L’àrea del triangle ABC.

b) El perímetre del triangle ABC.

c) Els tres angles interiors del triangle ABC. PAU VALÈNCIA TEC JUNY 2009

4. Donades les rectes

013

032:

zyx

zyxr i

2

2

1

:

z

y

x

s , es demana:

a) La recta paral·lela a r que passa pel punt (0,1,0)

b) El pla que conté la recta r i és paral·lel a s. PAU VALÈNCIA TEC JUNY 2016

5. Donades les rectes

2

21

:

z

y

x

r i

21

1

1

:

z

y

x

s , on i són paràmetres reals,

calculeu raonadament:

a) Les coordenades del punt de tall de r i s.

b) L’equació del pla que conté aquestes dues rectes.

PAU VALÈNCIA TEC JUNY 2012

Page 135: Geometria Lineal per al Batxillerat - toomates.net · La història dels vectors. 1.10 Recopilatori d’exercicis. 1.11 Taula resum dels tres productes. 2 Varietats lineals. Rectes

6. Donades les rectes

3

1:

z

y

x

r

i 31: zyxs , calculeu raonadament:

a) Un vector director de cada recta.

b) El punt d’intersecció de les rectes.

c) L’equació del pla que conté a aquestes rectes. PAU VALÈNCIA TEC JUNY 2011

7. Donades les rectes 21

1

3

1:

zyxr

i

0

1

:

z

y

x

s

, i el punt )2,3,0( P ,

calculeu:

a) Les equacions de la recta que passa pel punt P i és paral·lela a la recta r.

b) L’equació del pla que conté la recta r i és paral·lel a la recta s.

PAU VALÈNCIA TEC JUNY 2015

8. Donades les rectes

10

0:

z

yxr i

13

8:

zyx

yxs , determineu:

a) Un vector director de cada recta.

b) L’equació del pla que conté la recta s i és paral·lel a la recta r.

PAU VALÈNCIA TEC JUNY 2014

Page 136: Geometria Lineal per al Batxillerat - toomates.net · La història dels vectors. 1.10 Recopilatori d’exercicis. 1.11 Taula resum dels tres productes. 2 Varietats lineals. Rectes

9.2 Exercicis de repàs de perpendicularitat i paral·lelisme.

1. Considerem la recta

4

05:

zyx

zyxr i el pla 546: zyxa .

Es demana calcular:

a) El valor de a per al què la recta i el pla siguin paral·lels.

b) El valor de a per al què la recta i el pla siguin perpendiculars.

2. Determineu si el pla 123 zyx és perpendicular a la recta

12

33:

zy

zyxr

3. Per a quin valor d’a són paral·leles les rectes:

0543

03254:

zyx

zyxr

092

910

0725

:z

ayx

zayx

s

4. a) Determina el pla que passa pel punt de coordenades )1,1,1( i talla

perpendicularment a la recta 1

1

12

1

zyx

b) Determina el punt de tall entre la recta i el pla.

5. Donades les rectes

1

0:

zyx

zyxr i

0

0:

zbxa

zyxs , trobeu la relació que ha

d’haver entre a i b de forma que:

a) Siguin rectes paral·leles.

b) Siguin rectes perpendiculars.

6. Determineu els valors d’a per als què els plans 0132 zayx i

0235 zyx siguin perpendiculars.

7. Donats els plans 3 zyx i 0 zayx , trobeu els valors de a de forma que:

a) Els plans siguin perpendiculars.

b) Els plans siguin paral·lels.

Page 137: Geometria Lineal per al Batxillerat - toomates.net · La història dels vectors. 1.10 Recopilatori d’exercicis. 1.11 Taula resum dels tres productes. 2 Varietats lineals. Rectes

9.3 Exercicis de repàs de perpendicularitat, angle i distància.

1. Sigui el pla d’equació 012423: zyx . Es demana calcular raonadament:

a) Les equacions dels dos plans paral·lels a que disten 5 unitats de .

b) Els tres punts A, B i C, intersecció del pla amb cadascun dels tres eixos de

coordenades.

c) Els tres angles del triangle ABC.

2. Donats els punts )4,1,3( P i )1,0,1( Q i el pla d’equació

0522: zyx , es demana calcular raonadament:

a) L’equació de la recta r que passa pel punt P i és perpendicular al pla .

b) L’equació del pla ' que passa pels punts P i Q i és perpendicular al pla .

3. Donats els plans 3:1 zyx i 0:2 zayx , es demana calcular

raonadament:

a) El valor de a per al què els plans 1 i 2 siguin perpendiculars, i per a aquest valor

de a, obtenir les equacions paramètriques de la recta intersecció d’aquests dos plans.

b) El valor de a per al què els plans 1 i 2 siguin paral·lels, i per a aquest valor de a,

obtenir la distància entre tots dos.

4. En l’espai tenim la recta

0

1:

zyx

zyxr i el pla 0: zmx , on m és un

paràmetre real. Obtenir raonadament:

a) El vector director de la recta r.

b) El valor de m per al què la recta r i el pla són perpendiculars.

c) El valor de m per al què la recta r i el pla són paral·lels.

d) La distància entre r i en el cas anterior.

5. Donades les rectes r i s d’equacions

42

4

3

4:

z

yxr i

32:

zyxs

es demana calcular raonadament:

a) Les coordenades del punt P d’intersecció entre ambdues rectes.

b) L’angle que formen les rectes r i s.

c) L’equació implícita 0 DCzByAx del pla que conté a totes dues.

6. Es demana calcular raonadament:

a) L’equació del pla que passa pels punts )0,0,0(O , )0,3,6( A i )1,0,3(B .

b) L’equació de la recta r que passa pel punt )2,7,8( P i és perpendicular al pla .

c) El punt Q del pla la distància del qual al punt P és menor que la distància de

qualsevol altre punt del pla al punt P.

Page 138: Geometria Lineal per al Batxillerat - toomates.net · La història dels vectors. 1.10 Recopilatori d’exercicis. 1.11 Taula resum dels tres productes. 2 Varietats lineals. Rectes

7. Sigui r la recta intersecció dels plans 0 zy i 012 yx , i sigui s la recta

d’equació 312

: zyx

s . Es demana:

a) Obtenir, raonadament, les equacions paramètriques de r i s.

b) La posició relativa entre totes dues rectes.

c) Calcular la distància entre les rectes r i s.

8. Donats els punts )1,1,2(A i )1,0,1( B , i la recta r d’equació 2

25:

zyxr ,

trobeu:

a) El punt C de r que equidista de A i B.

b) L’àrea del triangle ABC.

Page 139: Geometria Lineal per al Batxillerat - toomates.net · La història dels vectors. 1.10 Recopilatori d’exercicis. 1.11 Taula resum dels tres productes. 2 Varietats lineals. Rectes

9.4 Llista d’exercicis de rectes en l’espai.

1. Determina el valor de m per a què els punts A=(m, 0, 1), B=(0, 1, 2), C=(1, 2, 3)

i D=(7, 2, 1) siguin coplanaris.

2. Donats els punts A=(m, 2,−3), B=(2, m, 1) y C=(5, 3,−2), determineu el valor de m

per al què els punts estiguin alineats i trobeu l’equació contínua de la recta que passa per

tots tres punts.

3. Escriu l’equació cartesiana de la recta 2

1

0

3

0

4

zxx

4. Determina un vector director i les equacions paramètriques de la recta

2

0

zy

yx

5. Escriu l’equació cartesiana de la recta zyx

1

1

2

6. Escriu la recta

03

013:

zyx

zyxr en forma paramètrica i contínua.

7. Determina l’equació contínua de la recta que passa pel punt )1,1,1( P i és

paral·lela a la recta que passa pels punts )1,0,2(A i )3,2,1(B .

8. Donats els punts A=(1, 3,2), B=(2, 5, 1) i C=(3, 0, -4), determineu l’equació

paramètrica de la recta que passa per C i és paral·lela a la recta determinada pels punts

A i B.

9. Determina l’angle que formen les rectes:

zyx

r

12

2: z

yxs

21

1:

10. Determina l’angle que formen les rectes:

022

0132:

zyx

zyxr

0132

033:

zyx

zyxs

11. Determina l’angle que formen les rectes:

3

2

2

1

1

1:

zyxr

0132

0:

zyx

zyxs

12. Determina el volum del tetraedre de vèrtexs A=(3, 2, 1), B=(1, 2, 4), C=(4, 0, 3) y

D=(1, 1, 7).

Page 140: Geometria Lineal per al Batxillerat - toomates.net · La història dels vectors. 1.10 Recopilatori d’exercicis. 1.11 Taula resum dels tres productes. 2 Varietats lineals. Rectes

13. Estudia la posició relativa de les següents rectes:

5

1

23

:

z

y

x

r

5

33

61

:

z

y

x

s

14. Estudia la posició relativa de les següents rectes:

0

:

z

y

x

r

z

y

x

s 3

3

:

15. Estudia la posició relativa de les següents rectes:

1

2

3

:

z

y

x

r

1

23

2

:

z

y

x

s

16. Calcula a i b para que els punts A=(1, 2, –1), B=(3, 0, –2) y C=(4, a, b) estiguin

alineats.

17. Estudia la posició relativa de les següents rectes i troba el punt de tall quan sigui

possible:

a) 4

1

2

2

3

1:

zyxr

3

2

2

3

1

2:

zyxr

b) 1

2

2

1

1

1:

zyxr

2

5

1

4

4

4:

zyxs

c) 3

11

2:

zy

xr

013

012:

zy

yxs

d) 432

1:

zyxr

84

63

43

:

z

y

x

s

18. Determina el valor de a per al què les rectes r i s es tallen, i troba el punt de tall:

azyxr : 0

2

2

3

3

12:

zyxs

Page 141: Geometria Lineal per al Batxillerat - toomates.net · La història dels vectors. 1.10 Recopilatori d’exercicis. 1.11 Taula resum dels tres productes. 2 Varietats lineals. Rectes

19. Determina els valors de m i n per als què les rectes r i s siguin paral·leles:

z

y

x

r 3

45

: n

zy

m

xs

3

3

1:

20. Estudia la posició relativa de la recta r determinada pels punts )1,1,1(A i

)2,1,3(B i la recta

02

012:

y

zxs

21. Demostra que les rectes )6,2,4()5,0,1(: r i )3,1,2()8,1,3(: s

són la mateixa recta.

22. Determina el valor de a per a què les rectes r i s siguin coplanàries:

2

02:

zy

yxr

azx

yxs

2

1:

23. Determina m per a què les rectes

2

21

3

:

z

y

x

r

0743

0754:

mzy

yxs

siguin coplanàries.

24. Estudiar, segons els valors del paràmetre a, la posició relativa de les rectes r i s:

az

y

ax

r 1

)2(

:

1

2

1:

3

a

az

a

yxas

25. Determina els valors de a i b per als què les rectes siguin paral·leles:

zyxr 624:

332

12:

zbyx

zyaxs

26. Trobeu els valors de m i n per als què les rectes r i s siguin paral·leles :

z

y

x

r 3

55

: n

zy

m

xs

3

3

1:

27. Comproveu que les rectes

0

0:

zx

yr i

4

0:

y

xs

es creuen i trobeu la distància entre ambdues.

Page 142: Geometria Lineal per al Batxillerat - toomates.net · La història dels vectors. 1.10 Recopilatori d’exercicis. 1.11 Taula resum dels tres productes. 2 Varietats lineals. Rectes

28. Determineu les coordenades del punt 'A , simètric de )3,0,2(A respecte a la recta

2

1

1

2

1

1:

zyxr

9.5 Llista de repàs general.

1. Considereu les dues rectes r i s de IR

3 donades per les equacions següents:

222

1:

zayx

azyxr

2

5323:

zx

zybxs

Determineu els valors d’a i b sabent que són paral·leles.

2. Considereu la recta de IR3 donada per

1

2

zx

zyx i el pla d’equació 3 yx .

a) Determineu el seu punt de tall.

b) Digueu quin angle formen aquesta recta amb aquest pla.

3. Busqueu el punt simètric de (1,2,3) respecte del pla 0423 zyx

4. Busqueu l’equació del pla que conté la recta 2

1

2

1

zy

x i que passa pel punt (-

1,2,1).

5. Considereu la recta que passa pel punt (1,2,0) i que té (1,1,1) com a vector director, i

la recta que passa per (0,0,1) i que té (1,2,1) com a vector director. Escriviu les

equacions paramètriques de la recta que talla les dues anteriors i que és perpendicular a

cadascuna.

Page 143: Geometria Lineal per al Batxillerat - toomates.net · La història dels vectors. 1.10 Recopilatori d’exercicis. 1.11 Taula resum dels tres productes. 2 Varietats lineals. Rectes

9.6 Llista de repàs general.

1. Troba l’equació del pla que conté la recta zyx 32 i és paral·lel a la recta

42

2

23

:

z

y

x

r

2. Troba el punt del pla 032243: zyx més proper al punt )4,2,1(P .

3. Estudia la posició relativa dels plans

12:

1:

12:

3

2

1

zymx

mzymx

zyx

en funció de m.

4. Es considera el punt )1,2,5(P i la recta 32: zyxr . Troba el punt simètric

de P respecte de r.

5. Siguin els punts )0,4,3(A , )3,6,3(B i )1,2,1(C els vèrtexs d’un triangle.

a) Calcula l’equació del pla que els conté.

b) Troba l’equació contínua de r perpendicular a que passa per l’origen de

coordenades.

6. Donades les rectes

24

1

3

:

z

y

x

r , 4

5

1

3

3

4:

zyxs

Calcula les equacions paramètriques de la recta perpendicular comú a r i s.

7. Determina el valor de m per al què la recta 33: zm

yxr i el pla

024: mzyx formin un angle de 30º.

Page 144: Geometria Lineal per al Batxillerat - toomates.net · La història dels vectors. 1.10 Recopilatori d’exercicis. 1.11 Taula resum dels tres productes. 2 Varietats lineals. Rectes

10 Apèndix.

10.1 Taula Paral·lelisme-Perpendicularitat-Angle.

Angle Perpendicularitat Paral·lelisme

Entre dos vectors:

vu

vu

cos 0 vuvu

vkuvu

//

Entre dues rectes:

3

3

2

2

1

11 :

u

pz

u

py

u

pxr

),,( 321 uuuu

3

3

2

2

1

12 :

v

qz

v

qy

v

qxr

),,( 321 vvvv

vu

vu

cos

0 vusr

vkusr

//

Entre dos plans:

DCzByAx :1

'''':2 DzCyBxA

)',','(,),,( 21 CBAnCBAn

21

21cos

nn

nn

021

21

nn

2121 // nkn

Entre recta i pla:

DCzByAx :1

3

3

2

2

1

1:v

pz

v

py

v

pxr

),,( 321 vvvv

un

un

cos

º90

vkn

r

0// vnr

I, a més a més, hem de

comprovar que el punt

base de la recta no

pertany al pla

Page 145: Geometria Lineal per al Batxillerat - toomates.net · La història dels vectors. 1.10 Recopilatori d’exercicis. 1.11 Taula resum dels tres productes. 2 Varietats lineals. Rectes

10.2 Que l’immens poder de la Projecció ortogonal t’acompanyi!

Si et donen un punt P i una recta r...

Si et donen un punt P i un pla ...

... determina el pla perpendicular a r que

passa per P, agafant vn

... determina la recta perpendicular a

que passa per P, agafant nv

... i troba el punt d’intersecció Q entre la

recta i el pla.

Aquest punt és la Projecció ortogonal de

P en la recta r

... i troba el punt d’intersecció Q entre la

recta i el pla.

Aquest punt és la Projecció ortogonal de

P en el pla

La distància entre el punt P i la recta r serà

PQQPdistrPdist ),(),(

La distància entre el punt P i el pla serà

PQQPdistPdist ),(),(

El punt simètric de P respecte a r serà

PQPP 2'

El punt simètric de P respecte a serà

PQPP 2'

Page 146: Geometria Lineal per al Batxillerat - toomates.net · La història dels vectors. 1.10 Recopilatori d’exercicis. 1.11 Taula resum dels tres productes. 2 Varietats lineals. Rectes

10.3 Algunes demostracions.

Demostració 1.3.3.

vuvu

Ho demostrarem per al cas bidimensional. Si ),( 21 uuu

i ),( 21 vvv

,

2

222

2

2

2

111

2

1

2

22

2

1122112121

22

)()(),(),(),(

vvuuvvuu

vuvuvuvuvvuuvu

2

2

2

1

2

2

2

12121 ),(),( vvuuvvuuvu

Hem de demostrar, doncs, que

2

2

2

1

2

2

2

1

2

222

2

2

2

111

2

1 22 vvuuvvuuvvuu

Si elevem al quadrat els dos costats de la desigualtat:

2

222

2

2

2

111

2

1

22

222

2

2

2

111

2

1 2222 vvuuvvuuvvuuvvuu

2

2

2

1

2

2

2

1

2

2

2

1

2

2

2

1

22

2

2

1

2

2

2

1 2 vvuuvvuuvvuu

Simplificant 2

2

2

2

2

1

2

1 vuvu a esquerra i dreta, hauríem de comprovar que la resta

compleix: 2

2

2

1

2

2

2

12211 222 vvuuvuvu

Si elevem de nou al quadrat:

2211

2

2

2

2

2

1

2

12211

2

22

2

11

2

2211 8442222222 vuvuvuvuvuvuvuvuvuvu

2

2

2

2

2

1

2

2

2

2

2

1

2

1

2

1

2

2

2

1

2

2

2

1

22

2

2

1

2

2

2

1 442 vuvuvuvuvvuuvvuu

Simplificant 2

2

2

2

2

1

2

1 44 vuvu a esquerra i dreta, s’ha de demostrar que 2

2

2

2

2

2

2

12211 448 vuvuvuvu

Simplificant: 2

2

2

2

2

2

2

122112 vuvuvuvu

Posant el primer membre en el segon: 2211

2

2

2

2

2

2

2

1 20 vuvuvuvu

Finalment veiem que la part de la dreta s’adapta a la igualtat 222 )(2 bababa

És a dir, que 222210 vuvu

I aquesta última expressió és correcta, ja que qualsevol nombre al quadrat és més gran o

igual a 0. Per tant, l’expressió inicial també és certa: vuvu

Demostració de 1.5.2.

vuvu

Aquí utilitzarem que BA , amb 0B 22 BA

2211 vuvuvu

, 2

2

2

1

2

2

2

1 vvuuvu

Elevant al quadrat:

2

2

2

1

2

2

2

1

22

2

2

1

2

2

2

1

2

2211 vvuuvvuuvuvu

2

2

2

2

2

1

2

2

2

2

2

1

2

1

2

1

2

2

2

1

2

2

2

1 vuvuvuvuvvuu

2211

2

2

2

2

2

1

2

1

2

2211 2 vuvuvuvuvuvu

Page 147: Geometria Lineal per al Batxillerat - toomates.net · La història dels vectors. 1.10 Recopilatori d’exercicis. 1.11 Taula resum dels tres productes. 2 Varietats lineals. Rectes

Eliminem els termes comuns 2

2

2

2

2

1

2

1 vuvu , i ens queda

022 2211

2

1

2

2

2

2

2

1

2

1

2

2

2

2

2

12211 vuvuvuvuvuvuvuvu

Però això és evident, ja que:

222112211

2

1

2

2

2

2

2

1 2 vuvuvuvuvuvu

i un quadrat mai és negatiu.

Page 148: Geometria Lineal per al Batxillerat - toomates.net · La història dels vectors. 1.10 Recopilatori d’exercicis. 1.11 Taula resum dels tres productes. 2 Varietats lineals. Rectes

10.4 Problemes PAU de geometria del pla. (Long time ago...)

1. a) Dibuixeu el gràfic de les rectes 3x –y –1 = 0 i x +3y –12 = 0.

b) Demostreu que les dues rectes anteriors són perpendiculars.

c) Calculeu el punt d’intersecció de les dues rectes.

d) Considereu el triangle format per les dues rectes anteriors i per l’eix d’ordenades.

Calculeu-ne l’àrea.

Solució PAU CAT CCSS SET 2003 3.6

2. Expliqueu quina condició han de verificar A i B si les rectes d’equacions

Ax + By + C = 0 i 3

2

2

1

yx

a) són paral·leles;

b) són perpendiculars.

Solució PAU CAT CCSS JUNY 2003 2.4

3. Un triangle rectangle té el vèrtex A, corresponent a l’angle recte, a l’origen de

coordenades. Un altre dels seus vèrtexs és el punt B(2,4), i la hipotenusa té per equació

la recta x = 2. Calculeu:

a) les equacions dels costats AB i AC;

b) el tercer vèrtex C;

c) l’àrea del triangle. Solució PAU CAT CCSS JUNY 2003 2.6

4. Considereu els punts del pla A(2,–1) i B(0,3) i la recta r d’equació x + y – 2 = 0.

Calculeu les coordenades d’un punt C de r que estigui alineat amb A i B.

Solució PAU CAT CCSS JUNY 2003 5.3

5. Calculeu el perímetre del triangle rectangle ABC, sabent que la longitud del segment

CP és 32 .

Solució PAU CAT CCSS JUNY 2003 5.4

6. El costat BC d'un triangle està sobre la recta d'equació 3x –2y +1 = 0. El vèrtex A té

coordenades (2, –1). Determineu el peu de l'altura relativa a A.

Solució PAU CAT CCSS SET 2002 1.2

Page 149: Geometria Lineal per al Batxillerat - toomates.net · La història dels vectors. 1.10 Recopilatori d’exercicis. 1.11 Taula resum dels tres productes. 2 Varietats lineals. Rectes

7. Un triangle té dos vèrtexs A i B en els punts A = (0, 0) i B = (2, 0). L'àrea val 3.

Sabent que el tercer vèrtex C té ordenada positiva i està situat sobre la recta 2x –y –5 =

0, calculeu les coordenades de C i el perímetre del triangle. Feu-ne la gràfica

corresponent.

Solució PAU CAT CCSS JUNY 2002 3.6

8. Considereu els punts del pla A(3, 2), B(–1, 8) i C( k, k+4 ), k real. Calculeu el valor

de k perquè A, B i C estiguin alineats.

Solució PAU CAT CCSS JUNY 2002 2.3

9. a) Determineu l'equació de la recta paral·lela a la bisectriu del segon i quart quadrant

que passa pel punt (0, a).

b) Determineu el valor de a perquè la recta anterior determini en el primer quadrant un

triangle d'àrea 8 amb els eixos.

c) Quina és la distància d'aquesta recta a l'origen de coordenades?

d) Quina és la distància d'aquesta recta al punt (–4, 0)?

Solució PAU CAT CCSS JUNY 2002 2.5

10. Sigui r la recta d'equació 6x –15y +4 = 0. Trobeu les equacions de les rectes

paral·lela i perpendicular a r que passen pel punt (4, 1) i feu un esquema gràfic.

Solució PAU CAT CCSS SET 2001 4.3

11. Siguin r i s les dues rectes del pla d'equacions

032: yxr , 2

2

4

1:

yxs

Calculeu l'equació de la recta que passa pel punt d'intersecció de r i s i que és paral·lela

a la recta d'equació 3x + 5y – 1 = 0.

Solució PAU CAT CCSS JUNY 2001 2.3

12. Al triangle de vèrtexs A = (0, 3), B = (3, 7) i C = (6, 0) determineu

a) el perímetre;

b) l'equació de la recta perpendicular al segment BC que passa per A, és a dir, l'altura

del triangle des del vèrtex A;

c) la distància del punt A a la recta que conté el segment BC;

d) la superfície.

Solució PAU CAT CCSS JUNY 2001 2.6

13. Sigui r la recta d'equació 3x –5y +2 = 0. Trobeu les equacions de les rectes

paral·lela i perpendicular a r que passen pel punt (–15, 4).

Solució PAU CAT CCSS JUNY 2001 5.3

Page 150: Geometria Lineal per al Batxillerat - toomates.net · La història dels vectors. 1.10 Recopilatori d’exercicis. 1.11 Taula resum dels tres productes. 2 Varietats lineals. Rectes

14. Els punts A = (2, 5), B = (6, 8) i C = (22, d) estan alineats. Calculeu d.

Solució PAU CAT CCSS JUNY 2001 5.4

15. Considereu la recta d'equació y = –2x + 2. Trobeu les coordenades del punt

d'intersecció d'aquesta recta amb la recta perpendicular a ella que passa pel punt (6, 3).

PAU CAT CCSS SET 2000 2.3

16. Considereu la recta d'equació 4x + y – 3 = 0.

a) Calculeu l'equació de la recta paral·lela i de la recta perpendicular a l'anterior que

passen pel punt A = (3, –1).

b) Dibuixeu la gràfica de la recta 4x + y – 3 = 0 i de les que heu trobat a l'apartat a).

Solució PAU CAT CCSS SET 2000 6.1

17. Determineu el valor que ha de tenir el paràmetre a perquè les tres rectes d'equacions

3x + y = 5, x – 3y = –5 i x + ay = a es tallin en un punt.

Solució PAU CAT CCSS SET 2000 6.2

18. D'un rombe ABCD coneixeu les coordenades de tres vèrtexs. A és l'origen de

coordenades, B = (4, 1) i D = (1, 4).

a) Calculeu les coordenades del quart vèrtex C.

b) Comproveu analíticament que les diagonals són perpendiculars i que es tallen en el

seu punt mitjà.

Solució PAU CAT CCSS JUNY 2000 1.5

19. Determineu el valor de a perquè la recta x – 2ay = 1 i la recta x + 3y = 8 siguin:

a) paral·leles

b) perpendiculars

Solució PAU CAT CCSS JUNY 2000 3.3

Page 151: Geometria Lineal per al Batxillerat - toomates.net · La història dels vectors. 1.10 Recopilatori d’exercicis. 1.11 Taula resum dels tres productes. 2 Varietats lineals. Rectes

20. Considereu dos eixos perpendiculars de coordenades. Considereu els punts O i A de

coordenades O = (0, 0) i A = (9, 12). Una persona situada al punt O inicia un viatge en

línia recta cap a A.

a) Quina distància haurà de recórrer per anar de O a A?

b) Escriviu l'equació de la recta que haurà de seguir per anar de O a A.

c) Digueu quines seran les coordenades del punt P on es trobarà la persona quan hagi

recorregut la tercera part de la distància de l'apartat anterior (sempre sobre la recta que

uneix O amb A).

d) Si després d'haver recorregut el segment OP, quan arribi a P decideix dirigir-se cap al

punt Q = (7, 1), quin angle haurà de girar cap a la dreta? (Angle respecte a la trajectòria

OP que havia seguit fins ara.)

Solució PAU CAT CCSS JUNY 2000 3.5

21. a) Considereu el triangle de vèrtexs A = (2, 1), B = (4, 3) i C = (0, 3). Dibuixeu-lo.

Comproveu després per algun raonament matemàtic (no només gràficament) que és un

triangle rectangle.

b) Calculeu la seva àrea.

PAU CAT CCSS SET 1999 2.2

22. Considereu el rectangle del pla representat en el dibuix (recordeu que rectangle és

un quadrilàter en què els quatre angles són rectes).

a) Sabent que les coordenades de A són (0, 0) i les de B són (3, 4), calculeu la longitud

del costat AB.

b) Escriviu l'equació de la recta determinada per C i A.

c) Determineu les coordenades del vèrtex C sabent que la longitud del costat CA és

doble de la del costat AB.

d) Calculeu les coordenades del vèrtex D.

PAU CAT CCSS SET 1999 5.6

23. Escriviu l'equació de les dues rectes que passen pel punt (3, 2) i formen un angle de

45º amb l'eix de les x tal com s'indica en el dibuix següent:

Solució PAU CAT CCSS JUNY 1999 1.2

Page 152: Geometria Lineal per al Batxillerat - toomates.net · La història dels vectors. 1.10 Recopilatori d’exercicis. 1.11 Taula resum dels tres productes. 2 Varietats lineals. Rectes

24. La hipotenusa d'un triangle rectangle és el triple que un catet. Busqueu el valor dels

angles d'aquest triangle i la relació entre la hipotenusa i l'altre catet.

(Useu calculadora per a les raons trigonomètriques. Si no, podeu deixar les operacions

indicades.)

Solució PAU CAT CCSS JUNY 1999 6.1

25. Els punts A = (1, 2) i D = (5, 4) representen els vèrtexs oposats d'un quadrat, tal com

s'indica a la figura.

a) Calculeu el punt mitjà M de la diagonal AD del quadrat (M serà el centre del

quadrat).

b) Escriviu l'equació de la recta que passa per M i és perpendicular a la diagonal AD

(aquesta recta serà l'altra diagonal del quadrat).

c) Calculeu les coordenades dels altres dos vèrtexs B i C del quadrat.

Solució PAU CAT CCSS JUNY 1999 6.6

26. Trobeu les coordenades del punt simètric de P= (3, –4) respecte a la recta

0 6 3y 2x (el punt simètric de P respecte a la recta r és el punt P' , que té la propietat

que la recta determinada per P i P' talla perpendicularment r en el punt mitjà del

segment PP').

PAU CAT CCSS SET 1998 5.4

27. Un triangle ABC té els vèrtexs A i B situats respectivament en els punts de

coordenades (1, 3) i (3, 1). El vèrtex C està situat sobre la recta d'equació 2x –y = 4.

Sabent que el triangle ABC és isòsceles i que AC i BC són els costats iguals, trobeu les

coordenades de C i l'àrea del triangle.

PAU CAT CCSS SET 1998 5.6

Page 153: Geometria Lineal per al Batxillerat - toomates.net · La història dels vectors. 1.10 Recopilatori d’exercicis. 1.11 Taula resum dels tres productes. 2 Varietats lineals. Rectes

28. Sigui v

el vector de components (1, 0). Considereu els punts del pla que tenen per

coordenades A=(–2, 9) i B=(4, 7).

a) Calculeu els components del vector u

, que va del punt A al punt B.

b) Calculeu el valor del producte escalar vu

c) Calculeu el valor de l'ordenada x del vector ),2( xw

, de manera que el vector

vu

3 sigui perpendicular al vector w

.

PAU CAT CCSS SET 1998 2.3

29. Considereu els punts A=(–1,3), B=(5,4), C=(4,1) i D=(–2,0). Comproveu que el

quadrilàter ABCD és un paral·lelogram i calculeu-ne les coordenades del centre (és a

dir, del punt mitjà de qualsevol de les dues diagonals).

Solució PAU CAT CCSS JUNY 1998 3.4

30. Considereu el parell de rectes donades per les equacions

2)2( ayaxa i 3 yax , on a és un paràmetre.

a) Calculeu un vector director de cadascuna d'aquestes.

b) Calculeu els valors de a per als quals les rectes són paral·leles.

c) Calculeu els valors de a per als quals les rectes són perpendiculars.

d) Calculeu la distància que hi ha entre les dues rectes quan a= 2.

Solució PAU CAT CCSS JUNY 1998 3.6

31. Comproveu que les rectes d'equacions

313 yx i 133 yx

es tallen en el punt (1, 1). Calculeu l'angle que formen.

Solució PAU CAT CCSS JUNY 1998 6.3

Page 154: Geometria Lineal per al Batxillerat - toomates.net · La història dels vectors. 1.10 Recopilatori d’exercicis. 1.11 Taula resum dels tres productes. 2 Varietats lineals. Rectes

Solucions

1.1.9 1. )5,1,4( vu

, 2. )4,2,1( vu

, 3. (2.6 , -3.9 , -6.5 ), 4. (0.7 , 2.8 , -1.4 )

1.2.7

1. a) b)

c) d)

2. a) (-1,0,1) b) (-4,2,3) c) (2,2,-1) d) (-4,-2,1)

1.3.6 1. 8.124 2. 23.664

1.3.7

1. 7 DACDBCAB , i per tant és un rombe.

2. a)

83)()2()2( 2222 AB

82)2(0 222 AC

83)2()2( 2222 BC

i per tant els costats AB i BC són iguals.

b) 0883 2

3. )0,,0( yP , )0,2,0(210)03()2()01( 222 Pyy

Page 155: Geometria Lineal per al Batxillerat - toomates.net · La història dels vectors. 1.10 Recopilatori d’exercicis. 1.11 Taula resum dels tres productes. 2 Varietats lineals. Rectes

4. )0,0,( xP

0,0,4

49

4

49)40()50()5(

)20()20()3(

222

222

Pxx

xPBPAPBPA

5. 231611 PQ , 23099 QR , 616416 PR

i per tant QRPQ i és isòsceles. 222 361818 PRQRPQ es

verifica el Teorema de Pitàgores, i per tant és un triangle rectangle en el vèrtex

Q.

1.3.8 1.

0,

2

1,9P

1.4.4 1. No. 2. Sí. 3. 5x .

1.4.5 1. 2,1,1

1

1

01

bak

kb

k

a

ACkAB 3. 2,5 ca

1.5.6 1. a) k=2 b) No té solució. 2. 52.589° 3. 30.894. 4. a=0

5. 2,2 yz , 3

2,

2

2 yz

1.5.7 4. 2a , 5/4a

1.6.4 1. )1,6,4( ba

, )1,6,4(ab

2. )3,3,3( o )3,3,3(

3. a) 19 b) (-8,-11,-1) c) 95º 46’ 5’’

4. )0,1,0(D , perímetre = 64 , àrea = 32 .

1.7.6 1. a) 6,33,21 , 72, 72 b) )35,15,31( , 27, 27

c) )10,15,18( , -89, -89

2. 11 DetVol

3. )0,0,0(F , )3,0,3(G , )4,0,5(H , volum = 3

4. a) Són coplanaris. b) No són coplanaris.

5. a) 4x , b) 1x

1.7.7 1. a=-2, b=0 o a=5, b=7 2. )2,0,1( C i )2,1,0( D

3. 91 4. 5/6 7 a) 6,0 , 3

9.

1

0

0

0)1(0

0)1()1(0

1

01

01

),,1(

),0,1(

)0,,1(

22

ba

b

a

baabababba

cbababa

ba

ba

ba

baAD

baAC

baAB

Si a = 0 els punts són: (1, 0, 0), (0, b, 0), (0, 0, b) y (0, 0, b); i per tant els dos

últims coincideixen.

Si b = 0 los punts són: (1, 0, 0), (a, 0, 0), (a, 0, 0) y (0, a, 0); i també hi

coincideixen dos.

Page 156: Geometria Lineal per al Batxillerat - toomates.net · La història dels vectors. 1.10 Recopilatori d’exercicis. 1.11 Taula resum dels tres productes. 2 Varietats lineals. Rectes

Per tant, per a que els punts siguin diferents i coplanaris és necessari que

1ba , amb a i b diferents de zero.

10. Volum=5/6

11.

3

4028210

331

520

111

)3,3,1(

)5,2,0(

)1,1,1(

aa

a

AD

AC

aAB

3/2

3/1072821

6

17),,det(

6

1

a

aaADACABV

1.10

1. b= 5, c= -6, e= 5

2. )6,5/18,1( , )6,5/21,4( , )6,5/24,7( , )6,5/27,10(

3. a) b=-4, c=1, d=10 b)

2

7,

2

3,

2

1 c) )6,3/4,3( i )8,3/5,4(

4. 0,2 ba o 7,5 ba 5. no existeix cap k.

6. k=18 7. 2,2 yz o 3/2,3/2 yz

8. )3,3,3( w

i )3,3,3( w

9. a) 11 , 13 b) )13/11arccos( c) z= -3

10. a) 203 b) 12 11. 2

103

12. a) z=0 b) 854 13. a) -49 b) 13 i 65 c) 117º52’

d) 4/3x

14. Hi ha dues solucions:

2

1,

2

1,

2

1u

i

2

1,

2

1,

2

1u

15. Hi ha dues solucions: 1m i 13/35m

16. 11 o 11 17. 5,5

1,

5

3 cba , 90º

19. a) 2a b) 1a 20. c) 24

21.

21

1,

21

4,

21

2 22. 0a o 1a

23. b) 1Àrea 24. a) 2

19 b) 7/6

25. a) D=(0,2,2) b) 22

26. a) 172 b) 3/50

27. a) 3/4a b) 3/10a o 3/2a

28. a) 24 CDAB , 42 ADBC , els seus costats oposats són iguals,

per tant és un paral·lelogram. Àrea= 2512

2.1.8 1. (-10, 2, -4), (15, -3, 6), (-2.5, 0.5, -1)

2. (8, 28, 20), (-2, 7, -5), (2/3, -7/3, 5/3)

3. 9Det per tant són lin.ind. 4. k = -13/4

Page 157: Geometria Lineal per al Batxillerat - toomates.net · La història dels vectors. 1.10 Recopilatori d’exercicis. 1.11 Taula resum dels tres productes. 2 Varietats lineals. Rectes

5. cbav

2

5

2

1

2

3

2.19 5. a) 7

001

312

411

per tant són lin.ind. b) 0375 ba

6. k=2

7. a) Si 1a lin. dep. en cas contrari lin. ind. b) wvuc

32

3

2

3

2.2.3 1. 11

6

27

10

3

11

zyx 2.

11

13

21

20

23

17

zyx

3. P = ( 15 , 8 , -6 ) 4. ( 0 , -5 , 7 )

2.3.3 1. a) 18582 zyx , b) 522 zyx , c) 1057 zyx ,

d) 15119 zyx

2. 032 zyx 3. 0742 zyx 4. 017124 zyx

Problemes.

1. a) 6236 zyx 2. a) 3a , b) 1152 zyx

3. b) 0522 zyx 4. a) 012 zyx b) 1k

5. 0 zyx 6. 12 zy 7.

2.4.1 1. ( 1 , 1 , -1 )

2.6.2 1. No són coplanàries. 2. Sí són coplanàries.

2.6.3 1. 1k

2.6.4 1. 0123 qp 2. 2/17a . 3. a) a=2, b=4 b) 082 ba

2.7.2 1.

5

7,

5

14,1P ,

5

30d

3.1.5 Exercicis.

1. 4

3

3

5

2

1

zyx (o equivalent)

2. 6

3

3

4

8

1

zyx (o equivalent)

3. 0,7,1 4. 0,1,4

3.1.6 3. El punt no es troba sobre la recta paral·lela.

4. 1k .

5. No es tallen per a cap valor de a. Són paral·leles per a 1a .

6. 3/5,0 ba , 1,1 ba .

3.2.4 1. zyx 415 2. )5(452 zyx

3.2.5 1. a) Veure teoria. b) 2a , 12

6dist i 1a ,

3

38dist

3.6 1. zyx 2

4.4.2 1. (-2/19, -2/19, -83/19) 2. (8/7, 10/7, -9/7) 3. (-151/22, -19/22, -21/11)

4.4.3 4. (-124/21, 67/21, -92/21) 5. (-39/19, -22/19, 16/19) 6. (8/7, -79/7, -38/7).

7. (-1,0,-2)

4.4.4 4.

3

3,

3

3,

3

3P , 1d 5.

23

42,

23

9,

23

15 6.

23

42,

23

9,

23

15

4.5.2 1. (5/21, 64/21, -73/21) 2. (-51/14, -9/7, 57/14) 3. (39/38, -159/38, -23/19)

Page 158: Geometria Lineal per al Batxillerat - toomates.net · La història dels vectors. 1.10 Recopilatori d’exercicis. 1.11 Taula resum dels tres productes. 2 Varietats lineals. Rectes

4. (7/3, -10/3, 4/3)

5. (-8/27, -127/27, -40/27) 6. (-2/13, 45/13, 5/13) 7. (10/3, -16/15, -92/15)

4.6.2 1. a) )0,1,1(4

1,

4

1,

4

1),,(

zyx ,

2

2dist

b) 42

5

2

3

z

yx, 3dist .

c) 1

2

2

1

2

1

zyx d)

3

6

z

ky

kx

e)

1

2

z

y

x

4.6.3 1. (1,1,-1) i (2,0,1) 2. )1,9/13,9/17(P , )9/11,9/11,2('P

3.

2

4

3

z

y

x

4. )6,4,3(A , )5,5,3(B

5.1.5 1. 2d

5.2.4 1. 3

311

5.3.2 1. 7.121 2. 3.818

5.3.3 1. 02422:1 zyx , 03622:2 zyx

5.5.4 2. 14

143

5.7 5. ( 1, 1, 0 ) i ( 2, 3, -1 )

6.1.3 1. La recta i el pla es tallen al punt )2/5,1,2/5(

2. Si 5k la recta i el pla es tallen al punt

k

k

k

k

k 5

312,

5

4,

5

1

Si 5k la recta i el pla són paral·lels.

3. Si t=1 la recta no està ben definida, és un pla. Si t=-2 és una recta continguda

en el pla i si 2,1t es tallen.

Problemes.

4. a) 937 zyx b) Recta i pla es tallen en un punt.

6.2.3 1. a) 1. Dos plans secants en la recta

2

15

5

0

4/13

4/3

z

y

x

b) Dos plans paral·lels i no coincidents.

c) Dos plans coincidents.

2. 2m i 6n .

6.2.4 2. Si 2a són paral·lels. Si 2a es tallen en una recta.

6.3.3 1. Es creuen. 2. Es tallen al punt )2,1,1(

3. Es creuen.

4. Si 1t no són rectes sinó plans coincidents, si 3t es tallen, si 3,1t es

creuen.

6.3.5 1. a=1, P=(1,0,1) 2. (2,1,3) 3. Les rectes no es tallen per cap

valor de a.

Page 159: Geometria Lineal per al Batxillerat - toomates.net · La història dels vectors. 1.10 Recopilatori d’exercicis. 1.11 Taula resum dels tres productes. 2 Varietats lineals. Rectes

4. 1k 5. 2a (per a 6a les rectes són coincidents)

6.4.3 1. Si t=1 els tres plans són coincidents. Si t=-1/2 es tallen dos a dos segons

rectes paral·leles, si 2/1,1t es tallen en un punt.

6.4.4 9. Si 1a el sistema és incompatible (Prisma triangular). Si 1a el sistema

és compatible indeterminat amb un grau de llibertat (tres plans que es tallen en

una recta). Si 1,1a el sistema és compatible determinat (tres plans que es

tallen en un punt).

10. Per 0a o 15a incompatible (prisma triangular). Per a qualsevol altre

valor de a, compatible determinat (tres plans amb un punt d’intersecció).

7.1.2 1. 49.107 ° 2. 42.906 °

7.1.4 1. a) Les rectes corresponen als valors 214a i 214a

b) L’angle té 151

1512cos

2. a=0

3. )1,,1( aw

)2,1,1( v

42

21

2)1(111

)2,1,1)(1,,1(),cos()º60cos(

2

1

2

222222

a

a

a

a

vw

vwvw

793.0828.4827.52

828.4827.52)414.2(2

414.221

2122

211

42

21

2

1

2222

2

2

22

aa

aaaaa

aaa

a

a

a

Valor exacte: 222

221

a

4. 214a i 214a

7.2.2 1. 32.513° 2. 42.45°

7.2.3 1.

2.

3. a) Es tallen. b) 222

221

a

8.1.1 1. 7.28 2. 21.564

8.2.1 1. 16 2. 58

9.1 1. a) )1,0,1(v

, 1m

2. a) P=(1,2,3) b) 44.4153º c) 02 zyx

3. a) A=(4,0,0), B=(0,1,0), C=(0,0,-2), 21A . b) 10.83

c) º805.29ˆ A , º773,83ˆ B , º422,66ˆ C

Page 160: Geometria Lineal per al Batxillerat - toomates.net · La història dels vectors. 1.10 Recopilatori d’exercicis. 1.11 Taula resum dels tres productes. 2 Varietats lineals. Rectes

4. a)

7

41

z

y

x

b) 06210 zyx

5. a) (-1,-1,3), b) 0324 zyx

6. a) )0,1,1( v

, )1,1,1(w

b) (1,0,3) c) 052 zyx

7. a) 2

2

1

3

3

zyx b) 0 zyx

8. a) )0,1,1(v

, )0,1,1(w

b) 5z

9.2 1. a) a = 26, b) a = -2

2.Sí és perpendicular.

3. Són paral·leles i no coincidents per 4a .

4. 42 zyx ,

2

1,

2

1,2

5 Són paral·leles si ba , són perpendiculars si ba

6. 19a

7. a) 2a , b) 1a

9.3 1. a) 029512423:1 zyx , 029512423:2 zyx

b) )0,0,4(A , )0,6,0(B , )3,0,0(C

c) º6560.63A , º9088.41B , º4352.74C

2. a) 2

4

2

1

1

3

zyx b) 01138:' zyx

3. a) 2a

1

2

z

y

x

b) 1a 3dist

4. a) )1,0,1(v

b) 1m c) 1m d) 3536.022

1d

5. a) )3,2,1(P b) º4153.44 c) 02 zyx

6. a) 0963 zyx b) 3

2

2

7

1

8

zyx c) )4,3,6(Q

7. a)

z

y

x

r

21

:

3

1

2

:

z

y

x

s b) Són dues rectes paral·leles

c) dist=3

5

8. a)

1,

2

1,

2

9C b)

2

66

9.4

1. m = -1 2. 6m , 1

3

1

2

1

6:

zyxr 3.

03

04

y

x

Page 161: Geometria Lineal per al Batxillerat - toomates.net · La història dels vectors. 1.10 Recopilatori d’exercicis. 1.11 Taula resum dels tres productes. 2 Varietats lineals. Rectes

4. )1,1,1( v

,

2z

y

x

(la solució no és única) 5.

022

02

yx

zx

6.

22

1

z

y

x

2

2

1

1

1

zyx 7.

2

1

2

1

3

1

zyx

8.

4

2

3

:

z

y

x

r 9. 80.41º 10. 48.70º 11. 68.48º 12. 5/6

13. Les rectes coincideixen. 14. Les rectes es tallen en un punt.

15. Les rectes són paral·leles. 16. 1a , 2

5b

17.a) Es creuen. b) Es tallen en el punt (0,3,3) c) Són paral·leles i no coincidents.

d) Són la mateixa recta.

18. a=3, )2,1,1( 19. m = 12, n = -3 20. Són paral·leles.

21. 22. 3/4a 23. 6m

24. 2a la recta r no està definida, 1a es tallen en un punt

2,1a es creuen.

25. 5

1b , 5a 26. 5m , 1n 27. 2 28. )1,5,1('A

9.5 1. a=0, b=5/3 i a=1,b=0 2. a) (2,1,-1) b) 30 graus. 3. )1,1,2(

4. 01322 zyx 5.

2

4

3

z

y

x

9.6 1. 023: zyx 2. )2,6,4(Q

3. Si 2,2

1m els plans es tallen en un punt.

Si 2m els plans 32 , són coincidents i 1 els talla.

Si 2

1m 21, són plans paral·lels i 3 els talla.

4. )1,8,1( Q

5. a) 032: zx b) 20

zy

x

6. 142

1`

2

8:

z

yxt

7. 0m i 4m

Page 162: Geometria Lineal per al Batxillerat - toomates.net · La història dels vectors. 1.10 Recopilatori d’exercicis. 1.11 Taula resum dels tres productes. 2 Varietats lineals. Rectes

Sèrie “Matemàtiques per al batxillerat”

Àlgebra Lineal:

http://www.toomates.net/biblioteca/AlgebraLineal.pdf

Geometria Lineal:

http://www.toomates.net/biblioteca/GeometriaLineal.pdf

Càlcul infinitesimal:

http://www.toomates.net/biblioteca/Calcul.pdf

Programació Lineal:

http://www.toomates.net/biblioteca/ProgramacioLineal.pdf

Compendium PAU TEC: Totes les Proves de la Selectivitat de Catalunya 1998 - 2019

http://www.toomates.net/biblioteca/Pautec.pdf

Compendium PAU CCSS: Totes les Proves de la Selectivitat de Catalunya 1998 - 2019

http://www.toomates.net/biblioteca/Pauccss.pdf

Links d'interès

Recull de Proves PAU Extremadura 2000-2017:

http://www.vicentegonzalezvalle.es/documentos/Examenes_selectividad_A4.pdf